You are on page 1of 130

Location:Emergencyroom

Vitalsigns:BP:90/60mmHg,HR:128/minregular,Temp:100.0F,R.R:30/minrapidandshallow
C.C:Vomitingsandabdominalpain.
HPI:
A20yroldwomanpresentstoE.Rwith5episodesofvomiting,abdominalpain,weaknessandincreasing
drowsinessofonedayduration.Duringthelast2monthsshehasnoticedincreasedthirstandincreased
urination.Theabdominalpainisdiffuse,45/10inseverity,constant,nonradiatingandtherearenoaggravating
orrelievingfactors.Vomitingisnonbloody.Shehasnoothermedicalproblems.Shehasnoknowndrug
allergies.Sheisnotonanyprescriptionoroverthecountermedications.Sheisnotasmokeroralcoholic,and
deniesIVdrugabuse.ShehasafamilyhistorypositiveforType1DiabetesMellitus.Herfather,andpaternal
uncleandgrandfatherarealldiabetics.
Reviewofsystems:
Shedeniesweightchanges,fever,chills,nightsweats,diarrhea,constipation,skin,hair,ornailchanges,blurry
vision,acutebleeding,easybruising,indigestion,dysphagia,changesinbowelmovements,bloodystools,
burningonurination,recenttravel,illcontacts,vaginaldischargeoritch,pregnancy,heatorcoldintolerance,
drugoralcoholuse.Lastmenstrualperiodendedfourweeksago,wasnormalinflowandduration.
Howdoyouapproachthiscase?
Firstquicklyexaminethepatient
General
HEENT
Neck
Heart
Lungs
Abdomen
Extremities
Herearetheresultsoftheexam:
General:Patientisinmildtomoderateabdominalpainandappearsverydistressed.
HEENT:Verydrymucusmembranes,noJVD,EOMareintact.Restisunremarkable.
Lungs:CleartoauscultationB/L.
Heart:Completelynormalexcepttachycardia.
Abdomen:Soft,nontender,normalbowelsoundsandnoguardingorrigidity.
Extremities:Noedema,calftenderness,butweekperipheralpulses.
Discussion:
Now,makeamentalchecklistofdifferentialdiagnosis,i.e.
1.Abdominalpathologylikeappendicitis,gastroenteritis,pancreatitis,acuteintestinalobstructionetc.
2.Menstrualsymptomsorpregnancyrelatedcomplications
3.DKA(Basedonthefamilyhistoryandpresentingclinicalfeatures)
4.NonketoticHyperosmolarstate
5.Alcoholicketoacidosis
6.Drugintoxication
Orderthefollowingstat:
Pulseoximetry,statandcontinuous
Oxygen,inhalation,continuous
IVaccess,stat
Cardiacmonitor,stat

Normalsaline,0.9%NaCl,continuous,stat(Thispatientisseverelydehydrated.Sheishypotensiveand
tachycardic.So,sheneedsIVfluids.)
Fingerstickglucose,stat
Results:
Pulseoxymetryshowed96%onroomair
Fingerstickglucoseshows600mg/dL
Order:
Urinepregnancytest,stat
CBCwithdifferential,stat
BMP,stat
Calcium,serum,stat
EKG,12lead,stat
Serumamylase,stat
Serumlipase,stat
UA,stat
ABG,stat
Serumosmolality,stat
Serumketones,qualitative,stat
Regularinsulin,IV,continuous
Phenergan,IV,onetime(fornausea)
Discontinueoxygen
Okherearetheresults:
Urinepregnancytestisnegative
WBC10,000/Landnormaldifferential
Sodiumis129,Potassiumis5.0,Chlorideis90,Co2is14,calciumis8.0,andabloodsugarof600mg/dL
EKGsinustachycardia,nothingconcerning
SerumAmylasemildlyelevated
SerumLipaseWNL
UAshowed4+sugar,2+ketonesbutnoevidenceofinfection
SerumOsmolality305
SerumKetoneshigh
ABGshowedmetabolicacidosis,compensatedbyrespiratoryalkalosis(pHof7.3)
Howdoyouapproachthiscase?
So,thispatientmostlikelyhaseitherDKAorNonketotichyperglycemia.Thediagnosisisbasedonclinical
features,elevatedbloodsugars,andincreasedaniongap.Toconfirmthediagnosisweneedtoorderserum
ketonesandserumosmolality,asabove.Shehaspseudohyponatremiai.e.secondarytoelevatedbloodsugars.
Treatmentofhyperglycemiaresolvesherhyponatremia.
Revieworders:
Admitthepatienttotheintensivecareunit
NPO
Bedrest
VitalsasperICUprotocol
Urineoutput
KCl,IV,continuous
HbA1Clevel,routine
Phosphorous,serum,stat(optional)
Followthepatientwith
1.BMPQ24hours,thenQ812hours,thenQday

2.ABGQ2hoursx2
After4hrs
1.Stop0.9%NSandgive1/2Normalsaline,IV,continuous
MonitorpotassiumdeficiencyandaddIVpotassiumchlorideasneeded
Considerantibioticsiftheprecipitatingcauseisaninfection,getachestXray,obtainbloodcultures,U/Aand
urinecultures.
Oncenauseaisdecreased,startoralfluids.
Oncethepatientisstabilizedtransfertoward/floor.
Duringdischarge:
D/CIVinsulin,IVfluids,cardiacmonitor
NPHinsulin,subcutaneous,continuous
Regularinsulin,subcutaneous,continuous
Diabeticdiet(Diet,Americandiabeticassociation)
Advancediet
Counseling:
Diabeticteaching
Patienteducation,diabetes
Diabeticfootcare
Homeglucosemonitoring,instructpatient
Noalcohol
Nosmoking
Safesex
Noillegaldruguse
Regularexercise
Seatbeltsuse
*Followupappointmentin10days
Discussion:
DiagnosisofDKAisbasedonanelevatedbloodglucose(usuallyabove250mg/dl),alowserumbicarbonate
level(usuallybelow15mEq/L),andelevatedaniongap,anddemonstrableketonemia.Bothamylaseandlipase
areoftenelevatedinpatientswithDKAbyanunknownmechanism(donottoconfusewithpancreatitis).
DiagnosisofHyperosmolarhyperglycemicisbasedon:serumglucoselevelsinexcessof600mg/dl,serum
osmolalitygreaterthan330mOsm/kg,absentorminimalketonemia,arterialpHabove7.3,andaserum
bicarbonateabove20mEq/L.Hyperosmolarhyperglycemicstateischaracterizedbyseverefluidandelectrolyte
depletionduetotheosmoticdiuresisproducedbytheextremelevelsofglucoseintheserum(often>1000mg
/dL).
Hydration:PatientswithDKAareprofoundlydehydratedandforemostinthetreatmentofDKAisrestorationof
theintravascularvolume.Estimatesoffluiddeficitsinthedecompensateddiabeticis4to10liters(usually56
liters).Initially,onetotwolitersofnormalsalineisgivenasbolus,followedby500mL/hforthefirstfour
hoursfollowedby250mL/hforthenextseveralhours.Thisinitialmanagementshouldbeguidedbythe
patient'sgeneralconditionandresponse,withmoreorlessfluidasindicated.Afterthefirst34hours,asthe
clinicalconditionofthepatientimproves,withstablebloodpressureandgoodurineoutput,fluidsshouldbe
changedto1/2normalsalineat250500ccanhourfor34hours.Ongoingreassessmentiscritical.
Insulin:Thestandardinsulindoseisaninitialbolusof0.1U/kgbodyweightfollowedbyacontinuousinfusion
atarateof0.1U/kgperhour.Whentheglucoselevelsbegintoapproach250mg/dl,insulininfusionsare
continued,butthefluidcompositionischangedtoinclude510%dextroseinwatertoavoidhypoglycemia.

Potassium:Potassium:Regardlessoftheserumpotassiumlevelattheinitiationoftherapy,duringtreatmentof
DKAthereisusuallyarapiddeclineinthepotassiumconcentrationinthepatientwithnormalkidneyfunction.
Potassiumreplacementisindicatedinallpatientswiththefollowingfeatures:Kof<5.3,noEKGchanges,and
normalrenalfunction.
BicarbonateTherapy:TheuseofbicarbonateinthetreatmentofDKAishighlycontroversial.Current
recommendationsforbicarbonatetherapyareasfollows.Useofbicarbonateisconsideredunnecessarywhen
thebloodpHisgreaterthan7.1.
PhosphateisnormallyanintracellularsubstancethatisdraggedoutofthecellduringDKA.Similarlyto
potassium,atpresentationtheserumlevelmaybenormal,high,orlowwhilethetotalbodysupplyisdepleted.
Despitethisdepletion,replacementofphosphatehasnotbeenshowntoaffectpatientoutcomeandroutine
replacementisnotrecommended.
Primarydiagnosis:
DKA

Location:Emergencyroom
Vitals:BP100/60mmHgHRis50/min,regularRRis10/minTemp.37C(98.6).
HPI:
28yroldwhitefemaleisbroughttoERinunconsciousstate.Familyreportsthatsheisaveryhealthyfemale,
hasnomedicalproblems,notonanymedications,anddidnotfindanyemptybottles.Shehasnoallergies.She
doesn'tsmokeordrinkalcohol.Shehasaboyfriend.Shehasneverbeenpregnant.Herfatherisveryhealthy
exceptborderlinehypertension.Motherhasdiabetes.Nootherhistoryisavailable.
Howdoyouapproachthispatient?
Discussion:
StepI:Emergentmanagement:Thispatientishemodynamicallyunstable,soA,B,C,Disthemostimportant
componentofthemanagementofthispatient.
A:Airwaysuction,pulseoximetry,stat,andcontinuousmonitoring,O2
B:EndotrachealintubationisindicatedinpatientswhocannotprotecttheirairwayorifO2saturationdoesnot
improvewithO2nasal/facemask,orPaO2<55,orPCO2>50onABG.
C:IVaccesscontinuouscardiacmonitorplaceaFoleyobtainafingerstickglucose.
D:Drugs:Administerthiamine,dextrose50%,andnaloxoneallareIVbolusonetimedose
Exam:
Respiratory(assessthebreathingpattern)
Orderreview:
Suctionairway,stat
Pulseoximetry,statandcontinuous
Oxygen,inhalation,statorIntubation
IVaccess,stat
Cardiacmonitor,continuous
Fingerstickglucose,stat
Thiamine,IVstat,onetime
Dextrose50%,stat,onetime
Naloxone,IVstat,onetime
Normalsaline0.9%NaCl,stat,continuous
ABG,stat
*Sheisslightlyawakewiththeabovetreatment
StepII:PhysicalExamination:
General
HEENT/Neck
Heart/CVS
Skin
Chest/Lung
Abdomen
Extremities
Neurologicalexam
Results:
Onexaminationsshefoundtohavepinpointpupils.Sheisverydrowsy.
So,shehasbradycardia,hypotension,andpinpointpupils,whichareclassicsymptomsfornarcoticoverdose.

StepIII:DiagnosticInvestigations:
EKG12lead,stat
CBCwithdifferential,stat
BMP,stat
CXR,portable,PA,stat
LFT's,stat
UA,stat
Urinetoxicologyscreen,stat
BHCG,serum,qualitative,stat
Bloodalcohol,stat
InitialTreatment:
NGtube,gastriclavage,stat(whichrevealedpillfragments)
Activatedcharcoal,oral,onetime
Naloxone,IV,stat,continuous
StepIV:
Decisionaboutchangingpatientlocation
MovepatienttoICU
NPO
Bedrest,complete
Urineoutput
BMP,nextday
*Oncethepatientisbetter
D/Coxygen,NGtube,cardiacmonitor,IVfluids,andnaloxone
Regulardiet
StepV:Educatepatientandfamily:
Psychiatryconsult,stat(Reason:28yearoldwithsuicideattempt)
Suicideprecautions
Suicidecontract
Patientcounseling
Reassurance
Noalcohol
Nosmoking
Safesex
Noillegaldruguse
Regularexercise
Seatbeltsuse
*Startthepatientonantidepressantifneeded
FinalDiagnosis:
Narcoticoverdose
Discussion:
Orthostatichypotensionresultingfrommildperipheralvasodilationiscommon.However,persistentor
severehypotensionshouldraisethesuspicionofcoingestants.
Inallpatientswithmoderatetoseveretoxicity,itisimportanttoobtainbaselinestudies,includingaCBC
withdiff,basicmetabolicpanel,LFT's,ABG,andCK(Creatinekinaselevel).
Positiveurinedrugscreensareobservedupto3648hourspostexposure.
A12leadEKGshouldbeobtainedonallpatientswithintentionaloverdose,asthereisalwaysa
possibilityofcardiotoxiccoingestants.

Chestxrayisimportanttoruleoutanypulmonaryedemaoraspirationespeciallyinapatientwithan
unprotectedairway.
NaloxoneshouldbegiventopatientswithsignificantCNSand/orrespiratorydepression.
ContinuousIVinfusionofnaloxoneisverysafeinpatientswhowerenotopioiddependent.However,in
patientswhoareopioiddependentthispracticeisdangerousandmayprecipitatewithdrawalsymptoms.
Activatedcharcoalshouldbeadministeredtoallpatientswithopiateintoxicationfollowingingestion.
Becauseofthedelayedgastricemptyingproducedbyopiateintoxication,itiseffectiveeveninpatients
whopresentlatefollowingingestion.Orogastriclavageisindicatedifthepatientpresentswithinonehour
ofingestion.
Allpatientswithsignificantrespiratorydepression,recurrentsedationshouldbeobservedinthehospital
foratleastaperiodof1224hours.Mostphysiciansadmitthepatientsiftheyrequireaseconddoseof
naloxone.Patientsshouldhavecontinuouscardiorespiratorymonitoring.

Location:office
Presentingcomplaint:A25yearoldAfricanAmericanmalepresentswithjaundice.
Vitals:Pulse:95/min,B.P:110/75mmHg,Temp:98.8F,R.R:16/min,Height:72inches(180cm),Weight:72Kg
(158.4lbs)
HPI:A25yearoldAfricanAmericanmalepresentstotheoutpatientclinicwiththesuddenonsetofjaundice
anddarkcoloredurine.Hecomplainsofbackpainandfatigue.Thepatientisafebrileanddeniesrecenttravel.
Hedoesnotsmoke,drinkalcoholoruserecreationaldrugsandusescondomswheneverheengagesinsexual
activity.Onfurtherquestioning,itisrevealedthathetookTMPSMZfordiarrheaafewdaysago.Anuncle
hasahistoryofsometypeofblooddisorder.
Hospitalization/Procedures
OtherMedicalProblems
Allergies
CurrentMedications
Vaccinations
FamilyHistory
SocialHistory
SexualHistory
OccupationalHistory
Recreational

None
None
NKDA
None
Uptodate
Motherdiedattheageof60yrsduetoMI.Fatheris
aliveandhealthyattheageof65yrs.Nosibling.
Heissinglebuthasagirlfriend.Deniestobacco,
alcoholanddruguse.
Heissexuallyactivewithhisgirlfriend.
Restaurantowner.
Heplaysbasketballandenjoystraveling.

ReviewofSystems:
General
Skin
HEENT
Musculoskeletal

seeHPI
pallor,itch,norashes
Icterus
Nomuscleachesorjointstiffness

Cardiorespiratory
Genitourinary
Abdominal

Withoutcomplaint
Darkcoloredurine,deniesdysuria
seeHPI

Howtoapproachthiscase:
Thisisapresentationofjaundice.Jaundicecanbeduetohemolyticcausesanddisordersinvolvingtheliveror
biliarytracts.
Firstperformaphysicalexamination:
Generalexamination
Skin
Lymphnodes
HEENT/Neck
Heartexam
Lungexam
Abdominalexamination
Extremities
Neuro
ResultsofPE:
General:IcterusisnotedonscleraHealsoappearspallor.Abdominalexamination:nomasses,tenderness,or

organomegaly.Normalbowelsounds.RestoftheexamisWNL
RoutineOrders:
CBCwithdifferential,routine
Basicmetabolicpanel,routine
LFTs,routine
Prothrombintime,routine
ResultsofLabs:
WBCcount:
RBCcount:
Hemoglobin:
Hematocrit:
Plateletcount:
MCV:
MCH:
MCHC:

8200/mm3
1.8million/mm3
9g/dL
33%
200,000/mm3
98cumicrons
28pg/RBC
35gHb/dL

WBCdifferential:
Segmentedneutrophils:
Juvenileneutrophils:
Lymphocytes:
Monocytes:
Eosinophils:
Basophils:

72%
2%
18%
5%
2%
1%

Peripheralbloodsmear:
Normochromicnormocyticerythrocytes,bitecellsarealsopresent.Leukocytesandplateletsarenormalin
numberandmorphology.
LFTs
Bilirubin,serum,total
Bilirubin,serum,direct
Aspartatetransaminase,serum
Alaninetransaminase,serum
Alkalinephosphatase,serum
Protein,serum,total
Prothrombintime

5mg/dL
0.5mg/dL
25U/L
20U/L
182U/L
7.2g/dL
11sec

Discussion:
Thispatienthasnohistoryoffeverorabdominalpainandalsonoriskfactorsforhepatitis.Thus,acute
cholangitisorhepatitisisunlikelyasacauseofhisjaundice.Normalabdominalexaminationwithabsenceof
tendernessfurtherexcludesliverorbiliarytractpathology.Thepresenceofpallor,anddarkcoloredurine
suggestsanintravascularhemolyticcauseofthejaundice.Otherpointstoelicitarepositivefamilyhistoryand
exposuretosulphadrugs.
NormalLFTsruleoutliverorbiliarytractdisease.Elevatedlevelsofindirectbilirubinareacluetowards
hemolysis.CBCshowsanemiaandpresenceofbitecellsonperipheralsmear.Thissuggeststhatthejaundiceis
duetohemolysis.Nextdothefollowingteststoconfirmthepresenceofhemolysisandtodetermineifitis
intravascularorextravascular.
RoutineOrderreview:
Admitthepatientinfloor/ward

IVaccess,stat
Normalsaline,IV,continuous
Diet:Regulardiet(avoidfavabeans)
Activityambulationatwill
Reticulocytecount,stat(toconfirmthatjaundiceishemolyticasreticulocytecountiselevatedincasesof
hemolysis)
Calculatereticulocyteproductionindex(tocorrectreticulocytecountforthedegreeofanemia)
Serumhaptoglobin
LDH,serum,stat(itiselevatedinintravascularhemolysis)
Urinalysis,stat(todetecthemoglobinorhemosiderinuria)
Typeandcrossmatch,stat(for2unitsofblood)
PRBC,transfuse,stat(packedRBCtransfusion)
*RepeatHbandhematocrit(H&H)in12hours
Results:
BUN
Serumcreatinine
RestoftheBMP
SerumLDH
Serumhaptoglobin
Urinalysis

12mg/dL
0.6mg/dL
WNL
400IU/L
20mg/dL
normal

ElevatedreticulocytecountconfirmsthepresenceofhemolyticanemiaandelevatedLDHwithlowhaptoglobin
indicatethatthehemolysisisintravascular.Apositivefamilyhistory,historyofexposuretosulphadrugsand
presenceofbitecellsonperipheralsmearareallsuggestiveofG6PDdeficiencyanemia.G6PDdeficiencyis
confirmedbyG6PDassay.Otherhereditarycausesofhemolyticanemiaaresicklecellanemia,thalassemias
andhereditaryspherocytosis.
Insicklecellanemia,peripheralsmearshowssickleshapedRBCsandinhereditaryspherocytosisRBCsexhibit
alossofcentralpallor.Thalassemiasproduceamicrocyticpicturewithtargetcells.Forthalassemiasandsickle
cellanemia,hemoglobinelectrophoresisprovidesusefuldiagnosticinformation.Autoimmunehemolytic
anemiaisanimportantnonhereditarycauseandinsuchcasescoomb'stestisanimportantdiagnostictool.
Intravascularhemolysiscanalsobeapartofthromboticthrombocytopenicpurpura,butinsuchcases
fragmentedRBCsarefoundonperipheralsmearandplateletcountsarealsolowalongwithrenalimpairment.
RegardingtreatmentofG6PDdeficiencyanemia,allaffectedindividualsshouldavoidexposuretodrugswith
oxidantpotential.Heterozygousfemalesshouldalsoavoidexposuretosuchdrugsduringpregnancyand
lactationastheymaytriggerhemolysisinthefetusorneonate.Transfusionsareneededwhenanemiaisvery
severeduetoimpairedcompensatoryerythropoiesis.
RoutineOrder:
G6PDblood,quantitative,stat
Coomb'stest,direct,stat(optionaldependingonthescenario)
Result:
G6PDlevelsarelow(levelscanbenormalduringorimmediatelyaftertheacutehemolyticepisode,therefore
youcanrepeatthetesttoconfirmthediagnosis)
Orderreview:
Scheduleanappointmentafter2monthsandatthattimereevaluateG6PDassay
Reassurance
Patientcounseling
Limitalcoholuse
Regularexercise

Safesexcounseling
PrimaryDiagnosis:
G6PDdeficiencyanemia

Location:Office
Presentingcomplaint:A2yearoldboyisbroughtwithcomplaintsoffailuretogainweightandloosestools.
Vitals:Pulse:100/min,B.P:80/56mmHg,Temp:101.2F,R.R:18/min,Height:27.2inches(68cm),Weight:
7.2Kg(16lbs)
HPI:
A2yearoldCaucasianchildisbroughttothedoctor'sofficebyhisparentsforanevaluationofloosegreasy
stoolsandfailuretogainweightdespiteadequatenutrition.Hisotherproblemsareintermittentproductive
coughandrhinorrhea.Theparentsstatethathehasbeenwheezingandcoughingupapurulentexpectorationfor
4days.Theydenyfeverorchills.Hehashadpneumonia4timessincebirth.Deliveryofthechildand
neonatalcoursewereuncomplicated.Hewasbreastfeduntiltheageof4months.ThereisahistoryofCFin
thefamilyofbothofhisparents.
ReviewofSystems:
General:
Skin:
HEENT:
Musculoskeletal:
Cardiorespiratory:
Genitourinary:
Abdominal:
Development:
Vaccinations:

Norashesorlesions
Nasaldischarge
Nojointswelling
seeHPI
Nocomplaints
seeHPI
Delayed
Uptodate

Howtoapproachthiscase:
Thischildpresentswithfailuretothrive.Failuretothrivehasbothorganicandnonorganiccauses.The
etiologyoffailuretothriveinthischildismostlikelycysticfibrosis.Cluestocysticfibrosisinthispatientare
positivefamilyhistory,repeatedchestinfectionsandmalabsorptivediarrhea.
Order:
Completephysicalexamination(exceptbreast)
Results:
Thechildseemstobeemaciatedhisheightandweightarelowerthanexpectedforhisage.Chestexamination
showsgeneralizedhyperresonanceandscatteredcrepitationsbilaterally.Healsohaswheezingespecially
duringexpiration.
OrderReview:
Admittotheward
Pulseoximetry,statandevery4hours
IVaccess,stat
Sputumgramstain,stat
Sputumcultureandsensitivity,stat(TypesputumC&S)
Bloodcultures,stat
CBCwithdifferential,stat
BMP,stat
CXRPA/Lateral,stat
Sinusxray,routine
Sweatchloride,routine
72hourfecalfatestimation,routine

Treatment:
Oxygeninhalation,continuous(Ifsaturationare<92%onroomair)
AmoxycillinandClavulonicacid,oral,continuous(TypeAugmentin)
NebulizedAlbuterol,inhalation,continuous(4timesaday)
Multivitamintablets,oral,continuous
Chestphysiotherapy
VitalsQ6hours
D5NS,IV,continuous
Regulardiet
Ambulationatwill
Results:
CBCshowsneutrophilicleukocytosis
Sweatchlorideis85meq/L
BMPshowedlowsodiumandpotassium
CXRshowshyperinflationofbothlungfields.
XrayPNSshowsopacificationofparanasalsinuses.
Gramstainingofsputumdoesnotshowanypredominantorganism.
Sputumcultureispending.
72hrfecalfatestimationispending
Discussion:
CysticfibrosisisanautosomalrecessivedisordercommonlyaffectingCaucasians.Itsclinicalmanifestations
includeacuteorpersistentrespiratorysymptoms,failuretothrive,meconiumileus,diarrhea,rectalprolapse,
nasalpolyps,electrolyteoracidbasedisordersandhepatobiliarydisease.Diagnosisofcysticfibrosisismade
whenevidenceofCFTRdysfunctionispresentalongwithtypicalclinicalfeaturesorpositivefamilyhistory.
ElevatedsweatchlorideontwoseparateoccasionsisanevidenceofCFTRdysfunction.Otherimportanttests
aregramstain,cultureandsensitivityofthesputum,chestXray,Xrayoftheparanasalsinuses.24hourfecal
fatestimationshouldbeusedtodiagnosethemalabsorption.Genetictestingisnotrequiredtomakethe
diagnosisasitisassociatedwithmultiplemutations.
AllchildrenwhoaresuspectedtobesufferingfromCFshouldbeadmittedtothehospital.Evaluationshould
includebaselinetesting,accuratediagnosis,initiationoftreatmentandeducationofthepatientandparents.
Followupshouldbedoneevery23monthsformonitoring.History,physicalexaminationandstainingand
cultureofsputumorpharyngealswabarerequiredoneachfollowupvisit.Prophylacticimmunizationagainst
influenza,measles,andpertussisshouldbegiven.
Treatment:
AntibioticsaregivenwhenapatientofCFdevelopsacuteorsubacuteincreaseinsputumproduction,
cough,dyspneaand/orfever.Antibioticsareselectedaccordingtotheresultsofsputumculture.Sputum
culturesareperformedatleastonyearlybasistohelpidentifythebacteria,whicharechronically
inhabitingtherespiratorytract.Oralantibioticsareusedwhenexacerbationismild,whileIVantibiotics
areusedwhenexacerbationissevereandwhenbacteriaareresistanttooralantibiotics.ForS.aureus,
oralantibioticsusedarecephalexin,dicloxacillinoramoxicillinclavulanate.ForP.aeruginosa,
ciprofloxacinisusedasanoralantibiotic.IVregimenforP.aeruginosaisacombinationoftobramycin
andantipseudomonalpenicillinlikepiperacillin.
Bronchodilatorslikealbuterolorsalmeterolareusedinpatientswithairflowobstruction.
DNaseisusuallyprescribedforthosewithdailyproductivecoughplusairflowobstruction.
Combinationofphysiotherapyandexerciseshouldbeconsideredinpatientswithmoreretainedpurulent
secretions.
InhaledglucocorticoidscanbeusedinpatientsofCFwhohaveclinicalevidenceofairwayhyperactivity.
Oxygentherapyshouldbeconsideredinallpatientswhohaveevidenceofhypoxemiaatnightorrestor
pulmonaryHTN.

Lungtransplantationistheonlydefinitivetreatmentinpatientswithsevereinfectionsandgrossly
damagedlungswitha
FEV1of30%orlessofthepredictedvalue.
Nutrition:Recommendeddietisonewithhighproteinandhighcalories.Vitaminsupplementationoffat
solublevitaminsandpancreaticenzymereplacementarealsoveryimportant.
Resultsreview:
72hoursfecalfatiselevated.
Sputumculturegrewstaphylococcusaureus,sensitivetocephalexin.
OrderReview:
D/CAmoxycillinandClavulonicacidD/CIvfluids
Statcephalexin,oral,continuous
Influenzavaccine
Pneumococcalvaccine
Consultdietitian
Pancreaticenzymes,oral,continuous
Geneticcounseling
*Followupat23months
PrimaryDiagnosis:
Cysticfibrosis

Location:
Emergencydepartment
Vitalsigns:
AfebrileBP:140/85mmHgP:87/min,regularRR:22/min
CC:
Shortnessofbreath
HPI:
A64yearoldmancomestotheEDwith2monthsofprogressivelyincreasingshortnessofbreath.His
conditionhasacutelyworsenedoverthelast2daysandhenowhasshortnessofbreathatrest.Thesymptoms
areslightlyrelievedwhenhesitsupright.Hehasnochestorabdominalpain.Hehasmildcoughwithdeep
inspirationandhashadoccasionalpinkfrothysputum.Thepatientreportsweightgainoverthepast6months
butcannotquantifytheamount.Healsohaseasyfatigability,legswelling,anddecreasedappetite.Hehashad
nofevers,chills,nausea,vomiting,orheadaches.Bowelmovementsareregularandhehasnourinary
problems.Hereportsthathisdiethasconsistedmostlyofcannedfoodssincehiswifepassedaway9months
ago.
PMH:Includeshypertensionandtype2diabetesmellitusfor19years.ThepatienthadanMI6yearsagothat
wastreatedwiththrombolytictherapy.Healsohasahistoryofhypercholesterolemia.
Medications:Metoprolol,lisinopril,glyburide,simvastatin,aspirin,andisosorbidedinitrate.Thepatientreports
medicationcompliance.
Allergichistory:NKDA
SH:Thepatienthassmoked1packdailyforthepast40years.Heoccasionallydrinksalcoholonweekends.
Heisretiredandlivesalone.
FH:Fatherdiedofaheartattackatage70.Motherdiedatage68frombreastcancermetastasis.
Approachtothispatient:
Elevateheadofbed
Pulseoximetry,statandcontinuous
Oxygen,inhalationandcontinuous
IVaccess,stat
Cardiacmonitor
12leadECG,stat
Results:
Pulseoximetryshowsoxygensaturationof89%onroomair
ECGshowsnormalsinusrhythmat70/minandanoldQwaveininferiorleadswithnosignificantchangesfrom
priorECGs
Physicalexamination:
General
HEENT/neck
Heart
Lungs
Abdomen
Extremities
Results:

General:64yearoldmanwhoappearshisstatedage,inmildrespiratorydistress.
HEENT/neck:ModerateJVD.Oralmucosaismoist.Nothyromegaly.
Lungs:Bilateraldecreasedbreathsoundswithbibasilarcrackles.
Heart:Rateandrhythmareregular.S1andS2heard.S3galloppresent.
Abdomen:Soft,rightupperquadranttendernessispresent.Liveris1cmbelowtherightcostalmargin.Bowel
soundsarepresent.Nofreefluid.
Extremities:3+bilateralpittingedemauptotheknee,nocalftenderness,nocyanosis.
Approachtothecase:
Differentialdiagnosisofprogressive(chronic)shortnessofbreathincludesobstructiveorinterstitiallungdisease
andcardiomyopathy.Thispatientspresentationissuggestiveofacutedecompensatedheartfailure(likelydue
tohighsodiumintakefromdietarynoncompliance).
Order:
Chestxray,PAandlateralviews,stat
CBCwithdifferential,stat
BMP,stat
TroponinI,statandQ6hoursx2
LFTs,stat
NTproBNP,stat
Furosemide(Lasix)IV,statandcontinuous
Results:
Chestxrayisconsistentwithbilateralpulmonaryedema.CBCwithdifferentialandtroponinIarenormal.
BMPshowsNa138mEq/LandK4.0mEq/L.Bloodglucoseis380mg/dL.NTproBNPis1800pg/mL
(elevated).RemainderofthelabstudiesareWNL.
Orders:
Admittoward
Telemetry
Ambulateatwill
Lowsalt,lowcholesterol,diabeticdiet
Fluidrestriction
Input/outputmonitor
Dailyweights
Continueallhomemedicationsexceptglyburide
KCLoral,continuous(aslongasLasixgiven)
Startregularinsulinslidingscale,ACHS
AccuChekACHS
Give10unitsregularinsulinnow
Lovenox,SQdailyforDVTprophylaxis
BMP,nextday(tocheckrenalfunctionandelectrolytesduetofurosemide)
HbA1c,routine
Lipidprofile,routine
Echocardiogram,routine(toassessleftventricularfunction)
Patientshouldbefeelingbetterafterfurosemide
Hospitalday2:
Examinethepatient
Transthoracicechocardiogramshowsmoderateleftventricularhypertrophy,leftventricularejectionfractionof
40%,abnormaldiastolicfunction,andnormalvalves
ConsiderconvertingIVtooraldiuretics
Determineifthepatientisstablefordischarge(improveddyspneaandperipheraledema)
Assesstheneedforoxygentherapy.Thegoalsaturationis92%96%.Weanoxygenifpulseoxygensaturation

>92%.
Atthetimeofdischarge,considerthefollowing:
Patienteducation
Cardiacrehabilitationprogram
Smokingandalcoholcessationprogram
Regularexercise
Dietaryandmedicationcompliance
Followupin1week
Discussion:
CHFisaclinicaldiagnosis,andimaging(chestxrayand2Decho)andlaboratorystudiesareusedtoconfirm
thediagnosis.InitialtreatmentincludesIVloopdiureticswithmonitoringofrenalfunctionandelectrolytes.
Patientswithacuterespiratoryinsufficiencyshouldbetreatedwithsupplementaloxygenand,ifneeded,
noninvasivepositivepressureventilation.PatientswithrefractorysystolicheartfailuremayneedtemporaryIV
inotropicsupport(eg,dobutamine,milrinone).LongtermuseofACEinhibitors/ARBs,betablockers,and
mineralocorticoidreceptorantagonistsreducesmortalityinpatientswithchronicsystolicheartfailure.
ContinuingthesemedicationsduringanepisodeofacutedecompensatedHFdependsonwhetherthepatientwas
takingthesemedicationspriortopresentationandhemodynamicstatusduringthedecompensatedstate.
Primarydiagnosis:
Acuteexacerbationofheartfailure(diastolicandsystolic)

Location:EmergencyRoom
Vitals:BP:102/68inbotharms,HR:110/minregularrhythm,RR:16/minute,Afebrile
C.C:
A75yroldnursinghomepatientwithdecreasedurineoutputandalteredmentalstatus.
HPI:
A75yearoldNursinghomepatientisbroughttoERforonedayofalteredmentalstatusanddecreasedurine
output.Thepatientisunabletogiveanyhistory.Thenursinghomestaffsaysthathehasbeenhavingwatery
stoolsforthelast4days.Hehas710loosebowelmovementsperday.Stoolshavenofoulsmell,nomucous
orblood.Hehasbeendrinkingfruitjuicesandoralformulaforthelast3days,buthasnoturinatedinthelast
12hours.Theydeniedanyhistoryoffever,chills,chestpain,shortnessofbreathandabdominalpain.
PMH:
HispastmedicalhistoryissignificantfortypeIIDM,hypertensionandosteoarthritis.
SH:
Thepatientdoesnotsmokeordrinkalcohol.
FH:
Fatherdiedattheageof90withAlzheimer'sdiseaseandmotherdiedinmotorvehicleaccident.Onebrother
hasdiabetes.
Medications:
Currentmedicationsarelisinopril5mgoncedaily,NPHinsulin10Uinthemorningand15Uintheevening,
metoprolol,simvastatin,aspirin,ibuprofenandmultivitamins.Hisvaccinationsareuptodate.
All:Iodineandshellfish.
Howwouldyouapproachthiscase?
Basedonthehistoryyoumayconsiderdecreasedurineoutputsecondarytoacuterenalfailure.Alsoconsider
dehydrationleadingtoacuteprerenalfailure,diabetescausingpossibleintrinsicrenalcause(DM),andpost
renalobstruction(BPH).TheotherpossiblecausesofalteredmentalstatusinthispatientincludeDKA,
hypoglycemia,hypertensivecrisis,strokeetc.
Order:
Completephysicalexamination
Resultsofthephysicalexamination:
HEENT:PEARL,EOMintact,mucosaisdry,noJVDandmoderatelydehydrated.
Heartandlungsarenormaltoauscultate.
Abdomen:Scaphoid,soft,nontender,BSarehyperactive,noorganomegaly
Rectal:Prostateisnormalinsize,sphinctertonenormal,nogrossblood,andnomasses.
CNS:Patientisnotalertandnotorientedtotime,placeorperson.Otherwisetherearenofocalsensoryor
motordeficits.
Extremities:Thereisnoedema,cyanosisorclubbing.
Skinisdrywithpoorturgor,norash,petechiaeorbruises.
Nowhowwouldyouapproachthispatient?
Thispatientisdehydrated,sotheprimarythingstoorderare:
Pulseoxy,stat
STATIVAccess

STATNSfluidbolusfollowedbycontinuousdrip
STATFoleycatheter,youarenotsureofbladderresidualvolumeinthiselderlypatient
12leadEKG,stat
ABG,stat
CBCwithdifferential,stat
MgandPhosphate,stat
BMP(Na,K,Chloride,HCO3,BUN,Cr,Bloodsugar,andCa)statandQ8hrs
Urineanalysis,stat
Urinecultureandsensitivity,stat
Urinesodiumandcreatinine,stat
TheFoleyisplaced30mlofresidualvolumeisnoted.NSSIVfluidisrunning.Whatdoyoudonow?
Yourinitialassessmentofthepatient
1)Whatisthehemodynamicstatus?
Dehydratedpatientshouldbehydrated.Avolumeoverloadedpatientshouldbegivendiuretics.CVP(central
venouspressureshouldberecordedbyplacinganorderforcentralvenouslineinunstablepatients)
2)Whatisthetypeofrenalfailure?
Prerenal,renalorpostrenalwilldecidethemainmodeoftreatment.
3)Isthereaneedforurgenthemodialysis?
Theindicationsforurgenthemodialysisarerefractoryacidosis,refractoryelectrolytedisturbances,intoxicants,
volumeoverload(pulmonaryedema),pericardialrub,uremicencephalopathy,bleedingtendencies,and
arrhythmias.
4)Whatisthemostlikelyetiology?
LabsrevealaBUNof90,creatinineof2.3,potassiumof5.6andmildprotenuria.Sohowwouldyouproceed?
IftheBUN(Normalis720)andcreatinine(0.6to1.2)iselevated,thenthediagnosisofrenalfailureis
confirmed.ChecktheelectrolytesthatarepartoftheBMP.Ifthepotassiumleveliselevated(normalis
3.5to5)thengiveD5/Insulinandrecheckthepotassiumlevel.Ifthepotassiumishighalwaysgetthe
EKG.Ifthepotassiumislow,thenreplacethepotassiumcautiouslyandrecheckthelevel.
IfthepatientistakingnephrotoxicdrugslikeNSAIDs,stopthedrugsimmediately.Ibuprofenisan
NSAIDanditaggravatestherenalfailure.ACEIsareindicatedinDMtype2fortheirrenalprotective
effect,butinestablishedrenalfailurewithacreatinineofmorethan2.0to2.5theiruseshouldbe
restricted.
Nowit'sthetimetodeterminethecauseoftherenalfailure.1stdeterminewhetherthecauseislowrenal
perfusion(prerenal)(or)disordersofparenchyma(renal)(or)obstructedurineflow(postrenal).
Thereare3waystodifferentiateprerenalfromrenalcauses.Thebestwayofassessingisbycalculating
FENa(FractionalExcretionofSodium).
Prerenal:
FENa<1
Spoturinesodium<20meq/L
DisproportionateincreaseofBUN/Crratio>20:1
Renal:
FENa>1
Spoturinesodium>20meq/L
ProportionateincreaseofBUN/Crration<20:1

Ashortexplanationtounderstandthecauses:
PrerenalazotemiacanleadtoATN,whichisthemostcommoncauseoftheacuterenalfailure.Twobasic
mechanisms:
a)Decreasedeffectiveintravascularvolume(includesthirdspaceloss)
b)Decreasedcardiacoutput.
Renalcauseshavespecificetiologiesrangingfrominfectioustotoxinmediatedandcanbetreatedwith
specificetiologydirectedtherapy.
Postrenalorobstructivemustbepromptlytreatedtoavoidanydamagetothekidney.Twomostcommon
areBPHandnephrolithiasis.
Herethecasescenarioisofaprerenalfailuremostprobablyduetodehydrationresultingfromdiarrhea.
Orderroutine:
Discontinuelisinopril
Discontinueibuprofen
Continuerestofhiscurrentmedications
Transfertothefloor/ward
Vitalsq2h
24hrurineprotein
Diabeticandrenaldietwith100%handassistance(Onlyifthepatientisawake)
Completebedrestuntilhismentalstatusreturnstobaseline
Heparin5000USQQ12hrstopreventDVT
Renalultrasound,routine.Statorderisplacedonlyifyoususpectaninfective/renal/obstructivecause.
Dailyweights
Strictinputandoutput
Acuchecks(bloodsugar),QID(4timesaday)
HBA1Clevels,routine
Slidingscaleinsulin
Finallytreatment:
Inprerenalthegoalofthetreatmentistoincreasetherenalbloodflow.Ifthepatientisdehydrated
continueI.Vhydration.Ifthepatientdoesnotrespondtofluids,startLasix(furosemide)toincreasethe
urineoutput.Ifthecauseoftherenalhypoperfusionisheartfailureyoucanstartdobutamineand
dopamine.
Theuseofsodiumbicarbonate(typeNaHCO)israrelyneededunlessthepatientisinsevereacidosisi.e.
pHof<7.2.
Revieworders:
Oncethepatient'smentalstatusisimproved
Discontinue(D/C)bedrest
Outofbedtochair
D/CFoleycatheter
Continuerenaldiet
Plentyoforalfluids
Otherconsiderations:
PericardialruborralesChestXray(BothPAandlateralview)
FeverBloodcultures,Immediaterenalultrasound,urinecultureandstartantibiotics(Ciprofloxacin)
Treathypertension
Obstructionorderurologyconsult.Beforegettingaconsultpleaseorderurgentrenalultrasoundand
don'tforgettocatheterize.

Patientmaytakedaystoweekstorecover
Recoverydiuresisshouldbetreatedwithhalf(0.45%)NSandfrequentelectrolytemonitoring.
Primarydiagnosis:
Acuterenalfailureorprerenalazotemia

Location:AmbulatoryClinic.
Vitals:
Temperature38rectally,heartrate155/min,bloodpressure95/55mmHg.Thepatient'sweightis7.2kg.
C.C:Vomitingsanddiarrhea
HPI:
Thepatientisasixmontholdwhitefemalewhoisbroughtinbyhermotherforevaluationofathreeday
historyofvomitingandprofusediarrheaandlowgradetemperatures.Motherbringshertotheclinictoday
becausetheinfanthasbecomeincreasinglyirritableandfussywhensheisawakeandtodayhasbecomemuch
moresomnolentanddoesnotwanttofeed.Shehasnothadanywetdiaperssincetheeveningbefore.However,
shestillishavingbowelmovements.Motherreportsthatthepatienthashadapproximatelyeighttotenbowel
movementsadayforthepasttwodays.Shehasbeengivingtheinfantanincreasednumberofbreastfeedsat
approximatelythreehourintervalshowever,thechildfrequentlyvomitsthemilkandalsohasrunnydiarrhea.
Thebabyhasrefusedtotakeanyofthericecerealwhichshehasbeenintroducedtoforthepastmonth.Mother
reportsthatthevomitingisnonbilious.Itmainlyconsistsofpartiallydigestedmilk.Itisnotprojectileanditis
nonbloody.Thediarrheaiswateryandyellowandtheredoesnotappeartobeanybloodormucusinthestool.
Themotherreportsthatthebaby'stemperaturehasbeenashighas101.Themotherreportsthatathervisitto
thedoctor'saboutthreeweeksagoforherimmunizationsshewasalmost8kg.
PMH:
Thepatientwasdeliveredat40weeksgestationbynormalspontaneousvaginaldelivery.Shehadnobirth
complicationsandwasdischargedhomewithhermother.Thispatienthasbeenmeetingherdevelopmental
milestonesappropriately.Sheisabletorolloverfronttobackandbacktofront.Shebabblesandcoosandcan
situprightforbriefperiodswithminimumsupport.
IMMUNIZATIONSareuptodate.Shejusthadhersixmonthvaccinationsabouttwoandahalfweeksago.
Socialhistory:
Sheliveswithhermotherandfatherandathreeyearoldsibling(brother)whoattendsdaycareonedayaweek.
Theparentsarenonsmokers.Therearesickcontactsinthehouseastheolderbrotherhadadiarrhealillness
earlierlastweek.Thefamilyhascitywater.Theyhavenottraveled.
Reviewofsystems:ispositiveforirritability,decreasedurineoutput,decreasedPOintake,increased
somnolence,anddecreasedphysicalactivity.Reviewofsystemsisotherwisenegative.
Howdoyouapproachthisbaby:
General
HEENT
Neck
Lungs
Heart
Abdomen
Genitourinary
Skin
Extremities
CNS
Herearetheresults:
Ingeneral,thepatientisawelldeveloped,wellnourishedwhitefemalelyingsomewhatlistlesslyinher
mother'sarms.Shedoesnotfightagainsttheexaminerduringthephysicalexamination.HEENT:Anterior
fontanelisopenanddepressed.Posteriorfontanelisclosed.Mucousmembranesaredry.Eyesareslightly
sunkenwithdarkcirclesunderneaththem.Tympanicmembranesarepalewithgoodlightreflexbilaterally.

Neckissupple.Thereisnoevidenceofmeningealirritationtoneckflexionorextension.Thereisnocervical
lymphadenopathy.Pupilsareequal,roundandreactivetolight.Oropharynxisclear.Cardiovascular:Heart
rateisregularrhythmbutsomewhattachycardic.ThereisaII/VIsystolicejectionmurmuralongtheleftsternal
border.Lungsarecleartoauscultationbilaterally.Abdomenissoft,nondistendedandnontender.Thereare
hyperactivebowelsoundsinallfourquadrants.Extremities:Pulsesare+2inallfourextremities.Capillary
refillisabout34seconds.GU:Normalfemalegenitalia,milddiaperdermatitispresent.Neurologic:The
patientissomewhatlistlessandirritablebutafteratimeisabletobeconsoled.Skinshowsdecreasedturgor.
Therearenorashes.
DIFFERENTIALDIAGNOSISforthischildwithacutediarrheaaswellasvomiting:Atthetopofthelistisan
infectiousgastroenteritiswiththetwomaingroupsbeingviralgastroenteritiswithetiologiesincluding
rotavirus,enterovirus,adenovirusandNorwalkagentandthebacterialenterocolitisagentsincludingShigella,
Salmonella,Yersinia,Campylobacter,enteroinvasiveE.coli,enteropathogenicE.coli,andC.difficilecolitis.
Otherpossibleetiologiesofacutediarrheaincludeextraintestinalinfectionssuchasotitismediaandurinarytract
infections,andGIcausesincludingintussusception,appendicitis,andhyperconcentratedinfantformula.Other
causesincludeantibioticinduceddiarrhea,vasculitidessuchhasHenochSchonleinpurpura,renaldiseases
includinghemolyticuremicsyndromeandtoxicingestionsincludingiron,mercury,leadandfluorideingestion.
Thispatient'sclinicalpresentationincludes(1)asickcontactwithathreeyearoldsiblingwhohadselflimited
gastroenteritissymptomsthepriorweek,(2)lowgradetemperature,(3)thelackofanyprecedinguseof
antibiotics,and(4)nonbloodystools.Themostcommoncauseforthispictureisacuteviralgastroenteritis.
Howdoyouproceed:
LABORATORYSTUDIES:Dependingonhowdehydratedthepatientappearsclinically,onemightforegoany
laboratorystudiesandjusttreathersymptomaticallywitheitheroralelectrolytereplacementsolutionsorIV
rehydrationintheclinicsetting.However,basedonherchangeinweight,itappearsthatshehaslostabout7
10%ofherbodyweightandisshowingclinicalsignsthatsheissignificantlydehydrated.Therefore,
appropriatelabstoobtainwouldbeabasicmetabolicpanelandaCBCwithdifferential.Stoolshouldbeheme
checkedandifitispositive,thenstoolsamplescouldbesentforculture.Iftherehadbeenapasthistoryof
antibioticuse,thenaC.difficileantigencouldbesenttoruleoutCdifficilecolitis.Ifthepatientisbeingseen
duringthewintermonths,thenstoolcanbesentforrotazymeassaysincerotaviruscancauseupto65%of
infantdiarrheaduringwintermonths.Stoolcanalsobesentforfecalleukocytes,whichalsohelptoindicate
thatitisaninfectiousorinflammatoryprocess.Becausethischildismoderatelydehydrated,aurinalysisshould
alsobesenttolookforurinarytractinfection.
ORDERS:Labsasabove.Thepatientshouldbegivenafluidboluswithnormalsaline20cc/kgIVuntilthe
labscomeback.Arepeatfluidboluscanbegivenaswell.Thepatientshouldbeallowedtofeedastolerated.
Inthiscase,thepatientisbreastfedsoordersforbreastfeedingadlib(frequentsmallfeedings)wouldbe
appropriate.Itcanbehelpfulifthemothercanfeedtheinfantinsmallermorefrequentfeedingswhetherbreast
fedorbottlefed.Inaddition,bottlefedinfantscanalsobegivenanoralelectrolytesolutionagainsmall
frequentfeedingsarepreferabletodecreasestomachirritationandreflexvomiting.
Revieworders:
IVaccess
CBCwithdiff,stat
BMP,stat
Urineanalysis
IVNS,bolus
IVNS,continuous
Stoolhemecheck
Stoolforleukocytes
Stoolculture
Breastfeedingadlib
Herearetheresults:

Laboratorystudiesreturn.CBCshowsawhitebloodcountof11,hemoglobin14,hematocrit42.1,platelets
370,differentialshowssegmentedcells52%,lymphocytes20%,monocytes5%,eosinophils10%.Basic
metabolicpanelshowsasodiumof131,chloride106,potassium3.2,CO218,BUN22,creatinine0.6,calcium
9.8,glucose87.Acatheterizedurinespecimenyieldedascantamountofdarkamberclearfluid.Specific
gravity1.032,negativeforesterase,negativenitrates,noblood,15whitebloodcells,0redbloodcells,no
casts.Cultureeventuallyisnegative.
DISPOSITION:ThispatientshouldbeadmittedtothehospitalforIVfluidrehydrationbecausesheisgreater
than5%dehydrated,nottakingenoughoralfluidstomeethermaintenanceneeds,andshowingadeclining
mentalstatuswithirritabilityandlistlessness.
Revieworders:
Admitinward/floor
IVPotassium
VitalsQ4hours
RechecktheBMPnextday
Repeatphysicalexamforevery46hours
OncethepatientisadequatelyhydratedandtheBMPisnormal,dischargethepatienttohome.
Discussion:Acutegastroenteritisisaverycommonpediatricproblemandbecausemanyoftheetiologiesare
selflimited,theclinicianshoulddirectattentiontothepatient'soverallfluidandelectrolytestatusastop
prioritiesinmanagement.Attentionshouldbepaidtothepatient'svitalsignslookingfortachycardiaand
evidenceofweightloss.Thephysicalexamshouldfocusonhydrationstatusbylookingatmucousmembranes,
sunkeneyes,skinturgorandcapillaryrefill,whichinnormalchildrenislessthantwoseconds.Abdominal
examshouldfocusonwhetherthereisanydistention,tendernessormassesfrequentlybowelsoundsare
hyperactiveinviralorbacterialgastroenteritis.Directinspectionofthestoolcanbehelpfulasbloodyor
mucousstoolismoreoftenassociatedwithbacterialpathogensandwoulddirectinvestigationstowardsthose
etiologies.OnewouldsendastoolcultureforShigella,Campylobacter,SalmonellaandYersiniaaswellasthe
enteroinvasiveE.coli.Pathogensthatwillrequiremorethanjustsupportivetreatmentwithfluidsand
electrolytesincludeCampylobacterjejuniinfection,whichistreatedwitherythromycinCdifficilecolitis,
whichistreatedwithmetronidazoleandsystemicsalmonellosis,whichistreatedwithfluoroquinolonesor
azithromycinorthirdgenerationcephalosporinssuchasceftriaxoneorcefotaxime.Bactrimmayalsobestarted
empirically,butthereisincreasingresistanceofSalmonellatoBactrim.Inthosechildrenwhosediarrheaisdue
toGiardia,avarietyofdrugsareused:metronidazole,furazolidone,orquinacrine,forexample.
Historytakingshouldincludequestionsabouttravel,wellwater(sinceGiardiacancontaminatewellwater),
immunizationstatus,sickcontacts,anddaycareattendance(sinceCampylobacteroutbreakshavebeenfoundin
daycarecenters).Oneshouldalsoaskaboutthedurationofsymptoms,fever,andthenumber,character,and
colorofthestools,particularlywhetherthereisanybloodormucusinthestools.
Primarydiagnosis
Acutegastroenteritis

Location:office
Presentingcomplaint:A22yearoldwomanpresentswithhirsutism.
Vitals:Pulse:80/minBP:122/82mmHgTemp:98.7F,R.R:16/minHeight:162.5cmWeight:90kg(198lbs).
HPI:
A20yearoldobesewhitefemalecomestothephysicianofficewiththecomplaintofmalepatternbodyhair.
Shestatesthatbesidesfacialhair,hairisalsopresentonherchestandonthelowerabdomen.Herother
complaintsareamenorrheaforthelast4monthsandobesity.Shestatesthathermenseshavealwaysbeen
irregularsinceherfirstmensesattheageof14years.Sheneithersmokesnordrinksalcohol.Sheisnotusing
anyprescribedorrecreationaldrugs.Shehasneverbeensexuallyactive.Shedeniesanyothercomplaints.She
isallergictopenicillin.FH:Fatherishealthy.MotherhasaH/ODM.Shedoesnotuseanyrecreationaldrugs.
Sheisunmarried.ROSareunremarkable.
Howtoapproachthiscase?
Importantpointstonoteinthisyoungfemalearehirsutism,secondaryamenorrhea,andobesity.Thesefindings
pointtowardsthediagnosisofpolycysticovariandisease(PCOD).Otherpossiblecausesare
hyperprolactinemia,lateonsetcongenitaladrenalhyperplasia,andandrogensecretingtumors.Diseaseslike
Cushing'ssyndromeandhypothyroidismalsoneedtoberuledoutasacauseofobesity.
Thispatientisverystableandiscomingforevaluationfirsttimewithyou.So,sheneedscompletephysical
examination.Wealsoneedtoexamineherskinforpatternofhairdistribution,andgenitaliaforanyevidenceof
virilization.
Okfirstordertheexaminationpart:
Completephysicalexamination(or)
General,HEENT/Neck,Lymphadenopathy,Lungs,Heart,Abdomen,Pelvicexam,Extremities,Skin,and
Neuropsychiatric.
Herearetheresultsofyourexamination:
Youngobesefemale,notinacutedistress.Hairarenotedontheupperlip,chin,aroundnipples,andonlinea
alba.
Restoftheexaminationisnormal.
Discussion:
ThemostwidelyusedcriteriaforthediagnosisofPCOSis:
1.Clinicalorbiochemicalevidenceofhyperandrogenism
2.Menstrualdysfunction(Fewerthan69cyclesperyear)
3.Exclusionofothercommoncausesofhyperandrogenism
ManybiochemicalabnormalitiesareencounteredincasesofPCOS.Theseincludehighserumandrogens,high
serumestronewithnormalserumestradiol,highserumLHwithnormalserumFSH,andimpairedglucose
tolerance.Bothtotalandfreeserumtestosteroneconcentrationiselevated.Findingofpolycysticovarieson
USGisnonspecificforthediagnosisofPCOS.
Differentialdiagnosis:
PCOSisadiagnosisofexclusion.PCOSandidiopathichirsutismaccountformorethan95percentcasesof
hyperandrogenisminfemales.Othercausesarelateonsetcongenitaladrenalhyperplasia,ovarianandadrenal
tumors,drugsandhyperprolactinemia.Incasesofhyperprolactinemia,menstrualdysfunctionisprominent
withoutanyevidenceofhyperandrogenismandserumprolactinlevelsareelevated.24hoururinecortisol,and
17ketosteroidsareindicatedforsuspectedCushing'ssyndrome.Lateonsetcongenitaladrenalhyperplasiaisa
raredisorderandcanberuledoutbypostACTHserum17hydroxyprogesteronelevels.

Incasesofandrogensecretingtumors,therearesignsofvirilizationandserumLHconcentrationislow.Serum
testosterone(>150ng/dl)andserumdehydroepiandrosterone(>800ug/dl)levelsarehighinovarianandadrenal
tumorsrespectively.
Orderroutinelabs:
UrinetestingforbetaHCG(asshehashistoryofamenorrhea)
Serumtestosteronetotalandfree
SerumDHEAS
Serumprolactin
24hoururinecortisol
24hoururine17ketosteroids
SerumTSH
SerumLH
SerumFSH
Pelvicultrasound,routine
Followupvisitwhenresultsareavailable.
Resultsoflabs:
UrinetestingforbetaHCG:negative
Serumtestosteronetotalandfree:totalis100ng/dlandfreeis5ng/dl
SerumDHEAS:normal
Serumprolactin:10ng/ml
Serumcortisol:normal
SerumTSH:1microU/L
SerumLH:60IU/L
SerumFSH:15IU/L
Pelvicultrasound:Ovariesshowsperipheral"stringsofpearls"sign.
TREATMENT:
AllwomenwiththediagnosisofPCOSshouldbeinitiallyevaluatedformetabolicriskfactors.Themost
commonmetabolicabnormalityassociatedwithPCODistype2DMandimpairedglucosetolerance.
Measurementofweight,bloodpressure,andfastinglipidsarerecommendedinallpatients.2hourglucose
tolerancetestwith75goforalglucoseloadisindicatedinobesewomenwithPCOD.Routinemeasurementof
seruminsulinlevelsisnotindicatedforvariousreasons.
Weightreductioninobesefemalesanduseofdrugsthatdecreaseinsulinresistanceinbothobeseandnonobese
reversemanyabnormalitiesofPCOS.Thedrugsusedforthispurposearemetformin,troglitazone,andDchiro
inositol.UnopposedestrogenactionmayresultinendometrialhyperplasiaandendometrialCa.Todecrease
thisrisk,OCPsaregivenandprogestinpresentinthemantagonizestheeffectofestrogen.Anotherbenefitof
thistherapyisinhibitionofandrogenproductionandincreaseinsexhormonebindingglobulinlevels.OCPsare
thereforethetreatmentofchoiceforwomenwhodon'twanttoconceive.PatientswithPCOSwhohave
evidenceofDMshouldbetreatedwithmetformin.Metforminispreferableoverthiazolidinedionesforpossible
teratogenicty.OCPsareindicatedinconjunctionwithmetforminiftheydonotwanttobecomepregnant.For
hirsutism,hairisremovedbyshaving,electrolysis,lasertreatment,anddepilatories.Hairgrowthisslowedby
anOCPaloneoranOCPcombinedwithantiandrogen.Incasesofinfertility,otherpossiblecausesofinfertility
arefirstruledoutbyappropriatetestingandonlythenovulationinductionisattempted.Clomiphenecitrateis
usuallyusedforthispurpose.OtheragentsincludeGnRHandexogenousgonadotrophins.Patient'swithLDL
ofmorethan160withoutriskfactors,or>130withriskfactors,>100withknownCADshouldbetreatedwith
HMGCoAinhibitor.
OrderReview:
Fastinglipidprofile
Glucosetolerancetest
Counselthepatient/Patienteducation

Weightreduction
Lowfat,lowcaloricdiet
Regularexercise
OCPs
Papsmear
Followupin1weekwiththeresults
PrimaryDiagnosis:
PCOD

Location:
Emergencydepartment
CC:
Confusion
Vitalsigns:
T:37.7C(99.9F)BP:160/88mmHgP:112/minRR:24/min
HPI:
A42yearoldmanwitha10yearhistoryofalcoholabuseisbroughttotheEDinastateofconfusion.The
patientisunabletoprovideanysignificanthistory,buthisfamilyreportsthathehashadfever,nausea,and
vomitingforaday.Theyalsosaythatheisshaky,hearingvoices,andhasn'tslept.Hewasseenon2previous
occasionsintheEDforalcoholintoxication.Furtherquestioningrevealsthathedrankexcessively3nightsago,
precedingthesesymptoms.Hehasnoheadacheorneckstiffness.Thereisnohistoryofprescriptionor
recreationaldruguse,fall,ortrauma.Hehasnoknownallergies.
SH:
Thepatientisunemployedandliveswithhisparents.Hehasbeendrinking56beersdailyandvodka
"wheneverIcangetit"forabout10years.Heusedcocaineforsometime5yearsagobutquit.Hesmokes
cigarettesoccasionally.
Approachtothiscase:
Thispatientwithchronicalcoholismmostlikelyhasdeliriumtremens(DT).DTisaseriousformofalcohol
withdrawalthatpresentswithalteredsensoriumandautonomicinstability.Ittypicallyoccurs23daysafterthe
lastdrink,andsymptomsmaypersistupto7days.Examinationincludesidentifyingcomplicationsofchronic
alcoholusesuchasmalnutrition,liverdisease,andpancreatitis,aswellasrulingoutothercausesofaltered
mentalstatus(eg,infection,trauma).
Performrelevantexamination,drawsamplesforlabanalysis,andimmediatelystarttreatmentintheED.
Orderthephysicalexamination:
General
HEENT/neck
Heartandlungs
Abdomen
Extremities
Neuropsychiatric
Resultsoftheexamination:
General:42yearoldmanwhoappearsunkemptandinmilddistress.
HEENT/neck:Pupilsareequalandreactivetolight.Noscleralicterusorpapilledema.
Mucousmembranesaredry.Noneckstiffnessispresent.
Heartandlungs:Heartandlungsoundsarenormal.
Abdomen:Bowelsoundsarenormalmildhepatomegalyisalsopresent.
Extremities:Noperipheraledema.
Neuropsychiatric:Patientisconfused,disoriented,andcombative.Heisunabletofollowcommands
consistently.
Emergencycare:
Pulseoximeter,statandcontinuous
Supplementaloxygen,inhalation,continuous
Cardiorespiratorymonitoring,continuous
IVaccess

IVnormalsalinebolus,thencontinuous
IVthiamine100mgdaily
IVfolicacid1mgdaily
Bloodglucose,stat(chronicuseofalcoholmayresultinhypoglycemiaduetodecreasedglycogenstoresand
malnourishment)
NPOexceptmeds(untilpatientismorealert)
ECG,12lead,stat
Lorazepam24mgIV,repeatevery1520minutes,asneededforagitation
Seizureandaspirationprecautions
Testscanbeorderedonceemergencymeasuresaretaken.
Orders:
CBC,stat(Chronicalcoholismcancausethrombocytopeniaandmacrocytosisalsotolookforevidenceof
associatedinfectionthatwillcauseelevationinWBCcounts)
BMP,stat
LFTs,stat(foralcoholicliverdisease)
PTT,PT/INR,stat(associatedliverdiseaseanditscoagulopathy)
Serummagnesiumandphosphorus,stat(maybeassociatedwithalcoholism)
Drawbloodforculture,stat(infectionmaybeassociatedwithDT)
ABGs,stat(toexcludealcoholicketoacidosis)
Urinetoxicologyscreen,stat
Bloodalcohollevels,stat
Chestxray,PAview(toruleoutfrequentlyassociatedchestinfectionandpossibilityofaspirationina
patientwithalteredmentalstatus)
CTscanofhead,stat(toruleoutassociatedheadinjury)
Lumbarpuncture,stat(toexcludemeningitis).Notneededinpatientswithoutsignsofinfection(eg,
fever,meningealsigns,leukocytosis)
Results:
Labresultsaresignificantformacrocytosis(MCV110fL)andmilddegreeofhyponatremia(sodium133
mEq/L).Magnesiumandphosphoruslevelsarelow.Bloodglucoseis50mg/dL.LFTsaremildlyelevated
withmildhypoalbuminemia.CTscanoftheheadisWNL.CXRshowsnoabnormalities.Bloodalcohollevels
areundetectable.Urinetoxicologyisnegative.MildelevationofPTisnoted.ABGsshowedelevatedpHand
lowPCO2.
Revieworders:
IV50%glucose(afteradministrationofthiamine)
Admittogeneralmedicalwardwithcontinuoustelemetrymonitoring
ContinueIVfluids(adddextrosewhilepatientisnotawaketotoleratefeeds)withdailythiamine,
multivitamin,andfolicacidsupplementation
Repletemagnesiumandphosphorus
Monitorandrepeatelectrolytesfrequently(Q46hoursatfirst,lessfrequentlywhenpatientismore
stable)
Frequentneurologicalchecks(Q24hours)
Continuelorazepam
Haloperidol,IVprn(onlyifpatientisagitated)
Addacetaminophenprnforfeverandondansetronprnfornausea
Oncethepatientisstableandcaneat,discontinueIVlorazepamandstartchlordiazepoxide,PO,
continuous
Orderreviewafterpatientrecovery:
Rehabilitation

AlcoholicsAnonymous
Counselaboutsafesex,limitingalcoholintake,smokingcessation,seatbeltuse,andsafetyplan
Discussion:
AlcoholisaCNSdepressant,andsuddenwithdrawalinchronicuserscauseshyperactivityofpartsofthe
CNS,especiallythesympatheticnervoussystem.Minorwithdrawalsymptomsincludetremors,diaphoresis,
insomnia,anxiety,headache,gastrointestinalupset,andpalpitations.Theyusuallyoccurwithin6hoursof
withdrawalandresolvewithin12days.
Generalizedtonicclonicseizurescanoccurwithin1248hoursofwithdrawal.Thepresenceofprolonged
seizuresorstatusepilepticusisrareandshouldpromptaninvestigationforothercauses,suchasCNSmassor
infection.
Alcoholichallucinosisischaracterizedbyhallucinationsoccurringwithin1248hoursofalcoholwithdrawal.
Thesehallucinationsaremostlyvisualbutcanalsobeauditoryortactile.Patientstypicallyhaveintact
orientationandnormalvitalsigns.
Deliriumtremensisdefinedbyhallucinations,disorientation,autonomicinstability(tachycardia,hypertension,
hyperthermia),andagitationthatbegin4896hoursafterthelastdrinkandcanlastupto7days.Patients
typicallyhyperventilateandhaveresultantrespiratoryalkalosis.
Severealcoholwithdrawalisassociatedwithfluidandelectrolytedisturbancessuchashypovolemia,
hypokalemia,hypomagnesemia(maypredisposetodysrhythmiasandseizures),andhypophosphatemia(may
contributetocardiacfailureandrhabdomyolysis).
Treatmentofalcoholwithdrawalsyndrome:
Alcoholwithdrawalisadiagnosisofexclusion.Investigationforothercausesofalteredsensorium,suchas
infection,trauma,metabolicderangements,drugoverdose,andhepaticencephalopathy,mustbeperformedin
conjunctionwithtreatmentofacutewithdrawal.
Patientsaretypicallyplacedinaquietenvironment,withfrequentmonitoring.Benzodiazepinesareusedto
controlpsychomotoragitationandpreventfurtherwithdrawalsymptoms.Mechanicalrestraintsmaybe
necessarytopreventharmtothepatientandcaretakers.Oncethepatientiscalm,theseshouldberemovedas
prolongedusecancausephysicalinjuryandrhabdomyolysis.Withdrawalseizuresaretreatedwith
benzodiazepinesandphenobarbitalinrefractorycases(usedonlyinICUsituations).
VolumedeficitsarecalculatedandtypicallyreplacedwithisotonicIVfluidsuntilthepatientiseuvolemic.
Multivitamins,folicacid,andthiaminearegivendailytoallpatients.Thiaminemustbegivenbeforeglucoseto
decreasetheriskofprecipitatingWernickeencephalopathyandKorsakoffsyndrome.Electrolytedeficiencies
shouldbecorrected.Patientswithmarkedacidbasedisturbances,respiratoryinsufficiency,persistent
hyperthermia,cardiacorliverdisease,orneedforfrequentorhighdosesofsedativesmustbemovedtotheICU.
DTistreatedinitiallywithIVlorazepam.Oncevitalsignsarestableandthepatientcantakeoralmedications,
standardchlordiazepoxide(Librium)protocolcanbefollowed.
Whentogiveantipsychotics:
Phenothiazinesshouldnotbeusedastheylowertheseizurethreshold.However,haloperidolisassociatedwith
lowriskandmaybeusedincombinationwithbenzodiazepinesforagitatedpatients.
Afteracutetreatment,allpatientsshouldbescreenedforalcoholdependenceandconsideredatriskforrecurrent
episodesofwithdrawal.Inhospitalevaluationisrecommended,andlongtermfollowupiscrucial.
Primarydiagnosis:
Alcoholwithdrawal

Location:Anurseryinacommunityhospital.
CC:Infantwithjaundice.
HPI:
Youarecalledevaluatea12houroldwhitemaleinfantwhoappearstobejaundiced.Thepatientwasbornat
39weeksgestationbyvaginaldeliverywithvacuumassistance.Laborwasslightlyprolongedat18hours.The
patient'sApgarscoreswere8and10at1and5minutes.Pointswereremovedforacrocyanosisandirritability.
Thepatienttooktothebreastwithinaboutthreehoursafterdelivery,hadgoodsuckreflexandtransitionedwell
toroomtemperatureenvironmentafterbeinginthewarmerforaboutfourhours.Initialevaluationofthepatient
bytheoncallresidentwasunremarkableotherthanacephalohematomaovertheposteriorparietalandoccipital
areasfromthevacuumassisteddelivery.
Maternalhistory:Motherisa28yearoldGroupBStreptococci(GBS)positiveG2,P2,nowL2whitefemale
ofMediterraneandescentwhoreceivedregularprenatalcareandhadintrapartumantibioticprophylaxisforher
GBSpositivestatus.Shehasnoprevioushistoryofsexuallytransmitteddiseases,hadnoinfectionsduringher
pregnancyandhadlaboratoryworkrevealingthatshewasRhpositive,bloodgroupO.Mother'sotherprenatal
laboratoriesshowedthatshehadpositiverubellatitersandpositiveIgGtotoxoplasmosis.Thereisnohistoryof
smoking,IVdruguse.Motherisnotonanymedicationsotherthanprenatalvitamins.FH:Thepatienthasan
18montholdsiblingwhohadjaundiceasaninfant.Mothercannotrecallhowhighherotherchild'sbilirubin
levelreachedhowever,shedoesnotethatherfirstchildrequiredphototherapyfortwodayspriortodischarge.
Howtoapproachthiscase:
Jaundiceinaninfantlessthan24houroldshouldalwaysbepresumedtobepathological.Therefore,you
shouldlookcarefullyfortheetiology,includingsepsis,hemolysis,polycythemia,andhemorrhages.Youneedto
lookattheinfanttodecidewhichetiologyishighestonyourdifferentialand,inparticular,todecidewhetherthe
childneedsimmediateantibiotics.Youneedtoattendtodetailsofthemother'smedicalhistory(GroupBstrep
status,TORCHexposure)andthedelivery(prolonged,traumatic,vaginalvs.csection,fetaldistress
intrapartum).
Therefore,performphysicalexam,including
Reviewofvitalsigns(checkfortemperatureinstabilityvoiding)
Generalappearance(lethargic?Arousable?)
HEENT(cephalohematomas,caputsuccedaneum,cornealopacity?)
Heartexam
Lungexam
Abdomen(hepatosplenomegaly?)
Genitourinary(patentrectum)
Neurologicalexam(tone,irritability,reflexes)
Skin(ecchymoses)
Checkinputs/outputs
Results:
Vitalsignsarestable.Welldevelopedwhitemaleinfantinnoacutedistress.
HEENT:Thereismildscleralicterusandjaundiceoftheface.Thereisa4cmcephalohematomaoverthe
posteriorparietaloccipitalareasoftheinfant'sheadthatissoftandslightlyecchymotic.Mucousmembranesare
moist.Pupilshavebilateralredreflex.Oropharynxisclear.Thepatienthasagoodsuckreflex.Neckissupple
withoutanysignsofmeningismus.Anteriorfontanelisopen,flatandsoft.
CardiovascularandLungs:normal.
Abdomen:soft,nondistended,nontender,nohepatosplenomegalycanbeappreciated,noabdominal
masses.Umbilicus:normal.
Extremities:normal.Thereisnoacrocyanosisappreciated.Capillaryrefillislessthantwoseconds.
Neurologic:Goodsuckreflex.Mororeflexsymmetric.Goodtone.Appropriatelevelofirritabilityandableto
beconsoled.

GU:normal.Theanusispatentandappropriatelypositioned.
Reviewofnurse'snotesrevealsthatthepatienthaspassedseveralmeconiumstoolsandvoidedonewetdiaper.
Sofar,thisinfant'sphysicalexamisreassuring.Yourinitialphysicalexamdidnotrevealanysignsofsepsis:
thepatienthadnotemperatureinstability,hadbeeneatingwell,hadpassedastoolandurineofnormalcolor,
anddidnotappeartobelethargic.Therefore,youcandosomeworkupbeforeinitiatingantibiotics.
Order:
Bloodtypingofinfantandmother(mother'sisusuallydonealready&onchart)
DirectCoomb'stest,stat
CRPstatandq12hr
CBCwithdifferential,stat
Totalandindirectbilirubin
Inputs/outputs
Vitalsignsq4hrs
Discussion:
Thisisafullterminfantwhopresentswithjaundiceinthefirstdayoflife.Becauseoftheearlyonsetof
jaundice,hemolysisandsepsismustberuledout.Otheritemsinthedifferentialincludepolycythemiaand
hemorrhages.Workupofthispatientshouldbetheinitialphysicalexamtoobserveforsignsofsepsisincluding
inaninfantsignsoflethargy,vomiting,poorfeeding,feverortemperaturelabilitywithalowtemperature,lack
ofstooling,orabnormalcoloredurine.Historytakingshouldfocusonanypossiblematernalinfectionswhich
mightbetransmittedtotheinfant,particularlyTORCHinfectionsincludingCMV,toxoplasmosisandrubella
infection,andmaternalGBS(GroupBstreptococcalinfection)status.Elicitinganyfamilyhistoryof
hyperbilirubinemiaisalsohelpfulbecausethereissomefamilialtendencyforneonataljaundice.Laboratory
workupshouldbeginbybloodtypinganddirectCoombs'testingontheinfant.Allinfantswhoarebornto
mother'swithTypeObloodgroupshouldroutinelyhavedirectCoombs'testingtocheckformaternalfetal
incompatibilityandthesechildrenshouldbefollowedcloselyforevidenceofjaundicefromhemolysis.
LaboratoryworkshouldalsoincludeCreactiveproteinforearlyassessmentofinfection,CBC,andinitiallya
directandtotalbilirubin.
Results:
MotherisgroupObloodgroup.ThepatientisGroupB.ThedirectCoombs'testwasweaklypositive.Initial
CBCwithdifferentialisnormalforage.CRPwasnormal.Totalbilirubinwas5.6,directbilirubin0.3,indirect
5.2.
AtthispointthepatientappearstohavemildhemolysissecondarytoABOincompatibilitywithmaternalblood
withaweaklypositiveCoombs'test.Thepatientneedstohavehishemoglobinandhematocritfollowedaswell
ashistotalbilirubintoobserveforevidenceofacontinuedclimbinthebilirubinlevelorasignificantdropin
hishemoglobinandhematocrit.
Therefore,Order:
Hemoglobinandhematocritq8hr
Totalbilirubinq8hr
Continuepofeeding,breastmilk
Maysupplementwithformula
Vitalsignsq4towatchforchangeinclinicalstatus
Results:
2ndCRPisnormal.
Hemoglobinandhematocritremainstable.
Totalbilirubinincreasesto8.4andthento13.4mg/dl
Order:
TransfertoNICU(NeonatalICU)

Phototherapy
Erythromycinointmentforeyeswhilereceivingphototherapy
StartIVfluidsatratetoensurepatient'stotalfluidintake(oralandIV)is1Smaintenancerate.D51/4NS
Results:
Totalbilirubinlevelsfallwithphototherapy
Orderreview:
Phototherapydiscontinued
D/CIVfluids
Followuptotalbilirubinqdailyuntilstable.
Discussion:
At12hoursofage,thepatient'stotalbilirubinis5.6whichroughlycorrespondstotheruleofthumbfor
assessingclinicaljaundicewhichcorrespondstojaundiceofthefacewithabilirubinofapproximately5,
jaundiceofthemidabdomenindicatingalevelofapproximately15mg/dlandjaundiceextendingtothefeet
indicatingalevelofroughly20mg/dl.Atthispoint,thepatientdoesnotmeetcriteriaforphototherapy.One
shouldconsiderphototherapyatlevelsofgreaterthan12mg/dlforaninfant2548hoursold.Definitelydo
phototherapyifthelevelisgreaterthanorequalto15mg/dland,ifthepatienthasalevelgreaterthan25mg/dl,
thenexchangetransfusionandintensivephototherapyisprobablywarranted.Q8Hmeasurementsof
hemoglobin,hematocritandtotalbilirubinwouldbeappropriate.Inaddition,asecondCRPlevelwouldbe
appropriatetomonitorforsignsofinfection.TiterscanbesentforCMV,toxoplasmosisandrubellaaswell,
althoughusuallythisdataisnotavailabletotheclinicianinthefirst2448hoursofevaluation.Ifthepatient's
clinicalstatusshouldchangeandheshouldbecomelethargicorhavedifficultyfeedingandyoususpectsepsis,
thenempiricantibiotictherapyshouldbebegunbuturineculturesandaspinalwouldbeappropriate.
Byhistory,thispatientalsohasanincreasedriskforhyperbilirubinemiasecondarytohispositivefamilyhistory
ofasiblingwhohadneonataljaundicerequiringphototherapy.Aninfantwithasiblingwhodevelopeda
bilirubinlevelofgreaterthan12hasapproximatelyathreetimesgreaterriskofdevelopingjaundicethanan
infantwithanegativefamilyhistory.
Inaddition,thisinfantobviouslyhasanABOincompatibility.Onewouldsuspectthatthebilirubinlevelwould
continuetoclimboverthenext24hoursandthispatientmightindeedmeetcriteriaforphototherapy.Inthe
meantime,thepatientshouldbecontinuedtobemonitoredandthemothershouldbecounseledtocontinue
frequentbreastfeedingtokeepthebabywellhydratedandtoobserveforanysignsofinfection.
Theinfant'sotherriskfactorforelevatedjaundiceincludeshiscephalohematomawhichcanbeanincreased
sourceofbilirubinproductionasthehemorrhagereabsorbs.Thepatientdidnotmeetcriteriaforpolycythemia
butthisisanothercauseofearlyjaundice.Forinstancewith"physiologicjaundice"thebilirubinleveldoesnot
risemorethan5mg/dlper24hoursanditusuallypeaksondaythreewithalevelnogreaterthan13mg/dl.
Thesechildrenusuallydonotpresentwithjaundicewithinthefirst24hoursoflife.Likewise,breastfeeding
jaundicewhichdoespresentwithinthefivedaysoflifeusuallydoesnotreachclinicallevelswithinthefirst24
hours.Breastmilkjaundiceisusuallyseentowardstheendofthefirstweekoflifeinaninfantwhoisotherwise
thriving.Jaundicewhicharisesgreaterthanoneweekfrombirthmayindicatebothbreastmilkjaundicebutalso
otherpathologicalmechanismsincludingliverdisordersuchasbiliaryatresiaormetabolicdisordersuchhas
hypothyroidism,galactosemiaorhereditaryhemolyticdisordersuchhasspherocytosisorG6PDdeficiency
whichincidentallyisanXlinkeddiseasewithvariablephenotype.Jaundicethatpersistsbeyondthethirdweek
reallyshouldpromptonetoinvestigatebiliarycausesaswellasthemetabolicandhereditaryhemolytic
diseases.Mostformsofjaundiceshouldresolvebytwotothreeweeksofageandthevastmajorityofinfants
whodohavejaundicehaveaphysiologicorbreastfeedingassociatedcause.
PrimaryDiagnosis
Jaundice

Location:Outpatientclinic
Vitals:Temperatureof36.3CH.R:95/minR.R:22/minBloodpressureis85/50lyingdownand77/46mmHg
standing.
C.C:Swelling
HPI:
Thepatientisathreeyearoldwhitemalewhopresentswithhismotherforevaluationoffacialandscrotal
swellingoftendaysduration.Motherreportsthatthechildhadbeenwelluntilonedaypriortoadmissionwhen
shenoticedtheonsetofswellinginhisface.Shealsonotedthathehadscrotalswellingbecauseheisalmost
pottytrained.Shenotessomedecreaseinhisurineoutputaswell,althoughnochangeincoloroftheurine,
otherthanbecomingsomewhatmoreconcentrated.Hehashadnoprecedingdiarrhealillness,sorethroat,
abdominalcomplaints,fevers,andrashes.Motherreportsthatthechildhasnotcomplainedofanypain
syndrome.Hedoesseemtobealittlebitmoretiredthanusual.Birthhistoryisunremarkable.Allofhis
immunizationsareuptodate.SH:Heliveswithhisparentstwooldersiblingswhoarehealthy,andhavehad
nohospitalizations.Therearenopetsinthehome.Therearenosmokersinthehome.Riskfortuberculosisis
low.Developmenthasbeennormal.
Howtoapproachthiscase?
Determinethenatureandetiologyofthe"swelling",includingwhetherit'sedemaorsomethinglikehives.
Examinethepatienttodecidewhetherheneedsinpatientoroutpatientmanagement.
Physicalexam:
Generalappearance
HEENT/Neck
Heart
Lung
Abdomen
Genitourinary
Extremities
Skin
CNS
Results:
General:welldeveloped,wellnourishedwhitemaleinnoacutedistress.HEENT:remarkableforperiorbital
edema.Mucousmembranesareslightlydry.Neckissupplewithoutlymphadenopathyorthyromegaly.Pupils
normal.Cardiovascular:Regularrateandrhythmwithoutmurmurs,rubsorgallops.Lungs:veryfaintralesat
thebasesandotherwiseclear.Abdomenissoft,nontender,nondistended.Therearenormoactivebowelsounds.
Thereis1+sacraledema.+fluidwave.GU:Thereisscrotaledemapresent.Thereisnotendernessto
palpationandthecremastericreflexisintactbilaterally.Extremities:Pulsesare2/4intheradial,femoral,and
dorsalispedisareas.Handsandfeetshow2+pittingedemaandareotherwiseunremarkable.Neurologicis
nonfocalandappropriate.
Discussion:
Thisisathreeyearoldpatientwhoismanifestingsignsofgeneralizededema,mostprominentinhisfacehands
andscrotum.Theleadingdiagnosisinthisageframeforsuchmarkededemaisnephroticsyndromesecondary
tominimalchangedisease.
Order:
Urinalysis,stat
Basicmetabolicpanel,stat
CBCwithdifferential,stat
LFTs

Lipidpanel
PT/INR,PTT
Complement3and4levels
Results:
Urinalysisshows4+protein,noblood,noRBCs,specificgravityis1.030,
CBCshowsawhitecountof7,hemoglobin12.6,hematocrit36,andplatelets240.
Completemetabolicpanel(LFTs+BMP)revealsanalbuminof1.5,normalliverfunctiontests,sodium130,
potassium4.0,chloride96,bicarbonate20,BUN10,creatinine0.7,glucose78,calcium9.4,cholesterollevelis
320mg/dl.Serumalbuminis1.5gm/dl.Serumproteinis3.7.PT,PTTarenormal.Complementlevelswithin
normal.Patienthasorthostatichypotensionandmilddehydration.
Order:
Admittofloor
Inputs/outputs
Vitalsignsq4Continuouscardiorespiratorymonitoring
Nephrologyconsult
Albumin25%solutionIV,1gr/kgbodyweight,infusedover8hours
Lasix(Furosemide),1mg/kg,administeredhalfwaythroughthealbumininfusion
CompletemetabolicpanelqAM
Nosaltadded,highproteindiet.
Results:
Patientrespondswithgooddiuresistoalbuminandlasixtherapyover24hours.
Vitalsignsremainstable.Orthostasisresolves.
Electrolytesandrenalfunctionremainstable.
Order:
Prednisone2mg/kgperday,maygiveindivideddose,po
Vitalsignsq12hours
Repeatalbuminandlasixtherapy.
Results:
Patienttoleratesprednisone.
Remainsclinicallystable.
Orderreview:
Dischargetohome.
Prednisonefor46weeks.
Followupin35days.
Discussion:
Themostlikelycauseofthispatient'sclinicalsyndromeisanephroticsyndrome.Thepatientisathreeyearold
withgeneralizededemaandscreeninglaboratorytestsindicatelowproteinaswellasproteinuria.Hetherefore
meetsthediagnosticcriteriafornephroticsyndromewhichare:
1.Generalizededema.
2.Hypoproteinemia,usuallyless2gm/dl,withadisproportionatelylowalbuminlevelinrelationtothe
globulinlevel.
3.Urineproteintourinecreatinineratioinexcessof2onfirstmorningvoidora24hoururineproteinthat
exceeds50mg/kgofbodyweight.
4.Hypercholesterolemiagreaterthan200mg/dl.
Inthevastmajorityofchildreninthisagerangethenephroticsyndromeisduetominimalchangedisease.This
isthecauseinapproximately76percentofchildrenintheoneto12yearagerangewhopresentwithnephrotic

syndromeinchildhood.Otheretiologiesarefocalsegmentalglomerulonephropathy,membranoproliferative
glomerulonephropathy,membranousnephropathyandthenothercauses.Formostchildren,clinicaldiagnosisis
sufficientandrenalbiopsyisnotwarrantedunlessthechildisnotresponsivetosteroidsasnotedbelow.
CLINICALFINDINGSinnephroticsyndromeincludefacialedemafrequentlyperiorbital,pretibialedemaas
wellasswellingofthescrotumorlabiawhichmaybeprominent.Thesechildrenalsohavereducedperfusionof
theirsplanchniccapillarybedandmayhaveabdominalpain.Asaconsequenceoftheirlowintravascular
oncoticpressure,theymayhavehypotensionaswellaspleuraleffusionswithtachypneaandchestpain.As
notedinthediagnosticcriteria,theydohavehypercholesterolemiawithincreasesintheirVLDLandLDL
becauseofchangesinhepaticcatabolismandenhancedsynthesis,respectively.However,thedisturbancesin
lipidmetabolismusuallydonotcauseanyotherclinicalfindings.Bycontrast,changesintheproteinlevelsof
theircoagulationcascadedoesputthematincreasedriskforthrombosis.Theirtendencytoformthrombusis
duetoacombinationoffactorsincludinghyperaggreatableplatelets,increasedfibrinogenconcentrations,lossof
antithrombinIII,increasedbloodviscosityanddecreasedbloodflow.Venousthrombosiscanoccurinthedeep
veinsoftheextremitiesandthecerebrocortialsystemandrenalveinsandthepulmonaryvenoussystemanditis
asourceofincreasedmorbidityforthesechildren.Asaconsequence,oneshouldobtaininitialcoagulation
studiesuponadmission.
Laboratoryanalysisalsoshowsreducedimmunoglobulins,particularlyIgG.ThislowlevelofIgGin
combinationwiththesteroidswhicharetheprimarytreatmentfornephroticsyndromeputsthechildrenat
increasedriskfordevelopmentofinfections.
Peritonitisisoneofthemoreseriouscomplicationsofnephroticsyndrome.Thecausativeorganismsinchildren
arestreptococcuspneumoniaeandEscherichiacoli.Oneshouldconsiderperitonitisinanychildwhopresents
withsignificantabdominalcomplaintsandparacentesisshouldbeperformedtoidentifytheorganismand
confirmthediagnosis.Forchildreninwhomthrombosisisaseriousconsideration,onecanstartheparintherapy
atadoseof50units/kgintravenouslyand100units/kgeveryfourhoursIVformaintenancetherapy.Human
nephroticsyndromeissupportiveandaimedatincreasingtheintravascularoncoticpressure,decreasingthird
spacedfluidmaintainingfluidbalance,monitoringnutritionandtreatinganyconcomitantinfection.Usuallythe
typicalapproachistogivethepatientintravenousalbumin1gm/kgofa25%solutionforexampleandinfuseit
continuouslyover8to12hoursunderclosesupervisionforthedevelopmentofheartfailure.Loopdiureticslike
Lasixcanthenbeadministeredhalfwaythroughthealbumininfusionorafteritatadoseof12mg/kgIVand
thiseffectivelyreducesthirdspacedfluidsandpullsinterstitialfluidintothevascularspace.Prednisoneis
begunonpatientsatadoseof60mg/M2or2mg/kgandthedailydoseismaintainedforfourtosixweeks.Itis
advisabletoobtainatuberculosisscreentestpriortoinitiatingthesteroidtherapy.Mostchildrenwillbeginto
showadecreaseintheirurineproteinexcretionafteraboutseventotendaysfollowingtheinitiationofsteroids.
Ifthechildisstablewithjustmildtomoderateedema,nopulmonaryedemaandhasgooddiureticresponse,
thentheydonotneedtostayinthehospitaluntiltheirproteinexcretionisreducedandinsteadmostofthemcan
bedischargedwithintwotothreedays.Fluidbalanceinthehospitalsettingisimportantandshouldbe
monitoredcloselyduringtheinitialdiuresis.Patientshouldbegivenahighquality,highproteindiettoimprove
theirgrowthandbecausetheyhaveanincreasedproteinneedtoregeneratetheiralbumin.Theyshouldhavea
nosaltaddeddiettodiscouragefurtheredemaformation.About8590%ofchildrentreatedinfashionwillhave
asatisfactoryresponseandobtainremission.Themortalityinminimalchangenephroticsyndromeis
approximately2%withthemajorityofdeathsduetoperitonitisorthrombusformationandthesecomplications
canoccurevenunderidealtreatmentcircumstances.Theother98%ofchildrenwhodevelopthissyndromeare
likelytohaveagoodresponsetosteroidsandreturntoanormalstateofhealth.Abouttwothirdsofthem
experienceatleastasinglerelapseandanotherthirdgoontohaveaseriesofrelapsesoverthecourseofmany
years.
Forchildrenwhoremainsymptomfreeforovertwoyearswithoutanymedications,theirprognosisisthebest
andtheyareconsideredrecovered.
Primarydiagnosis:
NEPHROTICSYNDROME

Location:EmergencyRoom.
Vitalsigns:Temperature38.6Cheartrate156/minrespirations62perminutebloodpressureis75/43mHg
Weight3.0kg.
C.C:Poorfeeding,anddecreasedresponsiveness.
HPI:
Thepatientisasixdayoldfemale,broughttotheEmergencyRoombyhermotherbecauseofdifficulty
arousingthebabyforfeedings,decreasedintakeandgenerallyseeminglessresponsiveaccordingtothemother.
Momdidnottakepatient'stemperature.Shehasnoticedthatthereareadecreasednumberofwetdiapersfrom
usualofaroundeightperdaytoonlyfoursincethesametimethedaypriortobeingseen.Thebabyshows
someincreasedsomnolenceandwhensheisawakesheseemstobemoreirritable.Mothernotesthatduring
breastfeedingthepatient'ssuckingseemssomewhatdiminished.Thepatienthashadnoepisodesofemesisor
diarrhea,otherthanherusuallooseyellowstoolsfrombreastfeeding.Motherhaswitnessednocyanosis,
episodesofapnea,grosshematuria,orseizurelikeactivity.
BIRTHHISTORY:Thepatientwasa3.210kgfemalebornat38weeksgestationtoaG2,P2,nowL2,GBS
positive,Rhpositive25yearoldnonsmokingmotherwhowasingoodhealththroughoutthepregnancyand
receivedregularprenatalcareincludingintrapartumantibiotics.Birthwasviaspontaneousvaginaldelivery
withlaborlastingapproximately14hoursanddeliveryoccurringwithoutcomplications.Thepatient'sApgar
scoreswere9,onepointtakenoffforcolor,and10atoneandfiveminutes,respectively.Thepatientwas
dischargedhomeafter48hourswiththemother.Thedischargeweightwas3.002kg.SOCIAL:Thepatient
liveswithparentsandathreeyearoldsibling.Fathersmokesinthehouse.Thereisnoprevioushistoryforthe
motherofanyelicitorIVdruguse.Thebabyisbeingbreastfedondemandapproximatelyeverytwotothree
hoursforthefirstapproximatelyfourdaysoflife.Eachbreastfeedingsittinglastingabout20minutes.Mother
reportsthatthebabyisnowfeedingeverythreetofourhoursandonlystayingatthebreastforabouttenminutes
beforefallingofftosleep.ThebabydidreceiveahepatitisBimmunizationpriortodischargefromthehospital.
Howtoapproachthiscase?
Inaninfantlessthanonemontholdpresentingwithdecreasedresponsiveness,thesuspicionforsepsisshouldbe
high,butthedifferentialisbroadandincludestrauma(e.g.,shakenbaby),congenitalabnormalities,andparental
misinterpretation.
Physicalexam:
PulseOximetry,stat
Generalappearance
HEENT/neck
Heart
Lung
Abdomen
Neuro
Musculoskeletal
Skin
Results:
General:Welldeveloped,wellnourishedwhiteinfantsleepinginhermother'sarms.Pulseoximetry91%on
roomair.HEENT:Normocephalic,atraumatic.Anteriorfontanelisopen,flatandsoft.Mucousmembranesare
slightlydry.Pupilsareequal,roundandreactivetolight.Redreflexispresentbilaterally.Naresshowmild
flaringandarepatent.Tympanicmembranesarewithinnormallimits.Oropharynxisclear.Neckflexionand
extensionarewithinnormalanddonotelicitirritability.Cardiovascular:Regularrateandrhythmwithout
murmurs,rubsorgallops,S1andS2auscultated.Lungsareclearbilaterally.Therearemildsubcostaland
suprasternalretractions.Abdomenissoft,nontender,andnondistendedwithnormoactivebowelsounds.No
hepatosplenomegalyappreciated.Umbilicusiswithouterythemaordischargearoundtheumbilicalstump.

Extremities:Pulsesare2/4intheradial,femoral,dorsalispedisareas.Capillaryrefillisbetween2and3
secondswithnoevidenceofcyanosis,edemaorclubbingoftheextremities.GUshowsnormalfemalegenitalia.
Atthispointintheevaluation,onehasasixdayoldinfantwithafeverof38.6andsomeevidenceofrespiratory
compromisewithdiminishedperipheralO2saturationandtachypneawithretractions.
Orders:
Supplementaloxygentokeepsaturations>94%
PlaceIV
CBCwithdifferential,stat
Basicmetabolicpanelorchemistrypanel,stat
Bloodcultures,urinecultures,and,giventhispatient'sage,CSFcultures.
CSFforprotein,glucose,cellcount,andGramstain.
ChestXrayshouldbeobtainedtoevaluateforpneumonia.
CRPisalsousefultoevaluateacuteinfectiousprocessesbutisnonspecific.
Results:
ChestXrayDiffusereticularnodularpatternbilaterallywithslighthyperinflation,andaverysmallright
pleuraleffusion.
CBC:Whitecount16,hemoglobin13.7,hematocrit38,platelets221,thedifferentialshows68%neutrophils
with5bandneutrophils,10%lymphocytes.
Chemistrypanelshowssodiumof135,potassium3.9,chloride99,CO220,Bun7,creatinine0.3,calcium10.1,
glucose71,
CRPelevatedat2.7.
Bloodcultureswereobtainedandpending.
UrinalysisSpecificgravityof1.028,14whitebloodcells,0redcells,negativenitrate,negativeesterase.
Urineculturesstillpending.
Lumbarpuncturewasperformed.CSFcellcount,glucoseandproteinwerewithinnormallimits.Gramstainno
organisms,noneutrophils.Cultureisstillpending.
Order:
Admittofloor
Continuouscardiorespiratorymonitoring.
Vitalssignsq4
Dietnooralifrespiratoryrateisgreaterthan60
IVFD5jNSatmaintenancerate
Ampicillin100mg/kg/daydividedq8hr
Cefotaxime150mg/kg/daydividedq8hr
Inputs/outputs
CBCwithdiff,BMPqdaily
Results:
Bloodculturegrowsoutgrampositivecocciinchains.(ThelikelihoodofGroupBstrepinfectionisveryhigh.)
Order:
Continuemonitoring.
Examinethepatientforevery2to4hoursuntilyouseesomeimprovement,thenevery8to12hours
Changediettopowhenrespiratoryrateis<60andnosignificantrespiratorydistress.
Weanoxygenforsaturations>94%
Results:
Patient'soxygenationimprovesandshe'sweanedtoroomair.
Respiratoryratenormalizesworkofbreathingnormalizes.
Patienttoleratespoandmaintainshydrationstatus.
BloodculturegroupBstrep.Sensitivetoamoxicillin.

Order:
Dischargehome
Changeantibiotictoamoxicillin5080mg/kg/daydividedq.
Discussion:
Thispatientpresentswithsignsandsymptomssuggestiveofarespiratoryprocesshowever,givenherageand
othernonspecificfindings,thepossibilityofurinarytractorCSFinfectionorgeneralizedsepsisisstillinthe
differentialandthereforeonewouldwanttobeginempiricantibioticstocoverforthemostlikelyorganismsto
infectthisinfant.Forachildofthisagewithrespiratorysymptoms,themostlikelyetiologiesareE.coliand
GroupBstrepinfections.Othercausesofrespiratorycompromisewithinfectiousetiologiesinclude
Haemophilusinfluenza,streptococcuspneumoniae,GroupBstrep,Listeria,andanaerobes.Formanyinfants,
theonlyindicationthattheirinfectionisrespiratoryinnatureistachypneawhichmayatfirstbemissed.
Oftentimestheseinfantsdonothavetypicalralesorsoundsofconsolidationonauscultation.Dependingon
howoldtheinfantis,hyalinemembranediseaseandtransienttachypneaofthenewbornaredifferential
considerationsfortheinfantinthefirst2448hoursoflifewhoexhibitsrespiratorysymptoms.
Bloodculturesobtainedonnewbornswithrespiratoryillnessesfrequentlywillgrowouttheoffending
organism.ThispatienthasariskfactorforGroupBstrepinfectionbecausehermotherwasGroupBstrep
positive.Despitethefactthatthemotherreceivedintrapartumantibiotics,itisstillpossibleforthisinfantto
haveaGroupBstrepinfection.Likewise,formaternalhistoriesinwhichtheGroupBstrepscreeningis
negative,itisstillpossiblethattheinfanthasGBSinfectionsincescreeningisnot100%sensitive.Empiric
antibioticcoverageshouldbedoneassoonaspossibleafterculturefluidsareobtained.Oneregimenis
ampicillin100mg/kg/ddividedevery12hoursforinfantswhoarelessthan1.2kgoreveryeighthoursfor
infantswhoaregreaterthan1.2kgandcefotaxime(Claforan)100mg/kg/ddividedevery12hoursor150
mg/kg/ddividedeveryeighthoursforinfantswhoaregreaterthan1.2kgandgreaterthansevendaysold.
Gentamicincanbeaddedaswellorusedasanalternativetreatmentwhenthereisnoevidenceformeningitis.
Infantsshouldbetreatedatleasttendaysandpossibly1421daysiftheyhaveagramnegativeinfection.
Aninfantofthisagewithrespiratorysymptomsshouldbeadmittedandmonitoredcloselyforworsening
respiratorycompromisewithdecreasedoxygenationandincreasedworkofbreathing.Inaninfantwhodoesnot
respondtoempiricantibiotictherapybutstillhasrespiratorysymptoms,thenCMV(cytomegalovirus)
pneumoniashouldbeconsidered.Ifthereisamaternalhistoryofherpessimplexvirusinfection,particularlya
primaryinfection,thenthatetiologyshouldalsostronglybeconsideredandacyclovirbegun.
Primarydiagnosis:
GroupBstreptococcalpneumonia

Location:EmergencyRoom.
Vitalsigns:B.P:108/75mmHgHR88/minRR:8/minTemp.36.5C.
CC:alteredmentalstatus,stumbling
HPI:
Thepatientisasevenyearoldboybroughtinbyhisparentsafterhecamehomefromaplaymate'shouseand
hisparentsfoundhimtobeconfusedandstumblinghethenbecamelessresponsivewithgarbledspeechand
somnolence.Hehadbeenpreviouslywellwithnorecentinfections,nosickcontacts.Hehadgonetoschool
thatdayandcomehomeappearingtobenormalatthattime.Hethenwenttoaschoolmate'shouseandreturned
homeseveralhourslaterwiththeabovesymptoms.SOCIALHISTORY:Heattendssecondgradeatasuburban
school.Heliveswithhisparentsandtwoothersiblings.Hisdevelopmenthasbeennormal.Hisimmunizations
areuptodate.ROS:noheadache,fevers,vomiting,diarrhea,photophobia,jointcomplaints,rashes,urinary
complaints,noseizureactivity.
Howtoapproachthiscase?
Thischildhassufferedanacutechangeinmentalstatus.InitialmanagementshouldfocusontheABCs.He
needsabrieffocusedphysicalexamtoguidethedifferential.
Order:
Pulseoximetry,stat
Supplementaloxygen
Continuouscardiorespiratorymonitoring
Fingerstickglucose
IVlock
Urinetoxicologyscreen
Narcan(naloxone),IV
Physicalexam:
Generalappearance
HEENT/neck
Heart
Lung
Abdomen
Musculoskeletal
Neuro
Results:
O2sat94%onroomair.
General:welldeveloped,wellnourishedsevenyearoldboyheappearsdiaphoreticandcool.Neuro:He
mumblesaresponsethatcannotbeunderstoodandwillnotopenhiseyestocommandhelocalizestopainful
stimuli.HEENT/Neck:Pupils3mmbilaterallyandresponsive.Necksupple,noadenopathy:mucous
membranestacky.Hehastheodorofalcoholonhisbreath.Heart:regularwithoutmurmurs.Lung:
respirationsareshallowandslow.Abdomen:normal.Musculoskeletal:Thereisnoevidenceoftrauma.
ThereisnochangeinlevelofresponsivenessafterNaloxone.
Fingerstickglucoseis48mg/dl.
Order:
D50,1ampule,IV
IVFNormalsalinebolus500cc,thenatmaintenance
Bloodalcohollevel(BAL)
Serumtoxicologypanel

Basicmetabolicpanel,stat
CBCwithdifferential,stat
Accuchecksq1houruntilstable
Results:
BALiselevated.
Urinetoxicologyscreenpositiveforethanol.
Serumtoxicologypanelpositiveforethanolonly.
BMPisnormal.
CBCwithdiffisnormal.
Accuchecksnormalize.
Order:
Admittoobservationunit/floor.
Continuouscardiorespiratorymonitoring.
IVFD5SNSwith20meq/LKCLatmaintenance
NPOuntilawake
BMPinAM
Repeatbloodalcohollevelin12hours.
Result:
Patientbecomesmoreresponsiveafter4hoursandisfullyawakeandbacktohisbaselinebythenextmorning.
RepeatBALiswithinnormallimits.
Order:
Dischargehome.
Patienteducationondruguse/toxicity.
Screenforabuseanddomesticviolencepriortodischarge.
Discussion:
Thisisaschoolagedchildwhopresentswithalteredmentalstatusandhypoglycemia.Hehadadepressed
mentalstatuswithshallowandslowrespiratoryrateandhypoglycemia.Thispresentationalongwithhelpful
informationobtainedduringthephysicalexam,suchasthesmellofalcohol,candirectthecliniciantolookfor
ethanolintoxicationasthecauseofthispatient'ssyndrome.Typicallychildrengethypoglycemiaanditisnot
necessarilyrelatedtothedoseorbloodleveloftheethanol.TheyhaveadepressedCNSsecondarytoethanolas
aCNSdepressant.EVALUATIONinvolvesrulingouttrauma,infectiousprocesses(particularlyCSF
infections),sepsis,orotherdrugintoxication,andthenbeginningsupportivecarewithdetectionand
maintenanceoftheairway.IVfluidsaregivenforfluidbalance,correctionofanyelectrolyteimbalancesand
correctionofhypoglycemia.Mostchildrenrespondwelltosupportivetherapyandtheirprognosisforafull
recoveryisexcellentintheabsenceofprolongedhypoglycemiaandrespiratoryarrest.
PrimaryDiagnosis
ChildIntoxication

Location:Emergencyroom
Vitalsigns:B.P:140/80mHgP.R:70/minR.R:18/minTemp:36.0C.
C.C:Headache
HPI:
A50yroldwhitefemalepresentstoE.Rforsuddenonsetofsevereheadachedevelopedoverthepasttwo
hours.Shetookibuprofenwithoutanyrelief.Shedescribesthepainalloverherhead,butmoreontheleft
temporalregion.Painis10/10inseverity,constant,nonradiating,associatedwithnausea,butnovomiting.She
alsonoticedsomevisualchangesandphotophobia.Shedeniesanyfever,chills,andweaknessornumbnessof
faceorextremities.Thereisnolossofconsciousness.Shesaysthatshe'sbeenhavingtensionheadachesforthe
pastfewyearsbutdescribesthisas"completelydifferentandtheworstheadacheinmylife".PMH:depression,
chronictensionheadaches.Allergies:None.SH:Quitsmoking,alcohol,andexcessivecaffeineintake.Lives
withhusband.FH:NoFHofaneurysmsorstroke.Medications:Fluoxetine20mgPoQDandIbuprofen400
mgTIDPRN.
Howdoyouapproachthispatient?
Thisisa50yroldWFwhohasaH/Odepressionandchronictensionheadachespresentedwithcompletely
different,suddenonsetsevereheadacheassociatedwithnauseaandvisualchanges.
D.D:
Migraineheadaches
Subarachnoidhemorrhage/Stroke
Temporalarteritis
Acutecongestiveglaucoma
Tensionsheadache(lesslikely)
Sinusitis(lesslikely)
Orderphysicalexam:
General
HEENT/Neck
Lungs
Heart
Abdomen
CNS
Extremities
Results:
Patientisinseverepain.HEENT:PERRLA,EOMI(extraocularmusclesintact).Neckstiffnessispresent.
Mildtendernessoverthelefttemporalareaispresent.Nosinustenderness.Nopapilledema.Novisualfield
defectsnoted.CNS:Completelynormal.Restisnormal.
Order:
IVaccess,stat
IVToradol(ketorolac),stat
ESR,stat
CThead(withoutcontrast),stat
Discussion:
IndicationsforCTheadinpatientswithheadacheare:
1.OccurrenceofseverenewonsetheadacheeveninapatientwithH/Ochronicheadacheandpreviously
normalCT

2.Suddenonsetofsevereheadacheinanypatientof>50yr.age
3.Presenceoffocalneurologicalfindings
Results:
ESRisWNL
CTheaddemonstrateevidenceofsubarachnoidhemorrhage.Nomidlineshift.
Patientisstillinmildtomoderatepain
*InthiscaseiftheCTscandoesnotshowbleeding,youshouldorderlumbarpunctureandthefluidshouldbe
sentforxanthochromia.But,theoptionofxanthochromiaisnotavailableinsoftware.
Order:
AdmitinICU
Continuouscardiacmonitoring
Pulseoximetry,every2hours
NPO
Completebedrest
Urineoutput
Neurochecks,every1hour
CBCwithdiff,stat
BMP,statanddaily
EKG,12lead,onetime(forbaseline)
PT/INR,stat
PTT,stat
TranscranialDoppler(Forbaseline)Notavailableinsoftware
Neurosurgery,consult,stat
Normalsaline,IV,continuous
Acetaminophenwithcodeine(Percocet),oral,continuousforpain
Stoolsoftener(Docusate),oral,continuous
Nimodipine,oral,continuousfor21days
Omeprazole,oral,continuous
Pneumaticcompressiondevices
Discussion:
Basedontheabovefindings,themostlikelydiagnosisinthispatientissubarachnoidhemorrhage(SAH).SAH
canoccurevenwithnormalCTscan.Iftheindexofsuspicionisveryhigh,suchasinpatientswithneck
stiffnessorothermeningealsigns,lumbarpunctureshouldbeperformed.LysisofRBCandsubsequent
conversionofHbtobilirubinoccursoveraperiodoftimeandturnstheCSFyellow.Presenceofthis
xanthochromicsupernatantisclassicforSAH.ST,Twavechangessimilartocardiacischemiamaybeseenon
ECG.Serumelectrolytesshouldbeobtainedatthetimeofadmissionandatleastoncedaily,astheyaremore
pronetodevelophyponatremia('cerebralslatwastingsyndrome')inthefirsttwoweeks.Vasospasmremainsthe
leadingcauseofmorbidityandmortalityinthesepatients.TranscranialDoppler(TCD)ultrasoundassessment
oftheproximalmiddle,anterior,posteriorcerebralarteries,andbasilararteryflowisveryhelpfulinpredicating
thevasospasm.
Treatment:
PatientsshouldbeplacedinICU,andkeptNPO,withcompletebedrest.Stoolsoftenersandmildlaxativesare
usefultopreventstraining.Controltheheadachewithacetaminophenpluscodeine.Stressulcerprophylaxis
shouldbegivenwitheitherH2blockersorPPIs.Patientsshouldhavepneumaticcompressionstockingsapplied
topreventtheDVT.ManagementofbloodpressureisimportantinpatientswithSAH.Uncontrolled
hypertensioncausesmorebleeding.Ontheotherhand,decreasedbloodpressurecancausecerebral
hypoperfusionresultingininfarction.Thegoalistokeepthesystolicbloodpressure120140mmHg.IV
labetalolisthedrugofchoiceifthebloodpressureishigh.CerebralhypoperfusionistreatedwithIVNSbolus,
andvasopressors.Patientswithuncontrolledbloodpressureshouldhaveintraarterialandcentralvenousline.

TheuseofprophylacticantiepilepticmedicationsinpatientswithSAHishighlycontroversial.Hyponatremia
shouldnotbetreatedwithfreewaterrestriction.IVNSwithsaltsupplementsisenoughtotreathyponatremia.
However,sometimes3%hypertonicsalinemaybeneeded.Becauseoftheriskof'centralpontinemyelinolysis',
hyponatremiashouldbecorrectedslowly.AcutehydrocephalusisacomplicationofSAH.Patientsshouldbe
frequentlyexamined,andifnecessaryventriculostomyshouldbeperformed.
Oncethemedicalmanagementhasbeenestablishedandthepatientisstable,astandard4vesselangiogram
shouldbeperformed.
Furthermanagementisbeyondthelevelofthisexam.
PrimaryDiagnosis:
Headache

Location:Office
Vitalsigns:B.P:148/90mmHgP.R:110/min,regularR.R:24/minTemp:37.0C.
C.C:Rapidheartbeatandpalpitations.
HPI:
A48yearoldwhitefemalepresentstoyourofficeforevaluationofrecentonsetofrapidheartbeatand
palpitations.Theyoccurwithoutwarning,areregularinrhythm,andresolvespontaneously.Shealsoreports
thatshehaslost12pounds,duringthelasttwomonths,despiteagoodappetite.Shesaysshe'shavingtrouble
gettingtosleep.Shedeniesanyconstipation,diarrhea,bloodinthestools,ormelena.Shedeniesanyheator
coldintolerance.Shehasnoticeddecreaseddurationofhermenstrualcyclesrecently,andthinksshe'sreaching
menopause.Shedeniesanychestpain,dizziness,syncope,legswelling,shortnessofbreath,orthopnea,or
PND.PMH:NoH/Oheartdisease,HTN,orDM.Nothingsignificantexceptananxietydisorder.Shehasno
knownallergies.FH:ThereisnofamilyH/Osuddendeath.SxH:Thepatientissexuallyactivewithhusband.
Theyusecondomsascontraception.SH:Shedeniessmoking,alcohol,orIVdrugabuse.Medications:None.
ROS:Unremarkable.
Howwouldyouevaluatethispatient?
Completephysicalexamination
Results:
Onexamshehasrapidspeech.Herhandsarewarmandmoist.Milddiffusenontenderenlargementofthe
thyroidisnoted.Restofherexamiscompletelynormal.
Orders:
CBCwithdiff,stat
BMP,stat
ECG,12lead,stat
SerumTSH
SerumFreeT3,andT4
Results:
CBCwithdiffiswithinnormallimits
BMPiswithinnormallimits
EKGshowedsinustachycardia
TSHislow0.08U/mL
T3andT4areelevated
Revieworder:
24hourradioiodineuptake
Followupwiththeresults
Results:
Radioiodineuptakeisincreased
Revieworder:
Propranolol,oral,continuous
Methimazole,oral,continuous
Followupin4weeks.Advisetostopmethimazole4dayspriortofollowup.
Revieworder:
CBCwithdiff
StopMethimazole

Radioiodine,onetime
Followupinonemonth
Discussion:
A48yearoldwhitefemalepresentedwithpalpitations,weightloss(despitehavingagoodappetite),difficulty
sleeping,andmenstrualproblems.Thisisoneoftheclassicpresentationsofhyperthyroidism.Thecombination
ofweightlossandgoodappetiteisclassicforhyperthyroidism.
Approachofapatientwithpalpations:
Acompletehistoryandphysicalexaminationshouldbeperformedinallhemodynamicallystablepatients.
Therearenoevidencebasedguidelinestodirectthelaboratoryworkuponpatientswithpalpitations.We
routinelyobtain12leadECG,CBCwithdiff,BMP,andTSHlevels.Identifycommoncauseslikeanemia,
electrolyteimbalance,andthyroidabnormalities.Iftheinitialapproachdoesnotestablishthedefinitive
diagnosis,furtherevaluationisindicated.Patientsareclassifiedaseitherlowriskorhighriskdependingonthe
riskfactors.
Highriskpatients:
PatientswithH/Osyncopeordizziness.
Patientswithafamilyhistoryofsuddendeath,arrhythmias,orlongQTsyndrome.
Anypatientwithunderlyingorganicheartdisease,whichincludescarringfrompriorMI,cadiomyopathy,
significantvalvulardisease,andHOCM.
Thesepatientsareathighriskofdevelopingventriculartachycardiasoshouldbeevaluatedwitheither
ambulatorymonitoringoraninpatientelectrophysiologicalstudy.
Lowriskpatients:
Patientswithnofamilyorpersonalhistoryofarrhythmias.
NoH/Odizzinessorsyncope.
Nounderlyingorganicheartdisease.
Thesepatientsshouldbereassured.However,ambulatorymonitoring(24hourHoltermonitor)is
indicatediftheECGshowssustainedarrhythmia.
Hyperthyroidism:
ThemostcostaffectiveapproachtoapatientwithsuspectedhyperthyroidismistoobtainserumTSHlevels.A
patientwithnormalTSHisveryunlikelytohavehyperthyroidism.However,itisreasonabletoobtaina
simultaneousfreeT3andT4iftheindexofsuspicionisveryhigh.IftheTSHislessthan0.1mlandfreeT4is
elevated,hyperthyroidismisconfirmed.IftheTSH,freeT4,andT3areelevated,aTSHproducingpituitary
tumorshouldbesuspectedandMRIofthebrainshouldbeordered.Oncethediagnosisofhyperthyroidismhas
beenestablished,theunderlyingcauseshouldbedetermined.
Basically,hyperthyroidismiscategorizedintotwobroadclassifications:
1.Graves'diseaseandtoxicmultinodulargoiter.
2.Postpartumthyroiditis,iodineinduced,orfactitioushyperthyroidism.
Often,Gravesdiseaseisdiagnosedonclinicalgrounds(diffusegoiter,ophthalmopathy)alone.Measurementof
TSHreceptorstimulatingautoantibodies(TSI),isnotroutinelyrecommendedforthediagnosisofGraves
disease.However,a24hourradioiodineuptakeisnecessarytoconfirmGravesdiseaseandtoexcludeother
possibilities(itwillbeelevatedinthefirstgroupanddecreasedinthesecondgroup).Oncethediagnosisis
made,treatmentshouldbestarted.Subacuteandpostpartumthyroiditisareusuallytransientandrequireonly
symptomatictreatment.
Radioiodineisthetreatmentofchoiceforpeopleover20yearsold.Obtainapregnancytestbeforeyoutakea
radioiodineuptakebecauseitiscontraindicatedinpregnancy.Radioiodinecausesdestructionofthethyroidand
oftenassociatedwithasmallriskofthyrotoxicosis.So,allelderlyandpatientswithcardiacproblemsshouldbe

pretreatedwithantithyroiddrugsforatleastonemonth.Antithyroiddrugsmustbestopped3to5dayspriorto
theradioiodinetreatmenttoachieveoptimumiodineuptake.Ophthalmopathymaybeaggravatedby
radioiodinetreatment.So,physiciansoftenstarttaperingcourseofsteroidsatthetimeofradioiodinetreatment
ifthepatienthasevidenceofophthalmopathy.Symptomsofhyperthyroidismcanbeseenupto2to3months
afterthetreatmentofradioiodine.Eitherantithyroiddrugsorbetablockerscanbeusedtocontrolthesymptoms
duringthisperiod.Patientsshouldbefollowedat46weekintervalswithclinicalexaminations,andthefreeT4
level(notTSH).Ifsymptomatichyperthyroidismpersistsaftersixmonths,radioiodineuptakeshouldbe
repeated.
Antithyroiddrugsareprimarilyusedinpregnancy,whenradioactiveiodineiscontraindicatedorinapatientof
lessthan20yearsold.Propylthiouracilisthedrugofchoiceinpregnancy.Theseareassociatedwithgranular
cytopenia,soalwaysobtainabaselineCBCwithdifferentialbeforestartingthetreatment.PTUisalso
associatedwithhepatotoxicityso,baselineLFTsareimportant.Finally,surgeryisindicatedinpregnantpatients
whodonotrespondtopropylthiouraciltherapy.Euthyroidstateisusuallyachievedintwotofourmonths.
FollowthepatientinsixweekswithfreeT4levels.
Betablockers(eg,propranololoratenolol)canbeusedifthepatientistachycardic,anxious,sweating,orhaving
tremors.Ifbetablockersarecontraindicated,suchasinsymptomaticbronchialasthmapatients,verapamilcan
begiven.
PrimaryDiagnosis:
Hyperthyroidism

Location:OutpatientClinic,Pediatrics.
Vitalsigns:Normal.
C.C:21/2yearoldgirlwithabdominalpainandconstipation.
HPI:
Thepatientisa21/2yearoldHispanicfemalewhopresentstodayforevaluationofabdominalpain,
constipationandanorexiathathadbeenpresentforapproximatelyeightweeks.Shearriveswithherparents
whoreportthathersymptomsstartedslowlyatfirstwithsomeanorexiaandthenslowlytheybegantonotice
thatshewasconstipatedmorefrequentlyandhadcomplaintsofabdominalpain.Theparentsnotethatthe
patient'sstoolsarenormalinappearance,otherthanbeingsomewhathardandround.Shehashadnoepisodes
ofrectalbleedingormelena.Shehadnocomplaintsofnauseaandhasnotbeenvomiting.Thepatienthasnot
hadregularmedicalcarebuthasreceivedimmunizationsatalocalhealthdepartment.
Thefamilylivesinahousebuiltinthe1930s.Theyhavebeenremodelingthehomeoverthepastyearanda
halfincludingtearingdownwallsandrefinishingtheflooring.Thehousestillhasitsoriginalwindows.The
fatherworksintheconstructionbusinessandmainlydoestheinitialdemolitionpriortoputtingupanew
structure.Motherstayshomeandwatchesthepatient.Theyhaveaschoolagechildwhoissixyearsofage.
Therearenopetsinthehouse.Theyhavecitywater.Thepatientherselfhasneverhadtroublewith
constipationoranorexiapriortothistime.Thepatient'sgrossmotordevelopmenthasbeennormalupuntilthis
pointandshehasbeenmeetingherdevelopmentalmilestones.Thepatientspendsallofhertimeathome.She
doesnotgooutsideofthehomefordaycare.
Howtoapproachthiscase:
Thisisa21/2yearoldchildpresentingwithanorexia,abdominalpain,andconstipation.Thedifferential
diagnosisisratherbroadandconstipationisanexceedinglycommonprobleminthepediatricpopulation.
However,thischildhassomeredflagsthatmightmakeonewanttopursueaslightlydifferentcourseinthe
workupofherconstipationsymptoms.First,shehasnothadroutinehealthcheckupssoshehasnothada
screeninghemoglobinleveltoidentifyirondeficiencyanemianorhasshepresumablyhadascreeningleadlevel
toidentifychildrenwhoarehighriskofleadpoisoning.Bothofthesestudiesshouldbeobtainedonthispatient
aspartofherconstipationworkup.
Orders:
Completephysicalexam
Results:
General:Thepatientiswelldevelopedandwellnourished,innoacutedistress.HEENT:Slightlypale
conjunctivae,otherwisenormal.Cardiovascular,andLungs:Normal.Abdomen:Bowelsoundsarepresentbut
slightlydiminished.Bellyissoft,nontender,andslightlydistendedwithstoolsfeltintheleftlowerquadrant.
Extremitiesarenormal.Neurologic:Nonfocalandappropriateforage.
Thepatientisa21/2yearoldwithconstipationsoonewouldwanttoatleasttreatthatproblem.Ordersshould
includeabowelregimentoimprovetheconstipationsymptoms.Hypercalcemiaandocculturinarytract
infectionmaypresentwithconstipation.
Orders:
Fingerstickleadlevel(UnfortunatelythisisnotavailableinCCSsoftwareSoyoucandirectlyobtain'blood
lead,quantitative'),
CBCwithdifferential,routine
Basicmetabolicpanel,routine
Calciumlevel,routine
Milkofmagnesia10ccQD,
Docusate20mgQD,

Urinalysis,
Followupappointmentinthreetofivedaystoreviewlaboratorystudiesandseeifimproved.
Results:
Fingerstickleadof70mcg/dl,
Hemoglobinis10.7,hematocritis33,MCVisslightlylowat76cl/cell.
U/AWNL
Discussion:
Thispatienthasanelevatedfingersticklead.Thecurrentguidelinesindicatethatanylevelover9isconsidered
elevated.ThepatientalsohasamilddecreaseinherMCVandherhemoglobinandhematocritareslightly
decreasedsuggestingmicrocyticanemia.Bothirondeficiencyandleadpoisoningcaninduceamicrocytic
anemia.Asthefirsttesttofollowupwiththepatient'selevatedfingerstickleadlevel,oneshouldobtaina
venousbloodleadlevel.
Order:
Venousbloodleadlevel.
Results:
54mcg/dl.
Discussion:
Thispatienthasanelevatedbloodleadlevelatalevelsufficientlyhightowarranttreatment.Treatmentis
aimedatassessingthepatient'senvironmenttoseewhatsourcesofleadmaybecontaminatingthepatient's
environmentchangingthechild'sbehaviors,particularlyhandmouthbehaviorwhichcancontributetothe
ingestionofleaddustensuringthatthechildhasadequatenutrition,particularlycalciumandirontodecrease
leadabsorptionandloweringthepatient'swholebodyleadlevelthroughchelationtherapy.Ingeneral,
chelationtherapyiswarrantedwhenthebloodleadlevelisgreaterthan45mcg/dl.Monotherapyisindicatedup
to69mcg/dlgreaterthan69mcg/dlwarrantstwodrugchelationtherapy.
Orders:
HomeinspectionforsourcesofleadType'Leadpaintassayathome'
Dietaryrecommendationstoincreasecalciuminthediettoapproximately1gramofcalciumperday.Thismay
beobtainedeitherthroughmilkandotherdairyproductsorcalciumfortifiedorangejuiceaswellas
recommendationsforirontherapysincethepatienthasirondeficiencyorsimplyaddanironcontaining
multivitaminifthepatientdoesnothaveirondeficiency.
Serumiron,ferritin,andTIBC
Succimer(DMSA)chelationtherapy,oralcontinuous
Liverfunctiontests,erythrocyteprotoporphyrinbaselinepriortosuccimertherapy
Followupinonemonth
Ifinspectionrevealsthehomeasthesourceofleadpoisoning,thenleadabatement(Type,'Leadabatement
agency')withinthehomeisalsoanecessarypartofthispatient'splan,andthepatientshouldberemovedfrom
thehomewhiletheabatementisoccurringandwhilethefamilyisremodeling.
Succimer,alsoknownasDMSA,isthefirstdrugofchoiceforchildrenwhohaveelevatedleadlevelsinthe45
100mcg/dlrange,atadosageof350mg/M2perdoseeveryeighthoursorallyforfivedaysandevery12hours
foranother14days.ToxicitiesassociatedwithsuccimerincludeGIdistress,rashes,elevatedliverfunctiontests
anddepressedwhitebloodcellcount.ThereforewhenorderingsuccimerfurtherordersincludeCBCand
hepaticpanel.Oneshouldobtaintheseatbaselineaswell.
Results:
Patienthascompletedthechelationtherapywithoutanysideeffects.
Ironstudieswerenormal.
Baselineliverfunctionswerenormal.

Erythrocyteprotoporphyrinlevelwaselevated.
Herconstipationhasimprovedandshenolongerneedsthemilkofmagnesiumordocusate.
Orders:
Repeatleadlevel,(Typebloodlead,quantitative)
CBC,
Erythrocyteprotoporphyrintoday.
Discussion:
Exposuretoleadcancausesubtlecognitivedefectsinchildren.Currentlytheacceptedlevelforthethresholdof
concernisabloodleadlevelgreaterthan9mcg/dl.Becauseofincreasedpublicawarenessofthetoxicities
associatedwithlead,screeningprogramsthatroutinelyscreenforleadaswellasanemiahavebeenable
successfullytoidentifychildrenwhohaveincreasedexposuretoleadandpossibletoxicityfromit.Patient
populationswhoareatincreasedriskofhighleadlevelsincludeimmigrantfamilies,particularlyHispanicones,
whomayuseceramicwareglazedwithleadpaintchildrenwholiveinpovertywhoareyoungerthansixyears
ofagewhoareAfricanAmericanorwhodwellwithinacity.Childrencanhaveincreasedexposureifthey
liveinahouseorspendconsiderabletimeinastructurebuiltbefore1950whenuseofleadbasedpaintwas
prevalent.Childrenwhosefamilymembersworkinareasthatmayhaveelevatedleadlevels(includingmetal
refineries,batteryrecyclingplants,maintenanceworkersonbridgesandboats,anddemolitionworkers)may
receive"secondhand"dustexposurefromcontaminatedclothing.Othersourcesincludewindowblinds,
zippers,paintedfurnitureandmineralsupplements,particularlyonesthathavebeenbroughttothiscountryfrom
acountryinwhichleadlevelsarenotvigorouslymonitored.Leaddustisaparticularproblemforchildrenasit
usuallyaccumulatesinplacessuchaswindowsillswherepaintalongthewindowisfrequentlyrubbedwiththe
releaseofdustparticlestoddlerswho,becauseoftheirdevelopmentalstage,tendtomouthobjectsincluding
windowsills,toys,andtheirhandsbecomeexposedtothisdust.
Becauseofmonitoring,mostchildrenpresentwithasymptomaticleadpoisoningthatisnotedonscreening
laboratorytests.Thefingerstickleadlevelistheinitialscreeningtesthowever,becauseofcontamination
problems,thislevelisfrequentlyhigherthanavenousbloodlevelandthereforeanyfingerstickscreenthatis
elevatedshouldbefollowedupin48hourswithavenousbloodleadlevel.Erythrocyteprotoporphyrinlevels
maybeelevatedandcanbefollowedtoseearesponsetochelation.
ItisfeltthatGIsymptomsoccuratapproximately50mcg/dlhowever,somedatasuggestthatnearlyhalfof
childrenwhohavebloodlevellevelsinthe2045mcg/dlrangemayalsohaveGIsymptomswhichmaybe
misinterpreted.
Encephalopathysecondarytoleadpoisoningusuallydoesnotoccurunlessthelevelisexceedinglyhighsuchas
over100mcg/dlandthiswillbeanindicationforpromptchelationtherapy.Asnotedabove,chelationtherapy
isrecommendedforlevelsgreaterthan45mcg/dlwithmonotherapybeingrecommendedforlevelsbetween45
and69.TherearecurrentlyfourdifferentchelatingagentsavailableintheUnitedStatesforleadpoisoning.
Theyaresuccimer,calciumedetate,BAL/dimercaprol,andDpenicillamine.Childrenwhoundergochelation
therapycanexpecttohavetheirbloodleadlevelreboundaboutfourtosixweekspostchelation,presumablydue
tothereleaseofleadfrombonestores.Thosechildrenwhohavelevelsabove100mcgarelikelytohavea
reboundbloodleadlevelgreaterthan45mcg/dlwhichwouldwarrantasecondroundofchelationtherapy.
Thosechildrenwhohavelevelsgreaterthan45willgenerallyhaveareboundintheirleveltoabouttwothirds
ofwhatithadbeenpriortochelationtherapy,andtheymayormaynotwarrantfurtherchelationtherapy.
Therefore,itisimportanttodofollowupbloodleadlevelmeasurementsonthesechildren.
Orderreview:
Followupappointmentinone46weekswitherythrocyteprotoporphyrinlevelandabloodleadlevel.Repeat
chelationifstillwarranted.
Continuemultivitaminwithiron.Continuecalciumsupplementationinthediet.
Primarydiagnosis:
Leadpoisoning.

Note:Youmaygetacaseofleadpoisoningwithadifferentpresentation.Forexample,childmaypresentwith
fatigue,lethargy,notdoingwellinschoolandonexamhehaspallor.YouorderaCBC,whichshows
microcyticanemiaandbasophilicstipplingetc.

Location:OutpatientClinic.
Vitalsigns:Bloodpressure:137/79supine,124/68erectHeartrate:85/min,regularRespirations:16/min
Temperature38.8C.
C.C:Cough.
HPI:
Thepatientisa65yearoldwhitemalewithapastmedicalhistorysignificantforCOPDwitha60packyear
smokinghistory.Hecontinuestosmokecigarettesoccasionally,althoughhehasrecentlycutback.Hepresents
withafivedayhistoryofincreasingcough,increasedsputumproductionandfeverupto38.7forthelasttwo
days.Hehasdyspneaonexertionandcurrentlyhassomemilddyspnea.He'shaddecreasedappetite,poorPO
intakeandatenpoundweightlossoverthepasttwomonths.ROS:Hedeniesanychills,hemoptysis,chest
pain,pleuriticchestpain,abdominalsymptoms/pain,diarrhea,constipation,bloodperrectum,ormelena.He
deniesanyneurologicsymptoms.Therestofhisreviewofsystemsisnegative.Hehadasimilarillness
approximatelyseventoeightweeksagowhichwastreatedwithcefuroximeandazithromycin,andthepatient
reportsthatafterthatcourseoftreatmenthegotbetterandhasbeenwellforthepastthreeweeksuntilthelast
fivedayswhenhehadreturnofthecough,increasedsputumproductionandfever.FH:Nothingsignificant.
Medications:Takesalbuterolpuffsasneeded.Allergies:None
Howtoapproachthiscase:
ThepatientisanelderlymanwithasignificanthistoryofCOPDnowpresentingwithasecondpneumoniainthe
courseofabouttwomonths.HeneedsanevaluationrightnowofhisO2saturation,physicalexam,andthenan
explorationintotheetiologiesbehindhisrecurrentpneumonia.Thesuspicionformalignancyisveryhighgiven
his60packyearsmokinghistory,theweightlossnotedinthereviewofsystems,andtherecurrenceofa
pneumonia,particularlyifthepneumoniaisinthesameplaceasthepriorone.
Orders:
Pulseoximetry
Results:
O2saturation90%onroomair
Order:
Physicalexam:HEENT/Neck,lungs,heart,abdomen,andextremities,
Results:
General:Elderlywhitemaleinnoacutedistresswithtemporalwasting.HEENTshowsaclearoropharynxwith
upperandlowerdentures.Thereisnonecklymphadenopathy.Temporalwastingispresent.Conjunctivaeare
slightlypale.Cardiovascularisnormal.Lungs:Decreasedbreathsoundsthroughoutwithralespresentonthe
rightupperlungfieldsposteriorlyanddecreasedbreathsoundsintherightupperlunganteriorly.Increased
anteriorposteriordistanceonthechestwithbarrelchesthabitus,andmildsupraclavicularretractions.
Abdomen:Slightlyobesebutotherwisenormal.Extremities:Thereisbilateraltendernessofwrists,withnails
morecurvedlongitudinallyandbaseofnailbedfluctuantinallfingers.Rightindexfingerandmiddlefinger
shownicotinestaining.
Discussion:
Thepatienthashypertrophicosteoarthropathynoticedonexamination.Itischaracterizedaschronic
proliferativeperiostitisoflongbones,clubbingoffingersandsynovitis.Itismorerelatedwithsquamousand
adenocarcinomaofthelungs.Symptomsofthisconditionmayoccurbeforetheactualmanifestationoflung
carcinoma.AsaNo.1killerCancerinUSA,itremainsveryimportanttoknowthedifferentmanifestationsof
lungcarcinoma.Thispatient'sfindingofhypertrophicosteoarthropathyissignificantforlungcarcinomainthe
contextofhisrecurrentpneumoniaanddyspnea.

Orders:
Shifttohospitalward.
Beginsupplementaloxygentherapyat2lpmbynasalcannula(Typeoxygeninhalation)
IVaccess
IVfluidsat100ccanhourwithnormalsaline
Urineoutputs,Q4hours
Vitals:Every4hours
Pulseoximetryevery4hours
Activity:Bedrestwithbathroomprivileges
ChestXray,PAandlateral,stat
Bloodcultures,stat
Coughedsputumsampleforgramstain,cultureandcytology.
CBCwithdifferential,stat
Basicmetabolicpanel,stat
BeginantibiotictherapywithLevofloxacin(Levaquin)orallyorIVafterculturesobtained
AlbuterolandipratropiumnebulizedtreatmentsQ6HandalbuterolQ2HPRNforshortnessofbreath.
Results:
ChestXrayshowsaninfiltrateintherightupperlobewithsomeelevationofthetransverse/minorfissure
anteriorly.Therearenoeffusions.Thereisevidenceofhyperinflationandchroniclungchanges.
WheneverCalungissuspectedonthebasisofclinicalfeaturesandinitialdiagnostictests,weneedtoperform
advancedimagingproceduresandotherteststoestablishthetissuediagnosisoflungcancer.CTscanofchestis
doneformediastinalandpleuralextensionofthesuspectedlungtumor.FortissuediagnosisofthelungCa
followingdiagnosticmodalitiesareavailable:
Sputumcytology
Biopsyofsuspiciouslymphnodes
Flexiblefiberopticbronchoscopy:Biopsyspecimensaretakenwhenanyendobronchiallesionisnoted
Pleuralbiopsyifpleuraleffusionispresent
Mediastinoscopyandanteriormediastinotomywhenthereissuspicionofmediastinuminvolvementbythe
tumor
TransthoracicFNAbiopsyunderCTorfluoroscopicguidancewhenaperipheralpulmonarynoduleispresent
Orderreview:
SpiralCTscanofthechest
Arrangeforbronchoscopy
ConsultPulmonaryMedicine/cardiovascularsurgeryforbronchoscopy
CBC/diffwithbasicmetabolicpaneldaily
Continuesupplementaloxygentherapy
Results:
Thepatientundergoesbronchoscopicexaminationthefollowingday.Hetoleratestheprocedurewell.Broncho
AlveolarLavage(BAL)samplesaresentforcytology,gramstain,culture,AFBsmear,andfungalculture.The
patientcontinuestoshowslightimprovementinhisoxygensaturationsandoverallfunctionwiththe
levofloxacintherapy.HisIVfluidscanbediscontinued.Hissupplementaloxygencanbeweanedtoroomair.
Resultsofthebronchoscopyshowedanendobronchiallesioninthetakeoffoftherightsuperiorbronchus.The
areawasbiopsiedandbrushed.Cytologyrevealsmalignantcellsconsistentwithabronchogeniccarcinomaand
cytologyrevealssmallcellcarcinomaofthelung.
Order:
PulmonaryFunctionTests(PFT)
LiverFunctionTests(LFT)
Serumcalcium,stat
CToftheabdomenandpelvis

MRIbrainwithandwithoutcontrast
Bonescan
Consultoncology
Consultradiationoncologist
Quittobaccouse
Supplementdietwithhighproteinnutritionalshakes
Considerchangingalbuterol/ipratropiumnebulizertoMDI(MetereddoseInhalers)
Primarydiagnosis:
Bronchogeniccarcinomapresentingasobstructivepneumonia
Discussion:
Lungcancerincidenceisabout35per1000personsperyearandthemajorityofpatientsaresymptomaticat
presentation.Localsymptomsincludecough(70%),hemoptysis(40%),dyspnea(40%),chestpain,hoarseness,
superiorvenacavaobstruction,andwheezing.Systemicsymptomsincludeweightloss,anorexia,weakness,
andfever.Signsonexamincludebonepain,hepaticdysfunction,lymphadenopathy,andneurologicalorcranial
nerveinvolvement.Almostallpatientsdiagnosedhaveconstitutionalsymptoms,suchasthecaseabove.Lung
cancerstypicallymetastasizetobone,liver,lymphnodes,brain,andsofttissue.Unfortunately,screeningwith
chestradiographyandsputumcytologyinpatientsatriskhasnotbeenfoundtodecreasecancermortality
althoughitmaydetectdiseaseatanearlierstage.
Workupofsuspectedcasesincludesbronchoscopyforcytologyandvisualization,aswellasHighResolution
CT(HRCT)ofthechest.Ifsmallcelllungcancerisfound,thenanMRIorCTofthebrain,CToftheabdomen
andpelvis,andbonescanshouldbeperformedinallpatientsbecauseofthehighincidenceofmicro/macro
metastasisbythetimeofdiagnosis.Bonemarrowaspiration/biopsyiswarrantedinpatientsofSCLC(smallcell
carcinomaofthelungs)whenthereiscytopeniaorincreasedLDH.Thisworkupisalsoindicatedinpatientsof
NSCLCinwhominvolvementofthespecificorgansissuspected.PFT'swithdiffusioncapacity,spirometry,
andoxygensaturationsshouldbeobtainedearlyon.Afterstaginghasbeencompleted,about3040%of
patientswillhavelimitedstagediseaseand6070%willhaveextensivedisease.

Location:Emergencyroom
Vitals:B.P:110/70mmHgP.R:100/minuteTemperature:102FR.R:15/minute.
C.C:Feverandchills.
HPI:
A54yearoldretiredbusinessmanisbroughtintotheemergencyroom.Familymembersreportthathehashad
amildfever,chills,andbodyaches,fortwodays.However,thismorningthepatientexhibitedahighgrade
feverwithchills,rigors,alteredmentalstatus,andasevereheadache.Heisnauseatedandhadnonbloody
vomiting.Thepatientdeniesanyneckpain,sensorychangesinhisextremities,weakness,seizures,orvisual
changes.Hisbowelandbladderfunctionsareintact.PMH:Significantforhypertensionthelasttenyearsand
hasbeentakingatenolol50mg.oncedaily.SH:Hedeniessmokingordrinkingalcohol.Hehasnoknown
allergies.FH:Nothingsignificant.ROS:NoH/Oheadtrauma.Restisunremarkable.
Howwouldyouapproachthispatient?
Thisisa54yearoldmalewithatwotothreedayhistoryofhighgradefeverwithchills,severeheadache,
vomiting,andalteredmentalstatus.Themostlikelydiagnosisiseithermeningitisorencephalitis.Itisdifficult
todifferentiateencephalitisfrommeningitis,onclinicalgroundsalone.Allpatientsshouldbetreatedashaving
meningitis,untilprovenotherwise.
Orderphysicalexamination:
HEENT/Neck
CNS
Heart
Lungs
Abdomen
Extremities
Skin
Results:
Ongeneralexaminationthepatientappearsalert,awake,andorientedtotime,place,andperson.Thepatient
appearsmildlyconfused,andsleepy.Heappearsveryill.Theonlypositivefindingsonexamareneckstiffness
andKernigsign.Nofocusofinfectionorotherabnormalitiesarefound.Fundoscopydidnotrevealany
papilledema.
Orders:
Pulseoximetry,statandeverytwohours
IVaccess
IVNS,100cc/hr
NPOexceptmedications
Holdhisatenolol
Completebedrest
DVTprophylaxis(Type'Pneumaticcompressionstockings')
VitalsQ2hours
Urineoutputeverytwohours
Headelevation
Bloodcultures,stat
Urinalysis,stat
Urinecultureandsensitivity,stat
CBCwithdiff,statandQday
BMP,statandQday
PT/INR,stat
PTT,stat

Phenergan,IVPRNforvomiting
Acetaminophen,oral,PRNforheadacheandfever
Oncethebloodculturesareobtained:
IVceftriaxone,continuous
IVvancomycin,continuous
Lumbarpuncture,stat
SendtheCSFforcellcount,protein,glucose,Gramstain,Fungalstain,culture,andsensitivity
Results:
GramstainoftheCSFshowsGrampositivecocci
CBCshowedelevatedwhitecountwithleftshift
BMPisnormal
PT/INR/PTTiswithinnormallimits
Revieworders:
Changetheantibioticaccordingtotheorganismandsensitivities.Donotforgettostoptheinitialor
unnecessaryantibiotics.
Orderinterimhistoryandfocusedphysicalexameveryfourhoursuntilyouseeimprovement,thenQ12hours.
Oncethementationisimprovedstartclearliquids,thenadvancethediet.Order'outofbedtochair'.
D/CdailyCBCwithdiff,andBMPwhennolongerrequired.
Discussion:
Basically,thispatientisshowingsignsandsymptomsofmeningitis.Heneedstobehospitalizedimmediately
becauseofhisalteredmentalstatus.ThepatientshouldbekeptNPO.StartIVwithnormalsalinebecausethe
patientisonNPOandhisdiastolicbloodpressureis70,inspiteofhavingahistoryofhypertension.Blood
shouldbedrawnimmediatelyandsentforCBCwithadifferential,BMP,bloodcultures,andcoagulationstudies
(PT,INRandPTTtoruleoutthepossibilityofDICandtoobtainhisbaselinecoagulationstudies).Immediately
afterorderingtheseinvestigations,intravenousantibioticsshouldbestartedwhicharemostlyempirical.The
useofpriorimagingstudies,likeaCTscanofthebrain,isnotnecessarytoproceedwithalumbarpuncture.In
apatientwithanormallevelofconsciousness,withoutanyfocalneurologicalsigns,alumbarpuncturecanbe
safelyperformedevenwithoutpriorimagingstudies.Werecommendstartingintravenousantibiotics,even
beforeobtainingalumbarpuncture.Antibiotictherapyforseveralhours,priortolumbarpuncture,willnot
significantlyaltertheCSF,WBCcount,glucoseconcentration,ortheresultsofculture.However,blood
culturesshouldbeobtainedpriortostartingantibiotics.CSFshouldbesentforgramstain,cultureand
sensitivity,protein,glucose,andcellcountwithadifferential.Ifthepatienthasahistoryofseizures,withfocal
neurologicalsigns,herpessimplexshouldbeconsideredandempiricalIVacyclovirshouldbestartedalongwith
IVantibiotics.InallHIVpatients,CSFshouldbesentforcryptococcalantigenassaytoruleoutcryptococcal
meningitis.
Inacutebacterialmeningitis,theCSF,WBCcountwillbeelevatedandredbloodcellswillbeabsentunless
thereisatraumatictap.Glucoseisusuallylow(lessthan40mg/dl)andtheproteiniselevated(morethan40
mg/dl).Gramstainisusuallypositivein7090%ofuntreatedpatientsandcultureispositiveinaround80%of
cases.Theuseofempiricalantibioticsdependsonthepatient'sageandriskfactors.
Ininfantsoflessthanthreemonths,cefoxitinplusampicillinshouldbegiven.Cefoxitincoversmostof
thegramnegativesandampicillinistocoverListeriameningitis.Dexamethasonehasbeenindicatedfor
H.influenzameningitis.
Immunocompetentchildrenofmorethanthreemonthstoadultsageoflessthan50yearsshouldreceivea
thirdgenerationcephalosporin,preferablyceftriaxoneplusvancomycin.
Adultsofmorethan50yearsofageandindividualswithalcoholismorotherdebilitatingillnessesshould
receiveceftriaxoneplusvancomycinplusampicillin(tocoverListeria).
Meningitis,whichdevelopsafterheadtrauma,orneurosurgicalprocedures,orinpatientswith
neutropenia,shouldreceivevancomycinplusceftazidime.Ceftazidimecoversgramnegativeorganisms,
preferablyPseudomonas.

Oncetheorganismhasbeenidentifiedongramstain,antibioticsshouldbedirectedagainstaspecificorganism.
Ifyoufindgramnegativebacillionthegramstain,ceftriaxoneagainisthedrugofchoice.Ifyoufinda
Pseudomonasonthegramstainandtheculture,thedrugofchoiceisIVceftazidime.Ifyoufindgrampositive
cocciinclusters(staphylococcus),IVnafcillinisthedrugofchoice.Firstgenerationcephalosporinsshouldnot
beusedforstaphylococcusinfectionsbecausetheydonothavehighpermeabilityintotheCSF.IVvancomycin
isthedrugofchoiceforpenicillinallergicpatientsandmethicillinresistantStaphaureus.IftheGramstain
showsHaemophilusinfluenza,IVceftriaxoneisthedrugofchoice.Ifthepatientishavingmeningococcal
meningitis,thepatientshouldbeplacedinrespiratoryisolationandthepatientcanbetestedforterminal
componentcomplementdeficiencies(C6C9).IfyouidentifyListeriamonocytogenesandthepatientisan
immunocompromisedorundergoingdialysis,IVampicillinplusIVgentamicinshouldbegivenforatleastthree
tofourweeks.Usuallytheperiodofantibioticdosesisinbetween1014daysofintravenousantibiotics.
Primarydiagnosis:
Bacterialmeningitis

Location:Emergencyroom
Vitalsigns:B.P:130/70mmHgH.R:80/minR.R:18/minTemperature:38.7C.
HPI:
A50yearoldlady,withahistoryofchemotherapypostasuccessfulbreastsurgery,cametotheERwitha
fever.Patientwasdiagnosedwithbreastcancerthreemonthsearlier.Sheunderwentsurgerythatwasfollowed
bytwocyclesofcombinationchemotherapy.Shewasadvisedtocomebacktotheofficeimmediatelyifshe
developedanyfever.Shedeniesanycough,cold,headache,neckstiffness,SOB,chestpain,diarrhea,
abdominalpain,bloodinthestools,ulcers,orburningurination.Herothermedicalproblemsincludereflux
diseaseandosteoarthritis.ShetakeslansoprazoleQD,andacetaminophenasneeded.Shedeniessmoking,
alcohol,andIVdrugabuse.Familyhistoryisnothingsignificant.Sheisallergictosulfadrugs,andcodeine.
ROSisunremarkable.
(Patientwithhistoryofchemotherapyandfevershouldmakeyouthinkaboutpossibleinfectionsecondaryto
immunocompromisedstatus.Firststepistogetagoodhistoryandphysicalexam.)
Orderphysicalexamination:
Generalappearance
Skincheckforskinlesions
Lymphnodes
HEENT/Neckevidenceoffungalinfection.
Chest/Lungevidenceofrespiratoryinfectioni.e.decreasedbreathsounds,rales,rhonchi.Lungsarethemost
frequentsiteofinfectioninimmunocompromisedpatients.
Heart/CVS
Abdomen
Extremities
Neuro/Psych.mentalstatusevaluationlookingformeningismorfocaldeficits
Note:Digitalrectalexamisnotroutinelyperformedastraumaofthesefrailmucosalareascanprecipitate
infection.However,wegenerallyinspecttheperianalareasforanyfocusofinfection.Ifthereissuspicionfor
prostatitisorperirectalabscess,itcanbeperformedafterthebroadspectrumantibioticshavebeenadministered.
Results:
Completelynormalphysicalexam
Order:
Continuelansoprazole
IVaccess
CBCwithdifferential,stat
Urinalysis,urineculture&sensitivity,andGramstain,stat
Bloodculture,stat
SputumGramstain,andcultures(ifthepatienthassymptoms)
Comprehensivemetabolicpanel,stat
ChestXray,PA,andlateral,stat
Ceftazidime,IV,continuous
Treatment:
Admittofloor
Regulardiet
ActivityAstolerated
NursingVitalsQ4hours
Acetaminophen,oral,asneeded(Donotgivecontinuously,asyouhavetomonitorwhetherheisrespondingto
antibioticsornot)

CBCwithdiff,daily
ConsiderPT/PTTforbaselineifthereisariskofdevelopingDIC.
ConsideruseofNeupogen(GCSF)Notinthispatient
Reexaminethepatientinsixhours,obtaininterimhistory,thenseethepatientQ12hoursandmonitor.
Considerchangingtheantibioticsaccordingtothecultureandsensitivities
Duringdischarge:
After3daysD/CIVantibiotics
Startoralantibiotics
D/CdailyCBCwithdiff
Educatepatientandfamily:
Consolepatienttoavoidpeoplewithcold/flu
Consolepatienttoseekmedicalhelpifafeverdevelops
FinalDiagnosis:
Febrileneutropeniasecondarytochemotherapy
Discussion:
Feverinaneutropenicpatientisamedicalemergency.Febrileneutropeniaisdefinedasasingletemperatureof
>38.3C(101.3F),orasustainedtemperature>38C(100.4F)formorethanonehour.However,patients
whoareoncorticosteroidsmaynotdevelopfever.Neutropeniaisdefinedasanabsoluteneutrophilcount
(ANC)<500cells/mm3.Disruptionoftheskinandmucosalbarrierresultingfromthechemotherapyoften
resultsinseedingofbacteriaintothebloodstream(bacteremia)andthemostcommonsiteofmucositisisin
gastrointestinaltract.Chemotherapyalsoresultsinimpairedimmunologicfunction.
ThefrequentlyidentifiedorganismsinfebrileneutropenicpatientsareGramnegatives,particularlyP.
aeruginosa.ButtherehasbeenarecentincreaseinGrampositiveinfectionsforseveralreasons.AlthoughGI
tracthasabundantanaerobicorganisms,anaerobiccoverageisnotindicatedintheinitialempiricalantibiotic
regimen.Anaerobiccoverageisindicatedifthereisanyevidenceofnecrotizingmucositis,periodontalabscess,
perirectalabscess/cellulitis,intraabdominalorpelvicinfection,typhilitis(necrotizingneutropeniccolitis),or
anaerobicbacteremia.
EvaluationshouldincludeCBCwithdiff,basicmetabolicpanel,LFTs,bloodcultures,urineGramstainand
cultures,sputumGramstainandcultures,andchestXray.Lumbarpunctureisnotroutinelyindicatedunless
theclinicalfeaturessuggestiveofmeningitispresent.Othertestsareindicatedifthepatienthaslocalizingsigns
orsymptoms.Forexample,patientswithdiarrheashouldbeevaluatedwith:StoolforGramstain,culture,
Clostridiumdifficiletoxin,andovaandparasites.
Allcentrallinesandcathetersshouldbeinspectedcarefullyandmayneedtoberemovedifclinicallyinfected
andinfectionnotrespondingtoappropriateantibiotics.Wegenerallyrepeatthebloodcultureifthefever
persistsformorethan48hours.Chestxrayshouldalsoberepeatedforpersistentpulmonarysymptoms.The
findingsonchestXraymaybesubtleorabsenteveninapatientwithpneumonia.Itisimportanttoremember
thattheabsenceofneutrophilsonsputumGramstain,bloodsmear,orCSFdoesnotexcludethepresenceof
infection.
Treatment:
TheempiricalantibioticchoiceshouldcovertheGramnegativeorganismsespeciallyPseudomonas.The
followingarethecommonlyusedregimens.Weusuallyprefermonotherapy.
Monotherapy:Cefepime,ceftazidime,imipenem,ormeropenem.
Doublecoverage:Aminoglycoside+extendedspectrumantipseudomonalpenicillin.
WhenshouldIconsidervancomycin?
Ingeneral,vancomycinisaddedifthereisnoresponsetotheabovemonotherapyin23days.Theadditionof
vancomycintotheinitialempiricantibioticregimenshouldbeconsideredinpatientswhopresentwith

hypotension,mucositis,skinorcathetersiteinfection,historyofMRSAcolonization,orrecentquinolone
prophylaxis.
Whentoaddantifungaltherapy?
AntifungaltherapywithamphotericinBshouldbeconsideredatfivetosevendaysofneutropeniainpatients
withpersistentfever.
WhentouseGCSF:
Thebenefitofcolonystimulatingfactorsasanadjuvanttoantibiotictherapyinanuncomplicatedfebrile
neutropenicpatentsisnotprovenandisnotindicated.However,itisconsideredinpatientswithpredictivepoor
outcomesuchaspatientswithANC<100/L,uncontrolledprimarydisease,pneumonia,hypotension,
multiorgandysfunction,orinvasivefungalinfection.
Generalapproachtothetherapy:
Whatistheinitialantibioticchoice?
Doesthepatientneedvancomycin?
Ifyes,ordervancomycin+ceftazidime.
Ifno,orderceftazidimeoraminoglycoside+extendedspectrumantipseudomonalpenicillin.
Ifpatientbecameafebrileinthreedays:changetooralantibiotics,eithercefiximeorquinolone.
Ifpatientisstillfebrileafterthreedays:stopthevancomycinifaddedinitiallyandculturesarenegativeadd
vancomycin,ifnotadministeredinitially,obtainrepeatcultures,andchestXray.
Durationoftherapy:
Ifthesourceoftheinfectionisidentified,antibioticsshouldbecontinuedforthestandardduration(14daysfor
Staphaureusbacteremia).Ifthesourceisunknown,thenthefollowingapproachisindicated:
Ifthepatientisafebrilein3daysandtheANCis>500,stopantibioticsafter7daysIfthepatientisafebrileand
theANCislessthan500continueantibiotics.

Location:EmergencyRoom.
Vitalsigns:Temperature39.3C(103F)heartrate112/min,regularbloodpressure112/70mmHgrespirations
12perminute
HPI:
Thepatientisa39yearoldwhitemalewhopresentswithatwodayhistoryofincreasingfeversandchillswith
atemperatureupto39.2thepreviousevening.Hecomplainsofanorexia,diffusejointpains,andbackpain.He
alsohassomemyalgiasandhasnotedseveralpainfulspotsonhisfingers.Hedeniesanychestpain,SOB,
palpitations,cough,abdominalpain,headache,andseizures.Heisfeelingnauseatedbutnovomiting.His
REVIEWOFSYSTEMSispositiveforahistoryofrecentIVdruguse.Hispastmedicalhistoryissignificant
forpersonalitydisorder,tobaccoabuse,andalcoholabuse.SH:Hesmokes2PPDforthepast15years,drinks
alcoholalmosteveryday.HeadmitsusingIVdrugabusefromthepast5years.HewastestedforHIV,and
HepatitisBandCrecentlyforpreemploymentandwerenegative.Heisallergictosulfa.Heisnotonany
medication.Howdoyouapproachthiscase?
Orderphysicalexam:
General
HEENT/Neck
Lungs
Heart
Abdomen
Extremities
Skin
Neuro/psych
Results:
General:Thepatientisanillappearingwhitemalewhoappearshisstatedage.HEENT:Oropharynxisclear,
exceptforsomepalatalpetechiae.Pupilsareequal,roundandreactivetolight.Conjunctivaeshowasmall
hemorrhage.Funduscopicexaminationisunremarkable.Neck:Supplenolymphadenopathy,thyromegalyor
bruits.Cardiovascular:Tachycardic,softholosystolicmurmurheardattheleftlowersternalborder,increased
byinspirationanddecreasedbyexpiration.Lungs:Cleartoauscultationbilaterally.Abdomenissoft,
nontender,nondistended.Nohepatosplenomegalyappreciated.Extremities:Theotherarmshowstworecent
needletractmarks.Thereisafinepetechialrashnotedofthebilaterallowerextremitiesbelowmidtibia.
Severaldigitsonthefeethavesplinterhemorrhages.Inaddition,thedigitsofthehandshowseveralsplinter
hemorrhages.Therearetwopalpablepainfulsmallviolaceousnodulesonthedigitsoftherighthand.
Revieworder:
Pulseoximetry,stat
IVaccess,stat
CBCwithdifferential,stat
BMP,stat
PT/PTT,stat
Bloodcultures,statevery10minutesx3
UA,stat
ChestXray,PAandlateral,stat
ECG,12lead,stat
Urinetoxicologyscreen,routine
Results:
O2saturationis97%onroomair.CBCwithdifferentialshowsawhitecount18,000/cmmwith89%
polymorphonuclearleukocytes,10bandneutrophils,12lymphocytes,hemoglobin12,hematocrit35.5,and
platelets309,000/cmm.Xrayofthechestshows2verysmallwedgeshapedinfiltrateswithcavitation.
ElectrolytesarenormalBUNis25andcreatinineis0.9.PT/PTTareWNL.EKGshowssinustachycardia.

UAisWNL.Toxscreenispositiveforopiates.
Order:
Vancomycin,IV,continuous
Gentamicin,IV,continuous
Acetaminophen,oral,onetime(forfever)
Normalsaline,IV,continuous
*Admittofloor/ward
Vitalsevery4hours
Pulseoximetryevery4hours
Urineoutput,routine
Bedrestwithbathroomprivileges
Pneumaticcompressionstockings
NPO
*Advancetheclockfor8hours
TEE(Transesophagealechocardiogram),stat
CBCwithdiff,nextmorning
Considerorderingfollowingifhewasnottestedbefore:
HbsAg,routine(ForhepatitisBscreening)
HepatitisCantibody,serum,routine
HIV1/HIV2serology,routine
Check"Callmewiththenextavailableresult"
Results:
Preliminarybloodcultureresultsshowstaphylococcusaureus,methicillinsensitivegrowingin4outof4
bottles.Basicmetabolicpanel:Normalelectrolytesandrenalfunction.Resultsoftransesophageal
echocardiogramrevealvegetationsonthetricuspidvalveapproximately5mminlength.Thereisnoevidence
ofperivalvulardisease.
Revieworders:
D/CVancomycin(doubleclickonthisorderthesoftwarewillaskyou,doyouwanttocancelthisorder?)
OrderNafcillin,IV,continuous
Centralineplacement,routine(forcontinuedIVantibiotictherapyforatotaloffourtosixweeks).
Dailybloodculturesuntilsterileandonceafterthecompletionofantibioticcourse(4to6weeks)todocument
thecure
Examinethepatientnextday
Interimexam:
Thepatient'stemperaturedowntrendswithaTmaxof37.8onfirstdayofadmission.Heishemodynamically
stable.
Orderreview:
Thepatientiscontinuedonthenafcillinandgentamicinforfivedays.Heisthenswitchedtonafcillinalone
(D/Cgentamicinafter5days).
Smokingcessation
Limitalcohol
Safesex
Seatbeltuse
Noillegaldruguse
Exerciseprogram
AdvisepatientSBEprophylaxis

*Followupin1week.
Primarydiagnosis
RightsidedinfectiveendocarditisfromMSSA
Discussion:
Earlycasesofinfectiveendocarditismaybedifficulttodiagnosisifthereisaconcomitantinfectionelsewherein
thebody.Hisphysicalexamhadseveralfindingssuspiciousforanendocarditisinfectionincludingsplinter
hemorrhages,alsonodesonhishandsonthepulpofhisfingers,petechiaeofthelowerextremitiesandonthe
palate,conjunctivalhemorrhages.
'Dukecriteria'forthediagnosisofIE:
Major:
1.Positivebloodcultures
2.PositiveechocardiogramforIE
Minor:
1.PredisposingfactorssuchasIVdrugabuse
2.Feverof>38C(>100.4F)
3.Evidenceofembolicphenomena
4.Evidenceofimmunologicphenomenasuchglomerulonephritis,Osler'snodesetc.
5.Equivocalbloodcultures
6.Equivocalechofindings
Presenceof2majororonemajorand3minoror5minorcriteriaisrequiredforthediagnosisofIE.
What'empirical'antibioticshouldIuse?
1.InapatientwithH/OIVdrugabuse,theantibioticchoiceshouldcoverMRSA(methicillinresistant
staphylococcusaureus)andgramnegativeorganismsi.e.Vancomycinandgentamicin.
2.Bloodculturenegativenativevalveendocarditisistreatedwithceftriaxoneandgentamicin.
3.Bloodculturenegativeprostheticvalveendocarditisistreatedwithceftriaxoneandgentamicinplus
vancomycin.
WhenshouldIobtainCVTS(cardiovascularthoracicsurgeon)consult?
Theindicationforcardiacsurgeryinpatientswithinfectiveendocarditisisnotfullyagreedupon.However,
someoftheindicationsinclude,moderatetosevereheartfailuresecondarytovalvulardysfunctionorpartially
dehiscedunstableprostheticvalve,prostheticvalveendocarditiswithStaphaureusorStaphepidermidisor
relapseoftheprostheticvalveafterprostheticvalveendocarditisafterappropriateantimicrobialtherapy,and
large(>10mm)hypermobilevegetations,whichcanpotentiallycausesepticembolism.
Inpatientswithsuspectedinfectiveendocarditisoneshouldaimtherapyatthemostlikelyorganism.Staph
aureusisthemostcommonorganismisolatedinthissettingofIVdruguse.Thepatientshouldbescreenedfor
othersignsofendocarditisincludingaurinalysiswhichmayshowhematuria,chestXraywhichinthiscasewas
suspiciousforsepticpulmonaryemboliwhichcanbeseenmoreofteninthesettingoftricuspidvalve
endocarditisinIVdrugusers.Transesophagealechocardiographyhasbecomeanimportantmodeofhelpingto
diagnoseinfectiveendocarditisandhelpguidemanagement.Thereareanumberofcomplicationsofinfective
endocarditis,especiallywithleftsideddiseasethatshouldbemonitoredforvigilantly.Thesearemainly
embolicinnatureandincludeCNSemboluswithstrokelikesyndromesorsubtleneurologicdefects.Embolito
thekidneymaycausefocalglomerulonephritis,whichinduceshematuria,orrenalfailuremayensuresecondary
todiffuseproliferativeglomerulonephritis.Onemayseearrhythmiasincludingvariousdegreesofheartblock
andpericarditis,myocarditisormyocardialabscess.Heartfailureasnotedaboveintheindicationsforsurgery
isalsoapotentialmajorcomplicationofinfectiveendocarditis.Thispatientmanagedtoavoidmostofthe
complicationspossiblybecauseofearlypresentationandearlytreatmentofhisendocarditis.Inpatientsin

whomonesuspiciousofmajorcomplications,itcouldbeappropriatetoobtainCTscansofthehead,chest,
abdomen,andpelvislookingforothersitesofembolicdiseaseorinfarction.Oneshouldmonitoraswellrenal
functionforevidenceofkidneyfailuresecondarytoglomerulonephritisorinfarctionoremboli.Thepatient
shouldreceivefourtosixweekstotalofantimicrobialtherapydirectedattheresultsofthebloodcultures
obtained.InthiscasewithStaphaureusoptimaltherapyiswiththepenicillinaseresistantpenicillin,nafcillin,2
gramsIVQ4H.Healsoreceivedgentamicinforthreetofivedaysinitially.Inpatientsintoleranttonafcillinan
appropriatesubstituteantimicrobialtherapywouldbecefazolinwithorwithoutgentamicin.Inpatientswho
haveallergiesorwhohavemethicillinresistantStaphaureus,vancomycinwouldbetheagentofchoice.

Location:Office
Vitals:B.P:130/76mmHgH.R:130/min,irregularlyirregularpulseTemp:38.3CR.R:18/min.
HPI:A60yrwhitefemalewhohasknownH/OCAD,S/PCABGpresentstoyourofficewith2dayH/O
dizziness,lightheadedness,andpalpitations.Shedescribesthepalpitationsasirregular,andalmostcontinuous.
Shedeniesanychestpain,angina,SOB,orthopnea,PND,orsyncope.Shealsofeltlittlewarmsinceoneday.
Shedeniesanycough,URIsymptoms,dysuria,abdominalpain,andlegswelling.HerROSispositivefor
frequencyofurination.PMH:Shehadundergone3vesselCABG3yearsagoafteranacuteanteriorwallMI.
HerothermedicalproblemsincludeHTN,TypeIIDM,hypercholesterolemia,osteoarthritis,COPD,andgout.
Allergies:Shehasnoallergies.SH:ShequitsmokingafterherCABG.Sheoccasionallydrinksalcohol.She
liveswithherhusbandathome.FH:Fatherdiedattheageof70withMI.Motherdiedattheageof68from
stroke.ShehasonebrotherandonesisterbothhaveHTN,andDM.Meds:ShetakesASA81mgpoqd,
simvastatin20poqhs,lisinopril5mgpoqd,SLNTGprn,glyburide5mgpoQD,metformin850mgpobid,
albuterolpuffsprn,andacetaminophenwithcodeineforosteoarthritis.Howdoyouapproachthispatient?
Orderphysicalexam:
General
HEENT/Neck
Lungs
Heart
Abdomen
Extremities
RectalexamwithFOBT
Results:
HEENT/NeckisWNL.Therearefewralesanddecreasedbreathsoundsnotedatleftlowerbase.Heartexamis
WNL.AbdomenisWNL.NoedemaorJVDnoted.Hemnegativeforstools.
Orderreview:
Pulseoximetry,stat
IVaccess
12leadEKG,stat
Results:
94%onroomair
EKGshowedatrialfibrillationwithrapidventricularresponseataventricularrateof120140/min.ThereareQ
wavesinanteriorleads,consistentwitholdMI.LVHpatternisnoted.
Orderreview:
CardizemIV,bolus
CBCwithdiff,stat
BMP,stat
ChestXray,PAandlateral,stat
CKMB,andtroponinT/I,statandQ8hoursx.
U/A,stat
LFTs,stat
TSH,stat
FreeT4,routine
PT/INR/aPTT,stat
Orderreview:
Admittofloor/ward
Telemetry

VitalsQ4hours
PulseoximetryQ4hours
Order'oldrecords'
Diet:Consistentcarbohydratediet
Activity:Bedrestwithbathroomprivileges
Labs:
HbA1C,stat
AccuchecksQID(4timesaday)
2Decho,routine
Meds:
Continueallhomemedications:ASA81mgpoqd,Simvastatin20poqhs,lisinopril5mgpoqd,SLNTGprn,
glyburide5mgpoQD,metformin850mgpobid,albuterolprn,andacetaminophenwithcodeinefor
osteoarthritis
StartCardizem(diltiazem),IVdrip,
StartHeparin,IV,continuous
PTTevery6hours
DailyCBCwithdiff
Callmewhenlabresultsavailable
Results:
CBCwithdiffshowedaWBCcountof12,000with3%bands.Hbis13.5.Plateletcountis230,000.BMP
showedaNa:140,K:4.0,CL:102,Co2:22,BUN:20,Cr:1.0.ChestXrayshowedsmallleftpleuraleffusions
unchangedfromprevious1yrXray.TSHis1.5.FreeT4isWNL.LFTsareWNL.HbA1Cis7.2.U/A
showedpositiveesterase,50WBC,andmanybacteria.Urinecultureispending.PT/INRis14.0/0.98.PTTis
30.Firstsetofcardiacenzymesnegative.2DechoshowednormalLVfunctionwithanEFof50%,mildly
dilatedleftatrium,normalvalves,andmildhypokinesisoftheanteriorwall.Findingsunchangedfromprevious
echo.NoLV/LAthrombusnotified.
Orderreview:
Urinecultureandsensitivity
BactrimPOQD(TMPSMZ)
Examinethepatientin2hours
After2hours:
Interimhistory
Monitortelemetrystrip:HRisnow90100/minpatientisstillinatrialfibrillation
RepeatEKG:HRisnow90100/minpatientisstillinatrialfibrillation.
Callmewhenneeded.
Examinethepatientinnext6hours
Againorderinterimhistoryandmonitortelemetrystrip
OncetheHRislessthan80
D/CCardizemdrip
StartCardizemPO,continuous
Nextday
StartCoumadinpocontinuous
DailyPT/INR
Examinenextday:
CheckCBC,PT/INR,telemetrystrip
OncethePT/INRisabove2.0,D/CIVheparin
Dischargethepatient
Patienteducation
Outpatientfollowupin3dayswithrepeatCBC,PT/INR

Discussion:
Theprincipleissuesinmanagingapatientwithatrialfibrillationwithrapidventricularresponseinclude:
1.Rhythmcontrolorratecontrol
2.Anticoagulationtopreventsystemicembolization
3.Correctingtheunderlyingabnormality
Rhythmcontrol:Itisindicatedin:1.acuteatrialfibrillation(lessthan48hoursduration),2.Hemodynamically
unstablepatient,3.patientswithacutecoronarysyndromes,4.Patientswithsevereheartfailure.Itcanbedone
byeitherDCcardioversionorpharmacologiccardioversion.DCcardioversionisparticularlyindicatedin
unstablepatients.Instablepatients,andpatientswithareversibleunderlyingproblemcanbedealtwitheither
electricalorchemicalcardioversion.Thecommonlyuseddrugsforrhythmcontrolincludedofetilide,ibutilide,
andtoalesserdegreeamiodarone.Amiodaroneisparticularlyusefulinpatientswithleftventricular
dysfunction.Withoutchronicantiarrhythmictherapy,only2030%ofpatientswhoaresuccessfully
cardiovertedremainsinNSRformorethanoneyear.The2commonlyusedmedicationsforthemaintenance
therapyareamiodarone(patientwithleftventriculardysfunction)orsotalol(inpatientswithCAD).
Ratecontrol:The3mostcommonlyusedAVnodalblockersfortheratecontrolarebetablockers,calcium
channelblockers,anddigoxin.Digoxinisparticularlyindicatedinpatientswithheartfailureorhypotension.In
mostothersituationsdigoxinislesseffectivethanabetablockerorcalciumchannelblocker.Thechoice
betweenaCCBorbetablockerdependsuponphysicianpreference,andthepatientpresentation.Betablocker
ispreferredinpatientswithH/Oangina,acuteMI.Theuseofcalciumchannelblockersispreferredinpatients
withchroniclungdisease.InmostsituationsCardizem(diltiazem)isthepreferreddrugasitiseasyto
administerintheformofIVdripandthedosecanbetitratedforagoalheartrate.Patientswhofailtorespond
withpharmacologictreatmentsrequireEPstudyandradiofrequencyAVnodalHisbundleablation.
ChoosingbetweenRateandrhythmcontrol:
Untilrecently,rhythmcontrolhasbeenthepreferredmethodoverratecontrolforpatientspresentingwiththe
firstfewepisodesofatrialfibrillation.Thethoughtwascontrollingtherhythmcauseslowfrequencyofembolic
events.However,the2majorclinicaltrials(AFFIRM,andRACE)havedemonstratednosignificantdifference
betweenthe2groupsintermsofembolicevents,functionalstatus,orqualityoflife.Thus,bothrateandrhythm
controlareacceptableapproachesandbothrequireanticoagulation.Thereisagrowingsupportforratecontrol.
Anticoagulation:
Thereare3situationswhereyoushouldconsideranticoagulation:1.ChronicAF,2.RecurrentAF,3.Priorand
aftercardioversion.
AFofmorethan48hoursorunknowndurationrequiresatleast3to4weeksofwarfarinpriortoandafter
cardioversion.ThetargetINRis2.5(2.03.0).PatientswithrecurrentorchronicAFshouldbetreatedwith
longtermanticoagulationeveniftheyareinsinusrhythm.Patientswhohaveunderlyingrheumaticvalvular
disease,severeLVdysfunction,orrecentthromboembolismshouldreceiveanticoagulationeveniftheduration
ofAFislessthan48hours.
PatientswithAFoflessthan48hoursdurationwithoutconcurrentvalvulardisease,orsevereLVdysfunction,
orH/OthromboembolismaretreatedwithcardioversionunderIVheparincoveragebutwithoutlongterm
coumadin.
TheotheralternativeapproachforcardioversiontoavoidpriorprolongedanticoagulationisTEEguided
cardioversion.
WhenshouldIadmitapatientwithAF?
Lowriskpatients(patientswithoutvalvulardisease,orsevereLVdysfunction)withAFoflessthan48hour
durationanduncomplicatedclinicalstatuscanbecardiovertedanddischargedfromtheER.Hospitalizationis
necessaryinhighriskandhemodynamicallyunstablepatients.
Searchingfortheunderlyingcause:

ThecommoncausesofnewonsetAFincludeheartfailure,acutecoronarysyndromes,PE,HTN,
hyperthyroidism,andinfections.SerumTSHandfreeT4shouldbecheckedinallpatientseveniftheydonot
havesymptomsofhyperthyroidismasthereisa3foldincreaseofAFinpatientswithsubclinical
hyperthyroidism(LowTSHandnormalfreeT4).IftheAFiscausedbyanunderlyingproblem,cardioversion
shouldbepostponeduntiltheconditionhasbeensuccessfullytreated.However,anticoagulationshouldbe
started.
PrimaryDiagnosis
AtrialFibrillation

Location:Emergencyroom
Vitalsigns:B.P80/40mmHgP.R:130/minR.R:30/min
C.C:Chestpainfromaseveremotorvehicleaccident(MVA).
HPI:
A61yearoldmaninvolvedinamotorvehicleaccident(MVA),broughttotheERimmediately.Hecomplains
ofseverechestpain,10/10,andnonradiating.HealsoC/Oshortnessofbreath.Chestwallimpactedthe
steeringwheel.Nootherhistoryisavailable.Howdoyouapproachthispatient?
Order:
IVaccess
Pulseoxy,stat
Oxygen,inhalation
Ordergeneral,lungs,andheartexam
Results:
Patientisinseverechestpain,hisextremitiesareturningblue.LungsareCTAB/L.Thereisa15cmJVD.S1,
S2muffled.Nomurmursheard.
Pulseoxyshows88%onroomair.
Orderreview:
IVNSbolus,andcontinueat150cc/hr
Elevatethepatientlegs
Continuouscardiacmonitoring
Pericardiocentesis,stat
Results:
Patientstartedimproving
HisBPcameupto100/60mmHG
HRdecreasedto90/min
Orderreview:
StatEKG,12lead
ChestXray,portable
TransthoracicEchocardiogram(TTE),stat
Pericardialfluidforcellcount,stat
ABG,stat
CVTS(Cardiovascularthoracicsurgeon)consult,stat
Results:
12leadEKGshowssinustachycardia,lowvoltageQRScomplexes,andelectricalalternans.
CXRshowedglobularheartwithairfluidlevelinpericardialcavity.
TTERevealedfluidinthepericardialcavity.Impression:Cardiactamponade
IftheCVTSdoesn'twanttooperate,thenthepatientmanagementwillbeasfollows:
ShiftthepatienttoICU
Continuecontinuouscardiacmonitoring
SwanGanzcatheter,stat
DietNPO
Completebedrest
PlaceaFoleycatheter
UrineoutputQ2hours

Pneumaticcompressionsofthelegs
CBCwithdiff,stat
Basicmetabolicpanel,stat
PT/aPTT,stat
ContinueNS@150cc/hr
GastricprophylaxisOmeprazole(20mg)orally,oncedaily
Typeandscreenfor2unitsofblood
Acetaminophen+codeineforpain,continuous(actuallyasneededinreallife)
Note:TransfusebloodiftheHbislessthan8inapatientwithnoactivebleeding,lessthan10inactively
bleedingpatient.EachunitofbloodwillincreasetheHbapproximately1gm%.
Nextday:
D/CFoleycatheter
RepeatTTE
RepeatChestXray
Note:Ifyoudothismuch,yourcasewillendintheexam.Furthermanagementiscomplicated,itisbasedon
thepatientcondition,CVTSrecommendations,etc.
Explanation:
Cardiactamponadeisalifethreateningconditionandshouldbediagnosedandtreatedemergently.The
diagnosisoftamponadeisprimarilyclinical.Myocardialruptureinpatientswithtraumausuallymanifestsitself
ascardiactamponade.TheclassicdescriptionofcardiactamponadeisBeck'striad:Hypotension(100%),
distendedneckveins,andmuffledheartsounds.Theotherusefulfindingsaretachycardia,elevatedcentral
venouspressure,pulsusparadoxus,andcyanosisofthehead,neck,arms,andupperchest.Tamponadeshould
besuspectedinanypatientwithchestinjurywhosehypotensiondonotrespondtofluidsoroutofproportionto
theapparentbloodloss.
Differentialdiagnosisinpatientswithtraumashouldincludetensionpneumothorax(decreasedbreathsounds,
deviatedtrachea,hyperresonancetopercussion),rightventriclecontusion/failure,superiorvenacava
obstruction,rupturedtricuspidvalve,andaorticdissection.Pulmonaryembolism,pericarditis,andcardiogenic
shockshouldbeconsideredinpatientswithouttrauma.
EmergencypericardiocentesisisapotentiallylifesavingprocedureperformedintheED.Emergent
thoracotomyisindicatedwhenthepatientdoesnotrespondtopericardiocentesisandhasrapidlydeteriorating
vitalsignsorcardiacarrest.Afterpericardiocentesis,intrapericardialcatheterisleftinplaceandattachittoa
closeddrainagesystem.Drainshouldbecheckedregularlyforreaccumulationoffluid.Pericardialfluidshould
besentforcellcountinitiallyandperiodically(Q24hours)todiagnoseanimpendingbacterialcatheter
infection,whichcouldbecatastrophic.IftheWBCcountrisessignificantly,thepericardialcathetermustbe
removedimmediately.ASwanGanzcatheterisveryusefultomonitorthecentralvenouspressureanditcanbe
leftinplaceforcontinuousmonitoringandtoassesstheeffectofreaccumulationofpericardialfluid.Patients
shouldhavearepeatechocardiogramandchestxraywithin24hours.
PrimaryDiagnosis:
PericardialTamponade

Location:EmergencyRoom
Vitals:BP:90/60mmHgHR:124/minRR:24/minTemp:37.2C(99F)
C.C:Suddenonsetabdominalpain
HPI:
A55yearoldwhiteobesefemaleisbroughttotheERwithabruptonsetofepigastricpain.Thepainstarted5
hoursago,issteady,boringandsevereinnatureandradiatestotheback.Itismadeworsebylyingsupineand
isbetterbysittingandleaningforwards.Thepatientalsohasnauseaandvomiting.Shedeniesanyfeversor
bowelorbladderproblems.Shehasapasthistoryofepisodicrightupperquadrantpainandfattyfood
indigestion,forwhichsheneversoughtanymedicaladvice.Shehasnoallergiesandisnottakingany
medications.Thepatientdoesnotsmokeanddeniesanyalcoholuse.Familyhistoryisnoncontributory.Rest
ofthereviewofsystemsisunremarkable.
Howwouldyouapproachthispatient?
Theinitialapproachshouldbetotakesomegeneralresuscitativemeasures,adelayinwhichmightbelife
threatening.Simultaneously,thinkofadifferentialdiagnosisandordertherelevantteststoruleinandruleout
thediseaseprocessanditsetiology.Rememberyoualwaysneedathoroughphysicalexaminationbefore
establishingadiagnosis.
OrderNo.1:
IVaccess,stat
StartIVfluids:NormalSaline,IV,continuous
ContinuousBPmonitoring
PulseOximetry,stat
EKG,12lead,stat
ResultsforOrderNo.1:
OxygenSaturationis95%onroomair
EKGshowssinustachycardiawithoutevidenceofischemiaorinfarction
Orderphysicalexam:
Generalappearance
HEENT/Neck
ExaminationofCVS
Examinationoflungs
ExaminationofAbdomen
ExaminationofRectum
Extremities
ResultsofPhysicalExamination:
Generalappearance:Obesefemale,illlooking,diaphoretic,restless.HEENTNormalNoJVD.Lungsare
cleartoauscultationandpercussionbilaterallyCardiovascularS1S2normal,nomurmurs,ruborgallop.
Abdomenissoft,tendernessispresentintheepigastricareabutthereisnorigidity,reboundorguardingbowel
soundsarehypoactive,noorganomegalyorfreefluid.RectalNormalsphinctertone,nohemorrhoidsor
fissures,stoolishemenegative.Extremitiesnoedema,clubbingorcyanosis,nocalftenderness,peripheral
pulsesfeeble.
Meanwhilethenursetellsyouthatthepainisworse
OrderNo.2:
NPO
FentanylorMeperidine,IV,continuous

Phenergan,IV,stat,onetime(fornausea)
Serumamylase,stat
Serumlipase,stat
LFTs,stat
ErectabdominalXray,portable,stat
CBCwithdifferential,stat
BMP,stat
Calcium,stat
Callmewhenthelabresultsavailable
ResultsforOrderNo.2:
BP94/70mmHg
Amylase500IU/LLipase160IU/L
Hgb13g/dl,WBC14,000/uL,Platelet250,000/mm3,leftwardshiftindifferentialcount
BUN30,Creatinine1.1,Sodium131meq/L,Potassium3.6meq/L,Chloride101meq/L,Bicarbonate26
meq/L,Calcium10.1mg.
LFTTotalbilirubin6.0mg%,Directbilirubin4.5mg%,ALT35IU/L,AST40IU/L,Alkaline
phosphatase190IU/L
XrayabdomenNoairunderthediaphragm,nodilatedbowelloops
Discussion:
DifferentialDiagnosis:Thisisapatientwithacuteepigastricpain.Yourdifferentialdiagnosisshouldinclude
acutegastritis,perforatedduodenalulcer,acutepancreatitisandacutecholecystitis.Importantpointhereisthe
descriptionofpainthatradiatestotheback,ismadeworsebylyingsupineandisbetterbysittingandleaning
forwards.Thisistypicalofacutepancreatitisandmayalsobeseenwithaperforatedduodenalulcer.
Confirmingthediagnosis:Thediagnosisofacutepancreatitisisconfirmedbyelevatedamylaseandlipasewith
thelatterbeingmorespecific.Theseenzymesrisetothreetimestheirbaselinevalueswithin24hoursin90%
cases.Besides,youneedtoorderCBCwithdifferentialcount,BMP,calcium,LFTtolookforetiologyand
assesstheseverityofthediseasethatwillguideyouinthemanagementofthepatient.Theseverityofacute
pancreatitisisassessedusingthe"Ranson'sCriteria"whicharenotdiscussedhere.AplainXrayofthe
abdomenwouldhelpinrulingoutairunderthediaphragmandperforatedduodenalulcerthatishighonthelist
ofdifferentialdiagnosis.
Anultrasoundimagingtheliver,gallbladderandbiliarytractisausefulinitialinvestigationinpatientswith
suspicionofgallstonepancreatitisandanabnormalLFT.However,ultrasoundisaverypoormodalityfor
imagingthepancreas.Ontheotherhand,theCTscanoftheabdomencanmissgallstonesandductstonesbut
hastheadvantageofvisualizingthepancreasverynicely.Itmaybeorderedwhenthereisadoubtinthe
diagnosisorwhenyoususpectcomplicationssuchasnecrotizingpancreatitis,pancreaticabscessorpancreatic
pseudocyst(discussedinbrieflateron).Remember,thatultrasoundandCTscanoftheabdomenarenot
routinelyindicatedforestablishingthediagnosisofacutepancreatitisbutmaybeusefulwhenindicated
althoughmanymayarguefororderingboththesetestsinmostcasesofpancreatitis.
DiscussionTheabovepatientresultssuggestthatthepatienthasacutepancreatitiswithhypovolemiaand
prerenalazotemia.
LikelyEtiology:Thetwomostimportantcausesaregallstonesandalcohol.Thepasthistoryofrightupper
quadrantpainandtheLFTresultssuggestpossibilityofgallstonespancreatitisinthisobesefemale.Besides,
othercausestorememberincludehypertriglyceridemia(triglycerides>1000mg%),viralinfections(e.g.
mumps),drugs(e.g.valproicacid,estrogen,thiazidediuretics,azathioprine,didanosine)andfollowingERCP.
Establishingtheetiologyisimportantbecauseunlikeothercauseswheremanagementisconservative,thelatest
recommendationforgallstonepancreatitisisearlyERCP,biliarysphincterotomyandstoneextraction.Manya
timesetiologyisnotestablishedandisbelievedtobesecondaryto"occultbiliarymicrolithiasis."

OrderNo.3:
TransfertowardorICU(ifunstableorhasseverepancreatitis)
Bedrest
Pneumaticcompressiondevises
Omeprazole,oral,continuous(forstressulcerprophylaxis)
Urineoutput
Ultrasoundofliver,gallbladderandbiliarytract,stat
ResultsforOrderNo.3:
BP100/70
Ultrasoundmultiplegallstonesanddilatedcommonbileduct
Meanwhilethepatientcontinuestohavepainbutitisbetterthanbefore
OrderNo.4:
PT/aPTT,stat(preoperativepreparation)
GastroenterologyconsultforERCP:Reason:GallstonepancreatitisrequirespossibleinterventionwithERCP.
Pleaseevaluateandtreat.
Ifthecasestillcontinues,order:
Examinethepatient6hourslater
Order,repeatCBCwithdiff,BMP,Calciumnextday.
Management:
Inmostpatients,acutepancreatitisisamilddiseaseassociatedwithonlyedemaofthepancreatictissuesubsides
spontaneouslywithinfivetosevendays.Thesepatientsaremanagedconservatively.
1.TheyarekeptNPOandputonIVfluids.Inseverecasespatientsmaybeseverelyhypovolemicwith
prerenalazotemia,requiringmassiveamountofIVfluidsforresuscitation.Correctionofelectrolytes
especiallyhypocalcemiaisimportant.
2.Paincontrolisachievedusingnarcoticslikemorphine,meperidineandfentanyl.Contrarytotheprevious
belief,thereisnodatatosuggestthatmorphineincreasesthesphincterofOddipressureandmay
aggravateacutepancreatitisorcholecystitis.
3.Nasogastricsuctionisreservedforpatientswithprotractednauseaandvomitingorileusandisnot
requiredroutinely.
4.Iftheacutepancreatitisissecondarytogallstones(especiallywithtotalbilirubin>5mg%orevidenceof
acutecholangitis),urgentERCPandbiliarysphincterotomywithin72hoursofpresentationcanimprove
outcomebyreducingbiliarysepsis.IfthispatienthadnogallstonesortheLFTwasnormalthenitwould
beappropriatetomanagejustconservatively.
5.AcidsuppressionisnecessaryonlyinseverelyillpatientinICUsettingswheretheriskofstressulcer
gastrointestinalbleedingishigh.
Oncethepainsubsides,thepatientcanbestartedonclearliquidsanddietadvancedastolerated.
Complications:
a)NecrotizingPancreatitisisamoresevereformofpancreatitisthatusuallydevelopsinthesecondweek,
requiringCTscanoftheabdomenfordiagnosis.Itisassociatedwithincreasedmortalityandmorbidity
secondarytomultisystemorganinvolvementincludingpulmonary(ARDS)andrenal(ATN).Thenecrotic
tissueisusuallysterilebutmaygetinfected.ACTguidedaspirationmaybeneededtoconfirminfectionif
patienthaspersistentfever,leukocytosis,andmultisystemorganfailure.Inadditiontotheroutinemeasures
discussedabovethesepatientsrequireenteralfeedingsorTPNandantibioticsifinfectionispresent.The
antibioticofchoiceisPrimaxin(imipenem).Furtherapercutaneousdrainageprocedureormajorsurgical
debridementmaybeneededinverysickpatientswithinfectednecrotictissue.
b)Pseudocystissuspectedinpresenceofseverepainorpersistentlyelevatedamylaselevels.Theseare

diagnosedwithCTscanoftheabdomen.Asymptomatic,nonenlargingpseudocystsoflessthan6cmcanbe
followedclinicallywithserialimagingstudies.
FinalDiagnosis:
AcutePancreatitis,secondarytogallstones

Location:Officevisit
Vitals:BP:120/80mmHgHR:84/minRR:14/minTemp:37.2C(99F)
C.C:"Iamnotfeelingwell,can'teatanythingandmyurinehasbecomedarkyellow"
HPI:
A34yearoldwhitemalephotographercomestotheofficecomplainingofillhealthforlast1week.His
symptomsbeganwithlowgradefever,generalizedbodyachesandfatigue.Hehasbeennauseatedappetiteis
poor,withoccasionalloosestoolsandvomiting.Hehasnothadanyfeverforlast2daysbuthisurinehas
becomedarkyellowincolorandthestoolsseemtobeverylightcolored.Healsocomplainedofrightupper
quadrantdullache.Hedeniedanyhematemesis,melena,weightlossordysuria.Thereisnohistoryofjaundice
orbloodtransfusioninthepast.Hehasnoallergiesandisnottakinganymedications.Thepatientwasaheavy
smokerbuthasdevelopeddistasteforcigarettessincehisillnessstarted.Hedeniedanyalcoholuse.Hehad
beentoMexicoonanassignment3weeksago.Heismarried,liveswithhiswifeanddaughter.Heis
heterosexual,withonlyonesexualpartner.Familyhistoryisnoncontributory.Restofthereviewofsystemsis
unremarkable.
Howwouldyouapproachthispatient?Apatientwithnonspecificconstitutionalsymptomsanddarkyellow
coloredurinesuggeststhatthiscouldapatientwithjaundice.Hisvitalsignsandhistorysuggestthathecanbe
managedasanoutpatientanddoesnotneedadmission.Beforeorderinganytests,orderacompletephysical
examinationtoconfirmyoursuspicion.Thiswillalsohelpyouinformulatingadifferentialdiagnosisand
orderingtherelevanttests.
Orderphysicalexam:
Completephysicalexamination
ResultsofPhysicalExamination:
Generalappearance:Wellbuiltmale,illlooking,notindistress.HEENT:IctericsclerapresentNoJVD.Lungs
arecleartoauscultationandpercussionbilaterallycardiovascular:S1S2normal,nomurmurs,ruborgallop.
Abdomenissofttendernessispresentintherightupperquadrant,butthereisnorigidity,reboundorguarding
normalbowelsoundsliverisenlargedabout2cmbelowtherightcostalmargin,tendertopalpation,firmin
consistencywithasmoothedgeandsurfacenosplenomegalyorfreefluid.Rectal:Normalsphinctertone,no
hemorrhoidsorfissures,stoolishemenegative.Extremities:noedema,clubbingorcyanosis,nocalf
tendernessperipheralpulsesarefull.Skin:nopalmarerythema,nospiderangioma.CNS:normal,no
asterixis.Restoftheexaminationiswithinnormallimits.
OrderNo.1:
LFTs,stat
CBCwithdifferential,stat
Reticulocytecount,stat
BMP,stat
PT/INR,stat
*Callmewhenthelabresultsavailable
ResultsforOrderNo.1:
LFT:Totalbilirubin6.0mg%,Directbilirubin4.0mg%,ALT980IU/L,AST700IU/L,Alkaline
phosphatase190IU/L,Protein7.4g/dl,albumin3.8g/dl.
PT=13.2sec,
CBC:Hgb15g/dl,WBC9,000/uL,Platelet250,000/mm3,normaldifferentialcount
Peripheralsmear:normalReticulocytecount:normal
BMP:BUN18,Creatinine1.1,Sodium138meq/L,Potassium3.8meq/L,Chloride105meq/L,
Bicarbonate26meq/L.

Discussion:
Theetiologyofjaundicecanbedividedintothreebroadcategorieshemolytic,hepatocellularandobstructive.
Thehemolyticjaundiceischaracterizedbyatriadofanemia,mildjaundice,andsplenomegalybutthe
hyperbilirubinemiaisunconjugated(predominantlyindirectbilirubin).Theperipheralsmearmayshowsome
abnormalcellssuggestiveofhemolysisandreticulocytecountiselevated.Thispatienthasjaundicewith
conjugatedhyperbilirubinemia(predominantlydirectactingbilirubin)narrowingthepossibilitytohepatocellular
andobstructivepathology.Thesignificantelevationofaminotransferasesandonlymildelevationofalkaline
phosphataseinthispatientmakesthepossibilityofobstructivejaundice(e.g.stones,stricturesorcancer)less
likely.Thisimpliesthatthispatientmostlikelyhasahepatocellularcause.Thecausesofacutehepatocellular
jaundicewouldincludeinfections(mainlyviral),drugs(e.g.acetaminophen),toxins(e.g.mushroom),alcohol
andischemic.RememberthatinacutealcoholichepatitistheAST/ALTratiois>2:1,buttransaminasesare
never>300.
Thispatient'srecentvisittoMexico(developingnation),incubationperiodof2weeksafterreturnfromMexico,
onsetwithfeverduringtheanictericphase,feverresolvingwithonsetofjaundiceandaversiontocigarettes
suggestviralhepatitisA.HepatitisAisthemostcommonformofacuteviralhepatitisintheUSAand
worldwide.HedoesnothaveriskfactorsforhepatitisBorC.Remember,thatalthoughfecooralrouteisthe
mostcommonmodeofhepatitisAinfection,homosexualmenandIVdrugusersarealsoatanincreasedrisk.
Itsincubationperiodvariesfrom15to50days.
Confirmingthediagnosis:ThediagnosisofacuteviralhepatitiscanbeconfirmedbyorderingantiHAV
antibodies.TheseareoftwotypesIgMandIgG.Boththeantibodiesmaybepresentintheserumsoonafter
theonsetofillness.ButthepresenceoftheIgMantiHAVantibodyconfirmsthediagnosisofhepatitisA.The
IgMantibodypeaksduringfirstweekanddisappearswithin36months.ThepresenceofIgGantiHAV
antibodyintheabsenceofIgMindicatesapreviousexposure,noninfectivityandimmunityagainstrecurring
hepatitisAinfection.
OrderNo.2:
AntiHAVantibodies(IgMandIgG)
*CouldalsoorderaHepatitisB(HBsAg,IgMantiHBcab),HepatitisC(HepCantibody)screeningpanelif
riskfactorswerepresent.
Restathome
AntiemeticsPRN(Phenergan,oral,continuousbecausethereisnoPRN(asneeded)optioninsoftware)
Reassurepatient
Regulardiet
Noalcohol
Nosmoking
Hepatitisprecautionscounseling
Noacetaminophenorhepatotoxicdrugs(theseare2notavailableinsoftware)
Maysendthepatienthome,repeatappointmentoncetheresultsavailable
ResultsforOrderNo.2:
Patientcomesforreturnvisitthenextday
IgMantiHAVantibodypositive
IgGantiHAVantibodypositive
Order:
Interimhistoryandbrieffocusedphysicalexam
Results:
Patientfeelsweak,continuestohavepoorappetitevitalsstable
Patientquestionsaboutprophylaxisforhiswifeanddaughter(Maynothappeninrealexam)
OrderNo.3:
Maysendthepatienthomeagainandscheduleappointmentfor3days

LFTsin3days
PTin3days
Notifypublichealthdepartment
*HepatitisAimmuneglobulinandHepatitisAVaccineforwifeanddaughter(Maynothappeninrealexam)
ResultsforOrderNo.3:
Patientcomesforareturnvisit
LFTTotalbilirubin8.0mg%,Directbilirubin5.0mg%,ALT1500IU/L,AST1300IU/L,Alkaline
phosphatase210IU/L
PT/INR14.0sec,INR=1.36
*Patientstillfeelsweak,continuestohavepoorappetitebutvitalsstable
OrderNo.4:
Maysendthepatienthome
LFTin3days
PT/INRin3days
Repeatappointmentwithlabresults
ResultsforOrderNo.4:
Patientcomesforareturnvisit
LFTTotalbilirubin5.0mg%,Directbilirubin3.0mg%,ALT800IU/L,AST700IU/L,Alkaline
phosphatase190IU/L
PT14.0sec,INR=1.36
*Patientfeelsbetter,nauseaislessandappetiteimprovedvitalsstable
Ifthecasestillcontinues,order:
Examinethepatient3dayslater
Order,repeatLFTandPT/INRin3days
FinalDiagnosis:
AcuteHepatitisA
Discussion:
HepatitisAcausesaselflimitingacutehepatitis.TherearenochronicorcarrierformsofhepatitisA.Giventhe
generallybenignnatureofhepatitisA,mostpatientscanbetreatedathomewithsymptomaticandsupportive
therapies.Nospecificantiviraltreatmentisavailable.Intakeofalcohol,acetaminophenandotherpotentially
hepatotoxicsubstancesshouldbeavoided.Rememberthatconjugatedhyperbilirubinemiaisseeninviral
hepatitisanddonotbefooledbylightcoloredstools.Theseareacholicstoolsbecauseofcholestaticphaseseen
ininfectioushepatitiscausingapicturesimilartoobstructivejaundice.Donotbescaredbyhighandrising
levelsofaminotransferases.Theaminotransferasesmaybeashighas5000IU/Landmayshowarisingtrend
forcoupleofweeksbeforestartingtoresolve.Recoveryoccursin316weeks,althoughLFTmaybeimpaired
tilloneyear.Encephalopathyandcoagulopathypointtowardshepaticfailureandtheneedforadmission.
Doesthispatientneedvaccination?
No,sinceHepatitisAinfectionleadstolifelongimmunity.

Location:
Emergencydepartment
Vitalsigns:
T37.0C(98.6F)BP100/60mmHg(supine),80/50mmHg(sitting)HR124/minR24/min
CC:
Blackcoloredstools
HPI:
A55yearoldmancomestotheEDwith6episodesofblackstoolsoverthelast24hours.Hedecidedtoseek
medicalhelpafterhevomitedbrightredbloodaboutanhouragoandfeltweakandlightheaded.Thepatient
hasahistoryofepigastricpainforthelastmonththatoccursmostlyonanemptystomachandisrelievedwith
foodandantacids.Healsohaschroniclowbackpainforwhichhetakesoverthecounteribuprofenonaregular
basis.Hehasnohistoryofheartburn,constipation,orweightlossandhasneverhadacolonoscopy.Thepatient
hassmokedapackofcigarettesdailyforthelast30years.Hedrinksbeersociallyandonweekends.Family
historyisunremarkable.
Approachtothiscase:
Thispatientwithmelenaandhematemesisishemodynamicallyunstableasisobviousfromthehypotension,
orthostasis,andtachycardia.Initialmanagementisadequateresuscitation.
Orders:
IVaccess,stat2largeboreIVlines(16gauge)
StartIVfluids:Normalsaline,continuous
MakeNPO
Cardiacmonitoring,continuous
BPmonitoring,continuous
Pulseoximetry,stat
Results:
BP100/70mmHgHR116/min
Oxygensaturation95%onroomair
Orderfocusedphysicalexamination:
Generalappearance
HEENT/neck
Lungs
Heart
Abdomen
Rectum
Extremities
Resultsofphysicalexamination:
Generalappearance:55yearoldmanwhoappearshisstatedage,ispale,andismildlyanxious
HEENT:Paleconjunctiva,anictericsclera,drymucousmembranes
Lungs:Cleartoauscultationbilaterally
Cardiovascular:TachycardiaS1&S2normalnomurmurs,rubs,orgallopsnojugularvenousdistension
Abdomen:Softmildtendernessintheepigastricareabutnorigidity,rebound,orguardingbowelsoundsare
normalnoorganomegalyorfluidwave
Rectal:Normalsphinctertonenohemorrhoidsorfissuresstoolisblackandhemepositive
Extremities:Noedema,clubbing,orcyanosisnocalftendernessperipheralpulsesarefeeble
Orders:

CBCwithdifferential,stat
BMP,stat
LFTs,stat
PT/INR,stat
aPTT,stat
ECG,12lead,stat
IVpantoprazole,continuous
Typeandcrossmatch,stat
Discontinueibuprofen
Gastroenterologyconsult,stat(reason:GIbleed)
Results:
Hemoglobin
Platelets
Leukocytes

6.9g/dL
250,000/mm3
11,000/mm3

Sodium
Potassium
Chloride
Bicarbonate
Bloodureanitrogen
Creatinine
Calcium

138mEq/L
3.8mEq/L
103mEq/L
26mEq/L
32mg/dL
1.1mg/dL
10.1mg/dL

Totalbilirubin
Directbilirubin
Alkalinephosphatase
Aspartateaminotransferase(SGOT)
Alanineaminotransferase(SGPT)

1.0mg/dL
0.3mg/dL
100U/L
28U/L
30U/L

Prothrombintime
InternationalNormalizedRatio
Activatedpartialthromboplastintime

18sec
1.6
23sec

ECGshowssinustachycardiawithoutevidenceofischemiaorinfarction.
AdmittoICU.
Orders:
ContinueNPO
Bedrest
Urineoutput
Pneumaticcompressionstockings
StopIVNS
StartpackedRBCtransfusion(typePRBC),stat2units
FFP,stat1unit
Hemoglobinandhematocrit(H&H)every6hours
PT,INRafterFFPtransfusion
Continuepantoprazoleinfusion
Callwhenthelabresultsareavailable.
Results:
BP110/70mmHgHR100/min

After2unitsofPRBCand1unitFFP:Hemoglobin8.7g/dLPT14.5secINR1.4
Patientfeelsbetter.Endoscopyshowscleanbasedulcerinthefirstpartofduodenum.Biopsyistaken.
Thenextresultsare:
BP120/80mmHgHR90/min
Hemoglobin8.8g/dL
Orders:
DiscontinueNPO
StopIVNS
Startcleardietandadvancetoregulardietastolerated
H&Hevery12hours
StopIVpantoprazole
Startoralpantoprazole
Results:
Patientistoleratingregulardiet
BP128/80mmHgHR74/min
Hemoglobin8.9g/dL
Biopsyispositiveforinflammation,ulceration,nomalignantcells
TissueisnegativeforHelicobacterpylori
Orders:
Dischargepatienthomeafterovernightwatch
Sendhomeonpantoprazolefor48weeksmakefollowupappointmentin2weeks
Patientcounseling
Medicationcompliance
Recheckhemoglobinatreturnvisit
Startferroussulfate,continuous(optional)
AvoidNSAIDs
Smokingandalcoholcessation
Primarydiagnosis:
Uppergastrointestinalbleedingduetoduodenalulcer(fromNSAIDuse)
Discussion:
Hematemesisandmelenasuggestuppergastrointestinal(UGI)bleeding,whichbydefinitionisbleeding
proximaltotheligamentofTreitz.
MajorcausesofUGIbleedinclude:
Pepticulcer
Esophagogastricvarices
Arteriovenousmalformations
Tumors
MalloryWeisstears(suspectinindividualswithsevereretchingandvomiting)
Carefulhistoryandphysicalexaminationarerequiredtoguidesubsequentmanagementandavoid
complications.HistoryofNSAIDoralcoholuse,priorGIbleed,liverdisease,orcoagulopathycanhelpguide
thediagnosis.Physicalexamfindingssuchastachycardiaandorthostaticbloodpressurechangessuggest
moderatetoseverebloodloss,whereashypotensionsuggestslifethreateningbloodloss.Inadditiontoanemia,
patientswithupperGIbleedoftenhaveelevatedBUN:creatinineratios(>20:1).
ManagementofUGIbleedinvolvesresuscitationandterminationofbleeding.Twolargeperipheralcatheters

andsupplementaloxygen(ifneeded)mustbestartedimmediately.Isotoniccrystalloidsolution(ie,normal
saline)ispreferredforinitialtreatmentofhypovolemichypotension.Recentguidelinessuggestrestrictive
strategyforbloodtransfusiontoavoidcomplications(ie,transfusefor):
Hemoglobin<7inlowriskpatients
Hemoglobin<9inhighriskpatients(eg,coronaryarterydisease)
Hemodynamicinstabilitydespiteappropriatecrystalloidresuscitation
Patientsshouldnotbegivenanythingbymouth,andacidsuppressiontherapywithIVprotonpumpinhibitor
(eg,pantoprazole,esomeprazole)mustbestarted.Patientswithknownorsuspectedcirrhosisandvariceal
bleedingmustalsobestartedonsomatostatinanalogues(eg,octreotide)andappropriateantibiotics(eg,
ceftriaxone).
Nasogastrictube(NGT)placementinUGIbleediscontroversial.CurrentguidelinessuggestthatNGTlavage
beconsideredifbloodclotsneedtoberemovedtofacilitateendoscopy.Gastroenterologicalconsultationmust
beobtainedonallpatientswithGIbleeding.Surgicaland/orinterventionalradiologyconsultationisalso
recommendedifthereisalikelihoodofunsuccessfulendoscopictherapy,rebleeding,orothercomplications.
StablepatientswithoutseverebleedingorhighriskstigmatamaybedischargedonoralPPIafterendoscopy,
withappropriateclinicalfollowupinthenearfuture.Patientswithactivebleeding,avisiblevessel,oran
adherentclotshouldcontinuetoreceiveIVPPI72hoursfollowingendoscopy.Iftherearenosignsof
rebleeding,theycanbeswitchedtoanoralPPIanddischarged.Allpatientswithpepticulcersshouldbetested
forinfectionwithHelicobacterpylori(especiallyifNSAIDuseisexcluded)andtreated.

Location:
Emergencydepartment(ED)
Vitals:
T:98.4F(36.9C)BP:104/70mmHg(supine),80/50mmHg(sitting)HR:120/minRR:24/min
Chiefcomplaint:
Brightredbloodperrectum
Historyofpresentillness:
A65yearoldwomancomestotheEDdueto3episodesofpassingamoderateamountofbrightredbloodwith
stoolsoverthepastday.Shehasbeenfeelingweakandlightheadedbutreportsnonausea,vomiting,or
abdominal/analpain.Thepatienthasnohematemesis,melena,diarrhea,constipation,jaundice,orweightloss.
Normally,herstoolsaresoftinconsistency,andthereisnohistoryofanalfissuresorhemorrhoids.Herpast
medicalhistoryissignificantfortype2diabetesmellitus,hypertension,andhyperlipidemia.Shehasneverhad
acolonoscopy.Hermedicationsincludeglyburide,simvastatin,andlisinopril.Thepatientdoesnotuse
tobacco,alcohol,orillicitdrugs.Hermotherdiedofcoloncanceratage60.Therestofthereviewofsystems
isunremarkable.
Approach:
Thispatientwithhematocheziaishemodynamicallyunstable,whichisobviousfromherhypotension,
orthostasis,andtachycardia.Initialmanagementisaimedatresuscitation.
Order:
IVaccess,stat:2largeboreIVlines(16gaugeorlarger)
StartIVfluids:Normalsaline,bolus2L,thencontinuous
NPOstatus
Cardiacmonitor,continuous
ContinuousBPmonitoring
Pulseoximetry,stat
Results:
BP:100/65mmHgHR:114/min
Oxygensaturation:97%onroomair
Orderphysicalexamination:
Generalappearance,HEENT/neck,heart,lungs,abdomen,rectumwithfecaloccultbloodtest(FOBT,whichis
notrequiredifyouseefreshbleeding),andextremities
Resultsofphysicalexamination:
Generalappearance:Palelooking,anxiouswomanwhoappearsherstatedage.
HEENT:Paleconjunctiva,anictericsclera,anddrymucousmembranes.Nopalpablelymphnodes.
Cardiovascular:Tachycardic,S1andS2normal,nomurmurs,ruborgallop,nojugularveindistension.
Lungs:Cleartoauscultationandpercussionbilaterally.
Abdomen:Softandnontenderwithnorigidity,rebound,orguardingbowelsoundsarenormal,no
organomegalyorfreefluid.
Rectal:Normalsphinctertone,nohemorrhoidsorfissures,freshbloodinrectum.
Extremities:noedema,clubbingorcyanosis,nocalftenderness.
Restoftheexaminationisunremarkable.
Order:
CBC,stat
BMP,stat
LFTs,stat

PT/aPTT,stat
ECG,12lead,stat
Nasogastrictubeplacementandlavage
Typeandcrossmatch,stat
Results:
Hemoglobin(Hb)

6.8g/dL

Platelets

450,000/mm3

Leukocytes

12,000/mm3

Sodium

135mEq/L

Potassium

3.7mEq/L

Chloride

104mEq/L

Bicarbonate

25mEq/L

Bloodureanitrogen

15mg/dL

Creatinine

1.0mg/dL

Glucose

90mg/dL

Totalbilirubin

1.0mg/dL

Directbilirubin

0.4mg/dL

Alkalinephosphatase

110U/L

Aspartateaminotransferase(SGOT)

30U/L

Alanineaminotransferase(SGPT)

31U/L

Prothrombintime

17sec

InternationalNormalizedRatio(INR)

1.0

ECGshowssinustachycardiawithoutevidenceofischemiaorinfarction.
NGTlavageshowsbiliousfluidandnoblood.
Order:
AdmitinICU
ContinueNPO
Discontinuepatient'sglyburide,simvastatin,andlisinopril
StopIVNS
StartpackedRBCtransfusion2units
Hemoglobin(orhematocrit)every6hours
Bedrestwithbathroomprivileges
Urineoutput
Applypneumaticcompressionsfordeepveinthrombosis(DVT)prophylaxis
AccuChekevery6hours(useregularinsulinasneeded,basedonbloodsugarlevels)
Gastroenterologyconsult,statforcolonoscopy(reason:65yearoldpatientwithhematocheziaandlow
hemoglobin,nopriorcolonoscopypleaseevaluateforsourceofbleeding).
Examinethepatient4hourslater:orderinterimhistoryandfocusedphysicalexamination(makesureyoulisten
tothelungsaspatientsmaydevelopfluidoverloadduetoseveralIVinfusions).
Results:
BP:110/70mmHgHR:100/min
After2unitsofPRBC,Hb:8.5g/dL
Patientfeelsbetterexaminationisunremarkablebleedinghasstopped
Order:
ContinueNPO
RestartIVNS,continuous
MeasureHbevery6hours
Startbowelpreparationforcolonoscopy:4Lofpolyethyleneglycoltakenuntilstoolsareclear
Checkvitalsevery2hours
Callphysicianwhenlabresultsbecomeavailable
Results:
Colonoscopy:Multiplediverticulainsigmoidanddescendingcolontwosmall4mmpolypsareseeninthe
sigmoidcolonandbiopsiesaretaken.
Hb:8.7g/dL
BP:120/80mmHgHR:90/min
Order:
DiscontinueNPO
StopIVNS
Startclearliquiddietandadvancetohighfiberdietastolerated
MeasureHbevery12hours
Results:
Patientistoleratinghighfiberdiet
Hb:9.0g/dL
BP:128/80mmHgHR:74/min
Biopsyshowshyperplasticpolypsthatarenegativefordysplasia.

Order:
Dischargethepatienthomeafterovernightwatch
Highfiberdiet
Restartherhomemedications
D/CDVTprophylaxis
Followupappointmentin1weekwithrepeathemoglobin.
Discussion:
Lowergastrointestinalbleeding(LGIB)wasdefinedtraditionallybybleedingdistaltotheligamentofTreitz.
However,thereisnowadistinctionbetweenbleedingfromthesmallbowel(midGIbleeding)andcolon
(LGIB).MidGIbleedingusuallyrequiresprocedurestoevaluateforobscureGIbleeding(ie,afterupper
endoscopyandcolonoscopyfailtoidentifythebleedingsource).
MostcasesofbrightredbloodperrectumorhematocheziaareduetoLGIB,butabout10%resultfrombrisk
(massive)uppergastrointestinalbleeding(UGIB)(especiallyinpatientswithhemodynamicinstability,
orthostatichypotension,andanelevatedBUN/Crratio>20:1).Insuchpatients,itmaybehelpfultoperform
nasogastrictube(NGT)lavagetodifferentiatebetweenthetwo.Coffeegroundmaterialorbrightredbloodin
thelavagefluidsuggestsanUGIsource,whereasthepresenceofbiliousnonbloodyfluidusuallysuggeststhat
thereisnoactiveUGIB.
Thefollowingarethe4causesofacuteLGIB:
1.Anatomic(diverticulosis)
2.Vascular(angiodysplasia,ischemia)
3.Inflammatory/infectious
4.Tumors
Historyandphysicalexaminationalongwithevaluationandresuscitationmeasuresmustoccursimultaneously.
Pertinenthistorysuchaspriorbleed,useofantiplatelet/anticoagulantagents,painlessbleeding(suchasin
diverticulosis),abdominalpain,andchangeinbowelhabits(malignancy)areimportant.
LaboratoryevaluationincludesCBC,chemistrypanel,livertests,andcoagulationstudies.Althoughtheinitial
hemoglobinlevelmaybenormal,itshouldbemonitoredevery48hours,dependingontheseverityofthe
bleed.UnlikepatientswithUGIB,thosewithLGIBhaveanormalBUN/Crratio.
Hemodynamicstabilizationmustoccurpriortoanyproceduretoevaluatethecauseofbleeding.Allpatients
withGIbleedingshouldhave2largebore(16gaugeorlarger)peripheralIVlines.Anisotoniccrystalloid
solution(ie,normalsaline)ispreferredforinitialtreatmentofhypovolemichypotension.Recentguidelines
suggestarestrictivestrategyforbloodtransfusiontoavoidcomplications.
Transfusionsshouldoccurinthefollowingsettings:
Hemoglobin<7mg/dLinlowriskpatients
Hemoglobin<9mg/dLinhighriskpatients(eg,coronaryarterydisease)
Hemodynamicinstabilitydespiteappropriatecrystalloidresuscitation
AnearlygastroenterologyconsultationisrecommendedforallpatientswithacuteLGIBthesurgicaland/or
interventionalradiologydepartmentsshouldalsobeconsultedforcasesofmassivebleeding.Patientswith
shockshouldbeadmittedtotheICU,butmostotherscanbeadmittedtothegeneralmedicalward.Allpatients
admittedtotheregularwardshouldreceiveECGmonitoring.
OnceanUGIsourceofbleedingisexcluded,colonoscopyistheprocedureofchoicefordiagnosisandtreatment
ofmostacuteLGIBcases.Otheroptionsareradionuclideimagingandangiographyhowever,theyrequirethe
presenceofactivebleedingatthetimeofexamination.Colonoscopyistypicallyperformedwhenthepatientis
hemodynamicallystableandhashadadequatebowelpreparation.

Ifcolonoscopyisnegative,taggedredbloodcellscan(radionuclideimagingstudy)and/orangiographycanbe
performednext.
Finaldiagnosis:
Lowergastrointestinalhemorrhage,secondarytodiverticulosis.

Location:EmergencyRoom
Vitals:BP:80/50mmHgHR:40/minRR:24/minTemp:98.4F
C.C:Lightheadedness
HPI:
A55yearsoldmalevictimofamotorvehicleaccidentisbroughttotheERbyambulance.Hewasa
unrestraineddriverofacarthathitatreeduetopoorvisibilityonthatfoggynight.Thepatientcomplainsof
mildgeneralizedbodyache,severechestpainandlightheadedness.Herememberedhischesthavingstruck
againstthesteeringwheel.However,therewasnohistoryofheadinjury,headacheorlossofconsciousness.He
didnotcomplainofrespiratorydistress.ThepatientwasfeelinguncomfortablewiththeMiamiJcollarputby
theEMSteamaroundhisneckatthesiteoftheaccident.Hehasnoallergiesanddeniedbeingonany
medication.Restofthereviewofsystemsisunremarkable.
Howwouldyouapproachthispatient?
Thisisavictimofmotorvehicleaccident,whoishemodynamicallyunstableasisobviousfromthehypotension
andbradycardia.Theinitialapproachshouldbetotakethegeneralresuscitativemeasures,adelayinwhich
mightbelifethreatening.Simultaneously,thinkofreasonsforhypotensionandbradycardiainanaccident
victimandordertherelevanttests.Rememberyoualwaysneedathoroughphysicalexaminationtoruleout
seriousinjuriesanddecidewhichbodypartstoimage.
OrderNo.1:
IVaccess,stat2large(18G)IVboreneedles
StartIVfluids:NormalSaline,bolus
ContinuousBP,HRmonitoring
Pulseoximetry,stat
ResultsfororderNo1:
BP80/50mmHgHR34/min
OxygenSaturationis95%onroomair
Orderexamination:
General
Heart
Lungs
Resultsoftheexam:
Generalappearance:Wellbuilt,whitemale,inseverepain,holdingontohischestwithhisrighthand.Lungs
arecleartoauscultationandpercussionbilaterallyCardiovascularBradycardia,variableintensityofS1and
S2nomurmurs,ruborgallop.
OrderNo2:
EKG,12lead,stat
ChestXray,PAportable
Xraycervicalspine,stat
IVFentanylorKetorolac,bolus
ResultsofOrderNo2:
EKGshowscompleteheartblock,ventricularescaperhythmwitharateof40/min,QRSdurationof140msec.
NoevidenceofischemiaorinjuryexceptnonspecificST/Tchanges.
ChestXray:Fractureoftheleft3rdand4thribs.Nopneumothoraxoreffusion.Heartandmediastinumare
normalinsizeandconfiguration.
Xraycervicalspine:Normal

OrderNo3:
Atropine0.5mgIVstat
Putpatientontranscutaneouspacemaker
ConsultCardiology,stat(fortransvenouspacemakerplacement)
ConsultOrthopedics,stat(toruleoutcervicalspineinjuryandgetridofMiamiJcollar)
MakeNPO
CBCwithdifferential,stat
BMP,stat
PT/aPTT,stat
ResultsofOrderNo3:
CBC:Hgb13.0g/dl,Hct39%WBC9,200/uL,Platelet250,000/mm3,normaldifferentialcount
BMP:BUN19,Creatinine1.1,Sodium138meq/L,Potassium3.8meq/L,Chloride103meq/L,and
bicarbonate26meq/L.
PT=13sec,INR=1.23APTT=33seccontrol=35sec
OrderNo4:
ChecktheBPandHR
ResultofOrderNo4:
Transcutaneouspacemakerpacesatrateof80/min,BP90/60
Patient'slightheadednessandchestpainisbetter
Orderexaminationof:
HEENT/Neck
Abdomen
Extremities
Skin
CNS
ResultsofPhysicalExamination:
HEENT:Normocephalic,atraumatic,PERLA,EOMI,pinkconjunctiva,anictericsclera,moistmucous
membranes,noearornosebleedNeckMiamiJcollaronAbdomenissoft,notenderness,rigidity,reboundor
guardingbowelsoundsarenormal,noorganomegalyorfreefluid.Extremitiesnoedema,clubbingor
cyanosis,nocalftenderness,peripheralpulsesfeeble.Neurologicalexamawake,alertoriented,movesallfour
limbswithnofocalneurologicaldeficits.
OrderNo.5:
ContinuousHRandBPmonitoring
ContinueNPO
ContinueNS
CKandMB,stat
TroponinT,stat
Echocardiogram,stat
ResultsforOrderNo.5:
CK500MB11
TroponinT0.500
Echocardiogram:EF=5560,nowallmotionabnormalities,allvalvesarenormal,nopericardialeffusion
Cardiologisttakesthepatienttothecardiaccathlabforatemporarytransvenouspacemakerinsertion.
Ifcasecontinuesfurther,mayneedpermanentpacemakerinsertion.
Discussion:
Themostimportantcauseofhypotensioninatraumavictimishemorrhage.Thefirststepinmanagement
wouldbetostartIVfluidsandsendaCBCtolookfortheamountofbloodloss.Ifthereisnoovertbleeding

onemustlookforanoccultcollectioninthechestandabdomen,forwhichyouneedtodoimagingstudies.
Normally,patientsdeveloptachycardiainresponsetohypotensionsecondarytohypovolemia.Thebradycardia
accompanyingthehypotensionandthenormalhemoglobininthispatientshouldmakeyoususpiciousofan
etiologyotherthanbleeding.
TheEKGconfirmsthediagnosisofcompleteheartblock(CHB).CHBisathirddegreeAVblockthediagnosis
ofwhichismadebyAVdissociationwithaslowventricularescaperhythmofaround40beats/min.Theatria
maybeinsinusrhythmorinfibrillationbutthe'P'wavesdonotbearanyrelationshipwiththeQRScomplexes.
However,itisalsoimportanttoestablishtheetiologyofCHBsinceitaidsinthefurthermanagement.Themost
importantcausesarefibrosisordegenerationoftheconductionsystemandischemicheartdisease.Theothers
includedrugs(betablockers,calciumchannelblockers,digitalis,amiodarone),metabolicabnormalities
(hyperkalemia),valvularheartdisease,andcardiomyopathy(amyloid,sarcoid,hypertrophiccardiomyopathy).
Remember,traumaisanuncommoncauseofCHB.AbsenceofSTTchangessuggestiveofischemiainEKG
andnowallmotionabnormalitiesexcludedthepossibilityofacutecoronarysyndrome.TheelevatedCK,MB
andTroponinTwereprobablysecondarytomyocardialcontusion.Thepatientswasnotonanyheartrate
loweringdrugs,hiselectrolyteswerenormalandEchofurtherruledoutanyvalvularabnormalities,
cardiomyopathyorpericardialeffusion.
Theonlymodalityoftreatmentforcompleteheartblockispacing.Atropineisonlyoflittlebenefitandmay
sometimestransientlyimprovetheheartrateandthebloodpressure.Thesedaysthelifepacksareequipped
withpadsfortranscutaneospacing.Buttheseshouldbeusedonlyasabridgeforthetransvenouspacing.The
transvenouspacingmaybeatemporarypacingtobeginwith.Inthispatient,iftheCHBpersistsforthenext
coupleofdays,apermanentpacemakercanbeplaced.
Patientswithseconddegreeatrioventricularblockswhoareasymptomaticandhemodynamicallystablemaybe
managedwithoutapacemaker.However,acompleteheartblockevenintheabsenceofsymptomswarrantsa
pacemaker,sinceyouarenotsurewhenthepatientmaybecomeunstable.
Anotherimportantthingistoavoidmedicationsthatwouldcausebradycardiaandhypotension.Thispatienthas
ribfractureandalotofchestpain.Useofmorphinemayworsenhishemodynamicparameters.So,ketorolacor
fentanylwouldbebetteroptionsforpaincontrolinthesepatients.
FinalDiagnosis:
Motorvehicleaccidentwithcompleteheartblock(secondarytomyocardialcontusion)

Location:EmergencyRoom
Vitals:BP:100/60mmHgHR:104/minRR:30/minTemp:100.4F
C.C:Generalizedbodyacheandweakness
HPI:
A80yearsoldwhitemaleisbroughttotheERbyhisson.Hissonfoundhimlyinginthewoodsonahotsunny
day.Itseemedthatthepatienthadgoneforastrolllasteveningandfelldown.Hewasunabletogetup,
shoutedforhelpbutcouldnotgetany.Hehadbeenlyingonthegroundforthelast24hourstillhissonfound
him.Thepatientcomplainedofseverebodyache.Hefeltveryweakandwasthirsty.Hedeniedhavinglost
consciousness.Hedidnotpassurineforthepast24hours.Therewasnohistoryofheadinjuryorseizures.He
hasnoallergiesandisnottakinganymedications.Thepatientdoesnotsmokeanddeniesanyalcoholuse.
Familyhistoryisnoncontributory.Restofthereviewofsystemsisunremarkable.
Howwouldyouapproachthispatient?
Thisisan80yearsoldmanwhohadafallandhadbeenlyingonthegroundformorethan24hoursonahot
sunnydaywithnohelp.Heishemodynamicallystable.Thegeneralizedbodyacheisahinttowardspossible
muscleinjuryandshouldbeaguidefororderingfurtherdiagnostictests.Rememberyoualwaysneeda
thoroughphysicalexaminationtoruleoutseriousinjuriesanddecidewhichbodypartstoimage.
OrderNo.1:
IVaccess,stat
Pulseoximetry,stat
ResultsfororderNo1:
OxygenSaturationis95%onroomair
Orderexamination:
Complete
Resultsoftheexam:
Generalappearance:Wellbuilt,indirtladenclothes,appearsextremelydryandweak.HEENTnormalNeck
noJVDRespiratoryCleartoauscultationbilaterallyCardiovascularTachycardia,S1S2normal,nomurmur,
ruborgallopAbdomensoft,nondistended,nontender,normalbowelsounds,noorganomegalyExtremities
noedema,clubbingorcyanosis,nocalftenderness,peripheralpulsesfeebleNeurologicalawake,alert,
oriented,nofocalneurologicaldeficit
OrderNo2:
StartIVfluids:Normalsaline,bolus
InsertFoley'scatheter,stat
CBCwithdifferential,stat
BMP,stat
EKG,12lead,stat
Urinanalysis
ResultsfororderNo2:
Thenursereportsthatthepatientcouldgiveheronly5ccofdarkbrownurine
CBC:Hgb13.0g/dl,Hct39%WBC13,200/uL,Platelet250,000/mm3,normaldifferentialcount
BMP:BUN45mg%,Creatinine2.6mg%,Sodium134meq/L,Potassium5.5meq/L,Chloride92meq/L,
andbicarbonate17meq/L.Calcium8.0mg%
EKGshowssinustachycardia
UrinedipstickpositiveforbloodUrinemicroscopicnoRBC,noWBC,reddishgoldpigmentedcasts

OrderNo3:
CPK,stat
Ionizedcalcium,stat
Serummagnesium,stat
Serumphosphorus,stat
Serumuricacid,stat
Urinemyoglobin,stat
PT/INR,stat
APTT,stat
Admitinfloor
VitalsQ2hours
Urineoutput,hourly
Activityastolerated
IVNS,continuous
ResultsofOrderNo3:
CPK10,500IU/L
10ccurineinUrobag
Ionizedcalcium0.99mmol/L
Serummagnesium1.8meq/L
Serumphosphorus5.5mg/dl
Serumuricacid8.5mg/dl
Urinemyoglobinpositive
PT14.2sec,INR1.40APTT35sec
OrderNo4:
Informin4hours
ResultofOrderNo4:
BP110/80mmHg,HR104/min
Urineoutput75ml/hr
OrderNo.5:
Stop0.9%Saline
Start0.45%Saline(withmannitolandSodabicarbonateaddedtoit)
MonitorurinepHevery1hour
TitratethemannitolbicarbonatedripforurinepH>6.5andUrineoutputof>300mL/hr
CheckCPKin4Hours
CheckBMPin4Hours
CheckMagnesiumandphosphorusin4Hours
ResultofOrderNo5:
CPK9000IU/L
BMP:BUN38mg%,Creatinine2.1mg%,Sodium138meq/L,Potassium5.0meq/L,Chloride101meq/L,
andbicarbonate21meq/L.Calcium8.2mg%
SerumMagnesium1.4meq/L
SerumPhosphorus5.0mg/dl
BP130/80mmHgHR96/min
UrinepH7.2
Urineoutput1300ccinlast4hours
Nursesaysthatthepatientisfeelingbetter
OrderNo.6:
Stopmannitolbicabonatediuresis

Start0.45%saline,continuous
CheckBMP,everysixhours
Checkserummagnesiumevery6hours
Checkserumphosphorusevery6hours
CheckCPK,every12hours
Discussion
Thisisacaseofrhabdomyolysis.Prolongedimmobilizationandcompressionofmusclesleadtoischemic
muscledamage.Thehotclimateanddehydrationcontributedtothemyoglobininducedacutetubularnecrosis.
Thisresultedinacuterenalfailurewithaniongapmetabolicacidosisandtheelectrolyteabnormalitiesseenwith
rhabdomyolysis.
Rhabdomyolysisisasyndromeresultingfromskeletalmuscleinjurywithreleaseofmyoglobinandcreatine
phosphokinase(CPK)intotheplasma.Themyoglobinuria,acidurinepHandrenalhypoperfusionresulting
fromhypovolemialeadstoprecipitationofhemeproteinsandacutetubularnecrosis.
Etiology:
1.Traumaticcauses:Crushsyndrome,burns,electrocution,
2.Nontraumaticcauses:
Musclehyperactivitystrenuousphysicalexercise,seizures,deliriumtremens
Musclecompressionprolongedimmobilization,coma
Muscleischemiaacutearterialocclusion
Malignanthyperthermia,neurolepticmalignantsyndrome,hypothermia
InfectionsViralincludingHIV,bacterial,etc.
Drugsalcohol,heroin,cocaine,amphetamines,zidovudine,statins
Metabolicdisordershypocalcaemia,hypokalemia,hypophosphatemia,hypothyroidism,hyperthyroidism,
diabeticketoacidosis
Metabolicmyopathiese.g.Carnitinepalmitoyltransferasedeficiency.Theseshouldbesuspectedin
patientswithrecurrentepisodesofrhabdomyolysisafterexertion.
Otherscarbonmonoxide,snakebite
Rememberthatinflammatorymyopathieslikepolymyositisanddermatomyositisveryrarelygiveriseto
rhabdomyolysisandacuterenalfailure.
Diagnosis:
Themostcommoncomplaintismuscularpain,whichisverynonspecific.Moreover,acomatosepatientwill
notcomplain.Darkbrownurinemaybetheonlyvisiblesign.Suspectrhabdomyolysisinapatientwithrenal
failure,whohasbloodpresentonurinedipstickbutnoRBConmicroscopicexamination.Thisisbecausethe
myoglobinintheurinecausestheurinedipsticktobefalselypositiveforblood.Plasmacreatinine
concentrationrisesmorerapidlywithrhabdomyolysis(upto2.5mg/dLperday)thanwithothercausesofacute
renalfailure.Incontrasttootherformsofacutetubularnecrosis,FENaislessthan1percent.
ThediagnosisofrhabdomyolysisismadebymeasurementofCPK.Itbeginstoraise2to12hrsaftertheinjury
andreachesitspeakvalue1to3daysafterinjury.ThepeakmayrangefromseveralhundredIU/Ltoover
200,000IU/Linafullblowncrushsyndrome.Therefore,CPKshouldbemeasureddailyforatleast3daysto
followextentofmuscledamage.IftheserumCPKremainselevateddespitetreatment,ongoingmuscleinjury,
necrosisand/orcompartmentsyndromeshouldbesought.
Myoglobinisalsoreleasedfromtheinjuredmuscle.ItincreasesbeforeCPKanddecreasesmorerapidlyowing
toitsclearancebykidneysandmetabolismtobilirubin.Therefore,rememberthatanormalserummyoglobin
andabsenceofmyoglobinuriadoesnotexcludethediagnosisofrhabdomyolysis.
Variouselectrolyteabnormalitiesresultfromrhabdomyolysis.Thesecanbebetterunderstoodbygroupingthem
intotwocategories

1.InfluxfromExtracellularcompartmentintomusclecellswater,sodium,chloride(hypovolemicshock),
calcium(hypocalcemia)
2.Effluxfrominjuredmusclecellspotassium(hyperkalemia),purines(hyperuricemia),phosphate
(hyperphosphatemia),lacticacid(metabolicacidosis),myoglobin(myoglobinuria,nephrotoxicity),
thromboplastin(DIC),creatinekinase,creatinine(increasedserumcreatininetourearatio)
Management:
1.Fluidreplacementisthemainstayoftherapy.Usenormalsalineandinitiateat1.5L/hr.Theaimisto
washoffthemyoglobinfromtherenaltubules,establishagoodurineoutputandpreventorlimitacute
tubularnecrosis.Whileononehandmanyelectrolyteabnormalitiescanprecipitaterhabdomyolysis,the
syndromeitselfcanleadtovariousmetabolicderangements.HenceoneneedstomonitortheBMPand
electrolytesverycloselyfortheinitial2days.
2.Forcedalkalinediuresisusingmannitolandbicarbonateisrecommendedbysome.Alkalinizationofurine
preventsprecipitationofmyoglobininthetubules.However,thisshouldbeusedoncetheBPisstable
andaurineoutputisestablishedusingisotonicsaline.
Onehastobecarefulduringsuchlargevolumefluidreplacementasthereisalwaysariskoffluidoverload.
FinalDiagnosis:
Rhabdomyolysisduetoprolongedimmobilization

Location:EmergencyRoom
Vitals:BP:120/80mmHgHR:112/minRR:28/minTemp:37.8C(100F)
C.C:Fatigueandrightupperquadrantabdominalpain
HPI:
A74yearoldwhitemalepresentstotheERwitha3dayshistoryoffatigueandrightupperquadrantabdominal
pain.Hispainisadullincharacter,moderateintensity,poorlylocalizedwithnoradiationtobackorshoulder.
Itincreaseswithdeepinspiration.Hedeniesanyfever,coughorsputumproductionbutcomplainsofprofuse
sweatingoffandon.Hehaspoorappetitewithsomenauseabutnovomiting.Thereisnohistoryofbowelor
bladderproblems.ThepastmedicalhistoryissignificantfortypeIIdiabetesmellitus.Hehasnoallergiesand
istakingglipizideforhisdiabetes.Thepatientdeniesanytobaccooralcoholabuse.Thereisnohistoryofsick
contacts.Heisawidowerandlivesalone.Familyhistoryisnoncontributory.Restofthereviewofsystemsis
unremarkable.
Howwouldyouapproachthispatient?
Thisisa74yearoldpatientwithacuteonsetrightupperquadrantpainandnonspecificconstitutional
symptoms.Firstthinkofadifferentialdiagnosisofrightupperquadrantpain.Thepossibilitiesare:acute
cholecystitis,cholangitis,choledocholithiasis,hepatitis,pyelonephritis,appendicitis,andpneumonia.The
absenceofdysuria,backpainandnormalurinecolormakethepossibilityofhepatobiliaryandrenalpathology
alittlelesslikelybutnotimpossible.Moreover,absenceoffever,coughandsputumpointagainstthediagnosis
ofpneumonia.Insuchasituationoneshouldperformagoodphysicalexaminationtonarrowdownthelistof
differentialdiagnosisandorderrelevanttests.
OrderNo.1
PulseOximetry,stat
ResultsofOrderNo.1
OxygenSaturation89%onroomair
IVaccess,stat
OrderNo.2
Startoxygenbynasalcanula@4L/min
Orderphysicalexam:
Generalappearance
HEENT/Neck
Examinationofheart
Examinationoflungs
Examinationofabdomen
Examinationofextremities
Skin
ResultsofPhysicalExamination:
Generalappearance:Wellbuiltmale,toxiclooking,tachypneic.HEENT:Anictericsclera,NoJVD.Lungs:
cracklesovertherightlungbase,norhonchiorrubCardiovascular:Tachycardic,S1andS2arenormal,no
murmurs,ruborgallop.Abdomenissoft,nontender,norigidity,reboundorguardingnormalbowelsounds
noorganomegalyorfreefluid.Extremities:Noedema,clubbingorcyanosis,nocalftenderness,peripheral
pulsespalpable.Skin:Norash.
OrderNo.3:
XrayChest(CXR),PAandlateral,stat
EKG,12lead,stat

CBCwithdifferential,stat
BMP,stat
LFT,stat
Lipase,stat
ResultsforOrderNo.3:
XrayChestRightlowerlobeinfiltratesuggestiveofrightlowerlobepneumonia,normalcardiacsize,no
pleuraleffusion
Hgb13.5g/dl,WBC16,500/uL,Platelet350,000/mm3,Differentialcount:90%polymorphs,8%
lymphocytes,20%bands
BUN18,Creatinine1.1,Sodium138mEq/L,Potassium3.8mEq/L,Chloride105mEq/L,Bicarbonate26
mEq/L,Calcium10.1mg.
LFTsandLipaseCompletelynormal
EKGSinustachycardia
OrderNo.4:
Admitthepatientonregularfloor
Bloodcultures,stat
SputumGramstain,stat(Optional)
Sputumcultures,stat(Optional)
StartantibioticsafterdrawingbloodculturesLevofloxacin/gatifloxacinorCeftriaxone+azithromycin,IV
continuous
Acetaminophen,continuous(forfeverandpain)
Vitalsevery4hours
PulseOximetry,Q2hours
Bedrestwithbathroomprivileges
PneumaticcompressionforDVTprophylaxis
Diabeticdiet
Diet,oralfluids
Acuchecks,QID(4timesaday)
Continuehisoralglipizide
PneumovaxandInfluenzavaccinationifnotreceivedearlier
Reviewafter12hours
Orderinterimhistoryandfocusedphysicalexam
ResultsforOrderNo.4:
Vitals:BP:120/80mmHgHR:96/minRR:20/minTemp:99.
Oxygensaturation100%on4L/minofoxygenbynasalcanula
OrderNo.5:
Continuesametreatment
CBC/differentialafter24hours
*Callmewiththeresults
ResultsforOrderNo.5:
After24hours,thenursereportsthatpatientfeelsbetter.
Nonauseafeelsstrongerandwantstoeat
Vitals:BP:120/80mmHgHR:80/minRR:16/minTemp:98.
Oxygensaturation95%onroomair
Bloodculturesnogrowthafter24hours
Hgb13.0g/dl,WBC11,500/uL,Platelet350,000/mm3,Differentialcount:82%polymorphs,8%
lymphocytes,and10%bands
Bloodsugarstableondietandoralhypoglycemics

IfcasecontinuesStopIVantibioticsplantosendpatienthomeonoralantibioticsfor710days.Makea
followupinoneweek.
Counseling:
Patientcounseling
Medicationcompliance
Discussion:
Thisisacaseofcommunityacquiredpneumonia(CAP)withanatypicalpresentation.Withanabnormalchest
xray,normalLFTsandabenignabdominalexamination,noabdominalimagingstudiesareneededinthis
patient.
CertainimportantpointstorememberregardingCAP:
1.Pathogens:ThemostcommonpathogensareStreptococcuspneumoniaeandHemophilusinfluenzae.
Staphylococcusaureus,gramnegativebacilliandMoraxellacatarrhalisarelesscommonorganismscausing
CAP.AtypicalagentsincludingLegionella,MycoplasmapneumoniaeandChlamydiapneumoniaealthoughnot
verycommonneedtobeconsideredwhenchoosingabroadspectrumantibioticforempirictreatmentofCAP.
2.ClinicalPresentation:Cough,sputumproduction,dyspnea,feversandsweatsarethetypicalsymptoms.
Howeverfatigue,headaches,nausea,vomiting,diarrheaandabdominalpainaresomeofthenonspecificand
atypicalsymptoms.Elderlypatients(>75years)havefewersymptomsofCAP.
3.Diagnosticstudies:ChestXrayisamustfordiagnosisofCAP.CBC/Diff,basalmetabolicprofile,sputum
cultures,bloodcultures,andpulseoximetry(orABG)arerecommendedbeforestartingantibiotics.Theroleof
routinesputumGramstainandsputumculturesiscontroversial.Theselabsmaysupportthediagnosis,identify
thepathogenandhelpinmakingtreatmentdecisions,regardingtheneedforadmission.Bloodculturesare
positiveinonly11%casesofCAPwithStreptococcuspneumoniaeaccountingfor67%ofthepositivecultures.
IncaseLegionnaire'sdiseaseissuspected(hyponatremia,immunocompromised,noresponsetoBetalactam
antibiotics)thenurineshouldbetestedforLegionellaantigen.
4.Choiceofantibiotics:
Forapatientbeingadmittedinthegeneralmedicalfloor/ward:
a.FluoroquinolonealonelevofloxacinorgatifloxacinDonotuseciprofloxacin
b.2nd/3rdgenerationCephalosporin(e.g.Ceftriaxone)+Macrolide(e.g.Azithromycin)
*Remember,thecephalosporinsarenoteffectiveagainstatypicalslikelegionella,mycoplasmaandChlamydia
hence,itshouldbecombinedwithamacrolide.Levofloxacinalonealsocoversatypicalorganisms.
Foruncomplicatedpneumoniaintheoutpatientsetting:
a.AzithromycinorDoxycyclinealone
Durationofantibioticsdependsuponthepathogenbeingsuspectedandtreated.Ingeneralitvariesfrom710
days.However,itmaybe1014daysforMycoplasmaandChlamydiaand1421daysforLegionella.
5.Decisiontoadmit:Variousguidelinesandscoringsystemshavebeendevelopedtohelpindecidingwhether
toadmitthepatientornot.However,thesearedifficulttorememberoffhand.Thefollowingmajorpointsare
poorprognosticfactorsinpatientswithCAP.Thepresenceofanyofthesemaynecessitateadmission.
1.Agegreaterthan65years
2.Coexistingdisease:Diabetes,renalfailure,heartfailure,chroniclungdisease,chronicalcoholism,
immunosuppression,andneoplasticdisease.
3.Clinicalfindings:HypoxiarequiringoxygenRR>30breaths/min,SystolicBP<90mmHgorDiastolic
BP<60mmHg,Temperature>38.3C,alteredmentalstatus,extrapulmonarysiteofinfection(meningitis,

septicarthritis).
4.Laboratorytests:WBC<4,000/mm3or>30,000/mm3Pao2<60mmHgrenalfailuremultilobar
involvementonchestradiographpleuraleffusionHct<30%.
PrimaryDiagnosis:
Pneumonia

Location:Office.
Vitalsigns:
Pulse:88/min,BP:130/80mmHg,Temperature:100.0F,RR:22/min,Height:72",Weight:64kgs.
HPI:
A32yearoldpreviouslyhealthymalewhoisanimmigrantfromINDIApresentstoyourofficewith6weeks
historyoffatigue,lowgradefeveranddrycough.Recentlyhenoticedincreasingcoughwithyellowishsputum
since1week.Hesayshisappetitiveisdecreased,andhaveunintentionallossofaround10lbsofweightduring
thisperiod.Hedeniesandshortnessofbreathorhemoptysis.Hehasbeensmoking1packperdaycigarettes
since10years,butdeniesalcoholintake.Hehasnoothermedicalproblems.Familyhistoryisnothing
significant.Hehasnoknownallergies.Restofthereviewofsystemsisunremarkable.Heissexuallyactivein
amonogamousrelationshipwithhiswifeandusescondomalways.Hisimmunizationsareuptodate.
Howdoyouapproachthiscase?
Thisisa32yearoldINDIANmalewith6weekshistoryoffatigue,lowgradefever,drycoughandweight
loss.Thepossiblediagnosisinthismaybeduetopulmonarytuberculosis(mostlikely),fungalinfections,HIV,
carcinomaoflung(ageislittleatypical),chronicbronchitis,andbronchiectasis.Chroniccoughmaybedueto
postnasaldrip,asthma,GERD,medications(ACEinhibitors),andsomerarecauses.However,feverandweight
lossmakespostnasaldrip,asthma,GERD,andmedicationsunlikely.
Orderphysicalexamination:
Completeexamination
Findingsonexam:
Generalappearance:Thinbuiltmaleappearscomfortable.HEENT:Normalconjunctiva,anictericsclera,moist
mucousmembranesthereisnoJVDorlymphadenopathy.Lungs:Distanthollowbreathsoundsandposttussive
ralesareheardovertherightupperlobe.Decreasedbreathsoundsanddullnesstopercussionisnotedatthe
baseofrightlung.Cardiovascular,abdomen,andCNSexaminationiscompletelynormal.Extremities:No
edema,clubbingorcyanosis,nocalftendernessperipheralpulsesarefull.
Orderlabs:
CBCwithdiff,routine
BUN,routine
Serumcreatinine,routine
Chestxray,PAandlateralviews,routine
Sputumexaminationforgram'sstain,andAFB
Sputumcultureandsensitivity
ESR,routine
EKG12Lead,routine
Herearetheresults:
CBC:Hb:13gm%,Hct:40%,WBC:12,000/Cmmwithnormaldifferential,Platelets:230,000/cmm.Peripheral
smear:Normal
BUN:10mg/dLserumcreatinine:0.9mg/dL.
Chestxray:Rightapicalcavitywithsmallpleuraleffusionandrighthilaradenopathy.
Sputumexamination:AFBispositiveforrodshapedbacillisuggestiveofMycobacteriumtuberculosis.
Cultureandsensitivityresultsarepending.
EKG:Normalsinusrhythm.
Reviewoforders:
AdmittoWard
LFT,routine
Serumuricacid,routine

Ophthalmologyconsult.Reason:Assessmentofvisualacuityandotherabnormalitiesbeforestarting
ethambutol.
Results:
LFTsarewithinnormallimits.
Serumuricacidisnormal
Ophthalmologyconsultation:Normalocularstudy
Orderreview:
INH,oral,continuous*6months
Pyridoxineoral,continuous*6months
Rifampinoral,continuous*6months,
Pyrazinamideoral,continuous*2months
Ethambutol,oral,continuous*2months
Counseling:
Patientcounseling
Reassurance
Smokingcessationcounseling
Noillegaldruguse
Regulardiet
Medicationcompliance
Notifypublichealthdepartment
RepeatAFBuntil3consecutivesmearsarenegative
Changelocationtohomeandfixappointmentafter15days
Thenorderbriefhistoryandfocusedexamination
Orderreview:
Continuesamemedication.
FixappointmenteachmonthandcheckLFT.
RepeatchestXrayinonemonthtoseepleuraleffusioniscomingdownornot.
Primarydiagnosis:
Pulmonarytuberculosis
Discussion:
TuberculosisisnotuncommoninUSAbecauseoftheimmigrantsfromdiseaseprevalentcountries(INDIA,
China,Africa).Itisalsocommonamongpopulationslikemalnourished,homeless,inthoselivingin
overcrowdedsituationsandinimmunocompromisedpatients.Tuberculosisiscausedbyacidfastbacilli
Mycobacteriumtuberculosis.Themostcommonvariantisthepulmonaryform.InpatientswithimpairedCMI,
activeprimarytuberculosisoccurswithpulmonaryandconstitutionalsymptomslikefatigue,decreasedappetite,
weightloss,fever,nightsweatsandcough.Oncethepulmonaryformoccurs,thediseasespreadstootherparts
ofthebodybyhematogenousspread.AtypicalpresentationoccursespeciallyinelderlyandHIVpositive
patients.
Thediseaseismainlydiagnosedbychestxray,whichoftenshowsapicalorrightmiddlelobeinfiltrates(cavity)
withorwithoutpleuraleffusion,andsputumexaminationforAFBESRwillbeelevatedbutithasno
diagnosticvalue.PatientsusuallyhavepositivePPDtest.PositivePPDisnotdiagnosticinsymptomatic
patientsasitindicatesbothactiveandlatentinfection.Cultureofmycobacteriumispossiblebuttakesseveral
weeksandtreatmentcanbestartedwithouttheresults.
Thestandardtreatmentinvolvesadministrationof4medications(INH,Rifampin,Pyrazinamideand
Ethambutol)for2monthsfollowedby4monthsof2drugs(INHandRifampin).Themaincauseoftreatment
failureisnonadherence,whichcanbeimprovedbypatienteducationanddirectlyobservedtherapy(DOT)bya

healthcareworker.Inpatienttherapyisusuallyrecommendedandshouldbeinanisolatedroomwithgood
ventilationuntil3consecutivesmearsarenegativeforbacilliandclinicalimprovementisnoted.Outpatient
managementisconsiderediftherearenoimmunocompromisedmembersorchildrenunderfouryearsofageat
home.
Alltheantituberculardrugscausehepaticimpairment,inadditionINHcausespyridoxinedeficiencyleadingto
centralandperipheralneuropathy,whichcanbepreventedbyoralpyridoxine.Monitoringofliverfunctiontests
shouldbeindividualized.EventhoughthereisnospecifiedintervalbyCDC,itisrecommendedthatpatients
shouldbeseenbyaphysicianatleastoncemonthlyandmeasureliverfunctiontests.Ethambutolcausesvisual
acuitydisturbancesandopticneuritishencemanyphysiciansobtainophthalmologyconsultationbefore
treatment.Rifampincausesorangediscolorationofbodyfluidspyrazinamidecauseshyperuricemialeadingto
jointpainsbutmostoftenitisasymptomatic.Manyphysiciansrecommendabaselinetestingofuricacidbefore
startingpyrazinamide.Routinefollowupmeasurementofserumuricacidisnotrequiredunlesspatienthasa
historyofgoutorreceivingmedicationsthatalteruricacidmetabolismsuchasloopdiuretics.Alldrugsaresafe
inpregnancyexceptstreptomycinandspecialconsiderationtobetakenwhileusingpyrazinamideinpregnant
patients,asteratogenicityisnotclearlydefined.
TherapyinHIVpositivepatientsissimilartoHIVnegativepatientsexceptforalongerdurationwithDOTand
towatchoutfordruginteractions.Allcasesshouldbereportedtopublichealthauthorities.
Note:AllpatientsshouldhaveabaselineCBCwithdiff,BUN,serumcreatinine,LFTs,chestxrayandserum
uricacid.

Location:Office
Vitalsigns:Pulse:110/minBP:110/60mmHg,Temperature:39.6C(103.4F),RR:20/min,Height:70",Weight:
165Lb.
HPI:A28yearold,previouslyhealthy,Hispanicmalepresentswith3dayhistoryofburningmicturition.He
alsohasdiscomfortandpaininperineum,lowerabdomenandbackalongwithfever,chills,andnausea.He
statesthatheneedstostraintopassurine.Hepassedmoderateamountofurinefewhoursago.Hedenies
nocturiaorincreasedfrequencyeventhoughhehadnothadvomitingshefeelsverynauseousanddidnoteat
formpast24hours.Hehasnootherknownmedicalproblems.Hehasnoknowndrugallergies.Heissexually
activewithhiswifeanddonotusecontraception.Hedeniesmultiplesexualpartners,previousSTDsandillicit
druguse.Hedeniessmokingandalcohol.Familyhistoryisnothingsignificant.
Howdoyouapproachthiscase?
Thisyoungmanhasdysuria,perinealdiscomfortandfever.Thedifferentialstokeepinmindareacute
prostatitis,acutecystitis,acuteurethritis,acuteepididymitisandpyelonephritis.
Orderphysicalexam:
General
Lungs
Heart
Abdomen
Rectal
Genitalia
Extremities
Herearethefindings:
Gen:A28yearoldmalepatientinmoderatepainandappearsill.Abdomenisnondistended,bowelsoundare
activesuprapubictendernessispresentthereisnocostovertebralangletendernessbladderisnotdistended.
Digitalrectalexam(DRE)revealedexquisitelytender,diffuselyenlargedboggyprostate(avoidmessageand
repeatingDRE).Externalgenitaliaappearnormalwithoutanyulcerationsorurethraldischarge.Restofthe
examiswithinnormallimits.
Discussion:
Presenceofperinealdiscomfort,clinicalfindingofexquisitelytender,diffuselyenlargedboggyprostateon
digitalrectalexammakesacuteprostatitisthemostprobablediagnosis.
Orders:
Changethelocationtoward.
IVaccess,stat
CBCwithdiff,stat
BMP,stat
Urinalysis,stat
UrineGramstain,stat
Urinecultureandsensitivity,routine
Bloodcultures,stat
VitalsignsQ6Hrs
Regulardiet
Bedrestwithbathroomprivileges
Herearetheresults:
CBC:Hb:14mg%,WBC:17,000/cmmwith6%bands.
BMP:Na:135meq,K:4.0meq,HCO3:24meq,Cl:100meq,BUN:16mg/dL,S.Creatinine:0.8mg/dl,Blood
sugar:100mg/dL.

Urineanalysis:
Urineappearance
pH
Specificgravity
Bilirubin
Ketones
Glucose
Blood
RBC
Casts
Esterase
Nitrite
Proteins
Bacteria
WBC

Cloudy/yellow
6(Normal4.18)
1.026(Normal1.0031.030)
Negative
Negative
Negative
Negative
Negative
None
Positive
Positive
Trace
Toonumeroustocount
50+

UrineforgramstainingshowsGramnegativebacilli
UrineC/Sresultspending
Bloodculturespending
Order:
Ampicillin,IV,continuous*12daysuntilpatientisafebrile
Gentamycin,IV,continuous*12daysuntilpatientisafebrile
Normalsaline,IV,continuous
Acetaminophen,oral,continuous
Promethazine,IV,onetime
*IfthepainissevereconsiderIVNSAIDssuchasketorolac
Clickcallmeifneededadvancetheclockfor6hours
Obtainbriefhistory,anddofocusedphysicalexamcheckhisvitalsigns.Ifeverythingisokadvancetheclock
for16hoursagainobtainbriefhistory,anddofocusedphysicalexamcheckhisvitalsigns.
Order:
Oncethepatientisafebrile,D/CIVmedications(Doubleclickonthemedicationandchoosedelete)
OrderCBCwithdiffandurinalysisafter24hoursoftreatment
Ciprofloxacin,oral,continuous
D/CIVfluidsandPromethazine
Results:
Bloodculturesafter24hoursshowsnogrowth
Urinecultureisgrowing>100,000coloniesofLactosefermentinggramnegativerods.Sensitivitiesarepending
(Ittakes2days).
Counseling:
Patientcounseling
Regulardiet
Medicationcompliance
Safesexpractice
Contraceptioncounseling
Seatbeltwhiledriving
Dischargethepatient(changelocationtohome)onceafebrileandmakeanappointmenttoseein7days.
Orderthefollowingwithnextappointment:
Briefhistory
Focusedphysicalexamination

Availablelabresults:
UrineC/SrevealsE.coli,sensitivetoampicillin,cefazolin,gentamycin,azithromycinandciprofloxacin.
Order:
Urinalysis,stat
ChangeantibioticdependingonC/Sresultsforaperiodof4weeks
Primarydiagnosis
Acutebacterialprostatitis
Discussion:
Annually2millioncasesofprostatitiscasesaredocumentedinUSA.Prostatitisoccursinyoungandmiddle
agedmen.Thecommonformsofpresentationareacutebacterial,chronicbacterialandchronicabacterial
prostatitis,thecommonbeingacutebacterialprostatitis.
ThemostcommonorganismisE.coli.Pseudomonasmaybethecausativeorganismincaseofdiabetics
chlamydiaandgonorrheaiscommoninhighriskpatientssuchaspatientswithmultiplesexualpartnerswho
don'tpracticesafesex.Theorganismsascenduptoprostatethroughurethraprecipitatedbytrauma,dehydration
andbladdercatheterization.Thecommonpresentingsymptomsarefaver,malaise,irritatingvoidingsymptoms,
suprapubicandperinealpainordiscomfort.Theremaybeobstructivesymptomsbecauseofedematous
prostateinwhich'suprapubic'drainagemaybenecessarytodrainthebladder,avoidingbladdercatheterization
throughurethra,whichpredisposestobacteremia.Mostofthetimeitisconfusedandtreatedascystitis.The
strikingexaminationfindingwillbetenderdiffuselyenlargedboggyprostateonDRE,whichisdonebygentle
fashionavoidingprostaticmessage,asitwillbepainfulandalsotoavoidbacteremia.Urinanalysisisalwaysthe
firststepalongwithGramstainofurinetoguidetheantibiotictherapy.Positiveurine/bloodculturesand
leukocytosissupportthediagnosis.PSAmaybeelevatedbutisnotdiagnosticandifelevated,shouldbe
repeatedafterinfectionresolvesifremainselevatedbiopsyshouldbedonetoruleoutmalignancy.
Patientswithsepsislikepicture(alteredmentalstatus,hypotension)andwhocannottakeoralantibiotics
secondarytonauseaandvomitingshouldbeaggressivelytreatedwithIVantibiotics,IVfluidsandifneeded
vasopressors.ThemostcostaffectiveregimenwillbeIVampicillinandgentamycin(getBUNandCreatinine
beforeyoustartGentamycin)empiricallybeforethecultureresultsavailable.Oncepatientisafebrile,oral
antibioticscanbeused.EitherBactrim(TMP+SMX)orciprofloxacinaretheoralantibioticsofchoice.Once
thecultureresultsareavailablepatientshouldbemaintainedonappropriateantibioticsfor4weekstoprevent
complicationssuchasprostaticabscessorsepticemia.Patientshouldbeseenonregularvisitswithurine
analysis.Iftreatedappropriatelychancesofacuteprostatitisgoingforchronicformandabscessisrare.

Location:
Office
CC:
Fever,cough,andrightsidedchestpainfor3months
Vitalsigns:
T37.8C(100F)BP120/70mmHgP90/minR18/minH65cm(5'5")W54.5kg(120lb)
HPI:
A30yearoldpreviouslyhealthywomancomestothephysicianwithfever,drycough,andpainintheright
lowerchest.Thepainisgradualinonset,45/10inintensity,sharp,nonradiating,andincreaseswithdeep
breathing.Herothersymptomsincludemildexertionaldyspnea,irritability,decreasedappetite,andfatigue.She
hasalsohadunintentionalweightlossofabout4.5kg(10lb)overa2monthperiod.Hermenstrualcyclesare
regularandthelastmenstrualperiodwas2weeksago.Thepatientissexuallyactivewithherboyfriendandthey
usecondomsforcontraception.Herimmunizationsareuptodate.Shehasnoallergiesandtakesno
medications.Thepatienthasnottraveledrecentlyorhadanysickcontacts.Shedoesnotsmokecigarettesbut
occasionallydrinksalcohol.Hermotherhasajointdisease.
Approachtothiscase:
Becausethispatientisstablebasedonthevitalsigns,orderthefollowingphysicalexamination:
General
HEENTandneck
Chest/lungs
Heart
Abdomen
Extremities
CNS
Skin
Physicalexaminationfindings:
General:Thepatientisanxiousandsittingcomfortablyontheexaminationtable.
Skin:Anerythematousrashispresentoverthefaceandismorepronouncedinthemalarregion.
Chest/lungs:Decreasedbreathsoundsandtactilefremitus,alongwithdullnesstopercussion,arenotedoverthe
rightlowerlobe.Nowheezingorcracklesarenoted.
Therestoftheexaminationiswithinnormallimits.
Clinicalimpression:
A30yearoldpreviouslyhealthywomanwithsymptomsofpleurisyhasfindingssuggestiveofpleuraleffusion
andamalarrash.Shealsohasafamilyhistoryofjointdisease.Systemiclupuserythematosusishighonthelist
ofdifferentialdiagnosesforthispatient.
Becausethepatientneedstobeevaluatedwithpleuralfluidanalysis,shifthertothewardbyclickingon"change
locationtoward."
Orders:
Pulseoximetry,stat
CBCwithdiff,routine
BMP,routine
Chestxray,PAandlateralviews,routine
ECG12lead,routine
SerumANA,routine
Urinalysis,routine
ESR,routine
Activity:Ambulationatwill

Diet:Regular
Vitalsigns:Every4hours
Callwhenresultsareavailable.
Results:
Oxygensaturationis94%onroomair
Laboratoryresults:
Hemoglobin
MCV
Leukocytes
Platelets
Glucose
Sodium
Potassium
Chloride
Bicarbonate
BUN
Creatinine

10g/dL
86fL
10,000/mm3(neutrophils62%lymphocytes29%,eosinophils3%,monocytes6%)
140,000/mm3
118mg/dL
135mEq/L
3.8mEq/L
110mEq/L
18mEq/L
16mg/dL
1.0mg/dL

Urinalysis
SerumANA

normal(nohematuria,casts,protein,nitrites,orleukocyteesterase)
positiveat1:320

Chestxrayshowsrightsidedpleuraleffusion.ECGshowsnormalsinusrhythm.
Orderreview:
Chestxray,rightlateraldecubitus
PT/INR,routine
PTT,routine
AntiDsDNA,routine
ComplementC3,serum,routine
ComplementC4,serum,routine
Callwhenresultsareavailable.
Results:
Decubitusfilmshowsa1.5cmfreeflowingeffusionwithoutloculation
PT/INR/PTTnormal
AntiDsDNApending
Orderreview:
Thoracocentesis,diagnostic
Informedconsent
Pleuralfluidanalysis(Gramstain,culture[aerobicandanaerobic]andsensitivity,protein,glucose,lactate
dehydrogenase[LDH],cellcountwithdifferential,cytology,acidfaststainandculture,pH)
SerumLDH,routine
Serumprotein,routine
Performphysicalexaminationafterthethoracocentesis
Results:
SerumC3andC4arelow
AntiDsDNAispositive
SerumLDHis90U/L
Serumproteinis7g/dL,albuminis2.5g/dL

Pleuralfluid(PF)analysis:
Appearance
pH
LDH
Protein
Glucose
Gramstain

clear
7.6
240U/L
10.5g/dL
46mg/dL
negativefororganisms

(Calculatetheratios:PFLDH/serumLDH=2.7,PFprotein/serumprotein=1.5,whichischaracteristicofan
exudate)
Orderreview:
Rheumatologyconsult,stat(Reason:30yearoldwomanwithconfirmedSLE.Pleaseevaluateandtreat)
Prednisone,oral,continuous
Patientcounselingaboutsafesexcontraceptionregularexercisenosmoking,alcohol,orillicitdrugs
medicationcompliance
Lowfat,highfiberdiet
Dischargetohome
Reviewafter2weeks
Discussion:
Apleuraleffusioncanbeidentifiedonchestxraywithbluntingofthecostophrenicangle.Thenextstepwould
betoobtaindecubituschestradiographs,withthedependentside(sideofpleuraleffusion)onthetableinthis
case,rightlateraldecubitusxray.Ifafreeflowingfluidispresent,layeringwillbeseen(versusnotfree
flowing,inwhichcaseitisaparapneumoniceffusion).Ifthedistancebetweentheinsideofthethoraciccavity
andtheoutsideofthelungis1cm,thepleuraleffusionisnotclinicallysignificantandwouldbedifficulttotap
(riskoutweighsbenefit).Ifthedistanceis1cm,asamplecanbeobtainedfordiagnosis.
Adiagnosticthoracentesisshouldbeperformedonnearlyeverypatientwithfreepleuralfluidthatmeasures1
cmonthedecubitusfilms.However,inapatientwithobviouscongestiveheartfailure,theprocedurecanbe
withhelduntiltheheartfailureistreated.
PatientswithPTorPTT>twicenormal,plateletcount<50,000/L,and/orserumcreatinine>6mg/dLareat
increasedriskforbleedingcomplicationsfollowingthoracentesis.Insuchcases,cautionand/orjudicious
correctionofcoagulationabnormalitiesisrecommended.
Thepleuralfluidcanbeanexudateortransudate.Exudativeeffusionsresultfromincreasedcapillaryproteinleak
duetopleuralandlunginflammation.Transudativeeffusionsresultfromtheimbalancebetweenhydrostaticand
oncoticpressuresinthepleuralspace.
Light'scriteriaareusedtodifferentiateanexudatefromatransudate.Ifatleast1ofthefollowing3criteriais
present,thenthefluidisdefinedasanexudate:
1.Pleuralfluidprotein/serumproteinratio>0.5or
2.PleuralfluidLDH/serumLDHratio>0.6or
3.PleuralfluidLDH>2/3rdoftheupperlimitofnormalserumLDH
ThedeterminationofpHisimportantinparapneumoniceffusions,inwhichavalue7.2requireschesttube
aspirationtopreventempyema.
Themostcommonclinicallysignificantcomplicationofthoracocentesisisapneumothorax.However,routine
chestxrayafterthoracentesisisnotindicated.Carefulphysicalexaminationisallthatisrequired.
Commoncausesoftransudativepleuraleffusions:
Congestiveheartfailure

Cirrhosis
Nephroticsyndrome
Peritonealdialysis
Commoncausesofexudativepleuraleffusions:
Malignancy
Infection(bacterialpneumonia,tuberculosis,fungal/atypical,viral/parasiticinfections)
Pulmonaryembolism*
Gastrointestinaldisease(eg,esophagealperforation,pancreatitis)
Connectivetissuediseases(rheumatoidarthritis,SLE)
*Effusionssecondarytopulmonaryembolismcanbeatransudateoranexudate.
Diagnosisoflupuspleuritisisbasedprimarilyontheclinicalpresentationandserologicfindingsforlupus.
Treatmentinvolvesatrialofnonsteroidalantiinflammatorydrugsorglucocorticoids.Immunosuppressiveagents
arerarelyindicated.
Primarydiagnosis:
PleuraleffusionsecondarytoSLE

Location:Office
Vitalsigns:
T.38.9C(102F),HR96/min,RR21/minandBP118/65mmHg.
C.C:Chestpain
HPI:
A7yearoldAfricanAmericanchildwhohasahistoryofsicklecelldiseaseisbroughttotheofficebyhis
motherwithaonedayhistoryofrightarmpain,chestpain,fever,nonproductivecough,andmildshortnessof
breath.Hedescribetheleftsidedchestpain,89/10,constant,aggravatedbydeepbreath,butnotrelievedby
anything.Childdenieshavinghadanykindofinjury.Patientisonprophylacticpenicillin,andhadreceived
polyvalentpneumococcalvaccine.Hispastmedicalhistoryissignificantforsicklecellanemia.Developmental
history:AgeAppropriateperDenverDevelopmentalgrowthscale.FamilyHistory:Sicklecelldiseasein
maternalcousin.Noothermajormedicalproblemsinthefamily.Socialhistory:Liveswithmom,dadand
oldersister.Nosmokersathome.All:None.Meds:PenicillinVKandfolicacid.Howdoyouapproachthis
patient?
Orderphysicalexam:
General
HEENT/Neck
Lungs
Heart
Abdomen
Extremities
Skin
Results:
The7yearoldchildisinmoderatetoseverepain,andmildrespiratorydistress.Pallorispresent.Severelocal
tendernessisnotedovertherightarm,ribsandsternum.Heartsoundsarenormal.Nomurmurs/rubs/gallops.
Therearedecreasedbreathsounds,andincreasedtactilevocalfremitus(TVF)presentovertherightmiddle
lobe.
Orderreview:
Pulseoximetry,stat
Results:
88%onroomair
Orderreview:
Admitthepatienttofloor/ward
Oxygen,inhalation(tokeeppulseoxgreaterthan93%)
IVaccess
CBCwithdiff,stat
BMP,stat
Bloodcultures
Urinanalysisandurinecultures
Sputumgramstainandculture
ChestXray,PA,andlateral,stat
EKG,12lead
Typeandscreenforblood
VitalsQ4hours
Pulseoximetry,continuous(or)Q12hoursdependingontheseverityofhypoxia

Regulardiet
IVhypotonicfluidsD51/4NSSorD5WNS,bolusandthencontinuous
IVMorphine,continuous
IVCefuroximeandIVAzithromycin
Incentivespirometry
Albuterolnebulization,Q4hours
Considerbloodtransfusionifhypoxiadoesnotimproveonnasalcanula
Next:
Advancetheclocktoseethenextavailableresult
Reexaminethepatientin1hourandtakeinterimhistoryandfocusedlungexam
Order:
Repeathemoglobinandhematocritin4hoursandagainin8hoursifstable
Patientwillsaythatheisfeelingmuchbetter
StopIVfluidsnextday
RepeatCBCwithdiff,andBMPnextday
*Usuallythecasewillendifyoudothismuch.
Discharge:
Observethepatientfor48hours,stopIVantibioticsoncetheculturesarenegativeanddischargehimtohome
onoralantibiotics
Patienteducation
Highcaloriediet
Discussion:
Acuteandchronicpulmonarycomplicationsarethemostcommoncauseofdeathinpatientswithsicklecell
anemia.Recurrentmicrovascularobstructionleadingtopulmonaryhypertensionandpulmonaryfibrosisisthe
underlyingpathology.TheprimaryacutepulmonarydiseasesseeninpatientswithSCDinclude:
1.Acutechestsyndrome
2.Infectionorpneumonia
3.Infarctionduetoinsituthrombosis
4.Fatembolismfrombonemarrowinfarction
Differentiationofpneumoniafromacutechestsyndromeisdifficult,asthesputumandbloodculturesareoften
nondiagnostic.
Microvascularocclusionresultinginbonemarrowinfarctioncancausepulmonaryfatembolism(PFE).Sickle
celldisease(SCD)patientswhohavePFEmanifestalteredmentalstatus,thrombocytopenia,petechialrash,
fallinghematocrit,andseverehypoxemia.ThetherapeuticoptionsforPFEincludeexchangetransfusion,
plasmainfusions,andglucocorticoids.
AcutechestsyndromeisthemostcommonformofacutepulmonarycomplicationinpatientswithSCD.
Intravascularsicklingresultinginmacrovascularormicrovascularinfarctionwithinthepulmonaryvasculature
isassumedtobethemostfrequentcauseofACSinpatientswithSCD.Previousstudieshaveshownthatupto
50%ofACSistheresultofbacterialinfection.ACSshouldbesuspectedifthepatientpresentswithchest
pain/extremitypain,feverof>38.5C,tachypnea,wheezing,orcough.Onexamination,localtendernessover
theribsorsternumandsignsofconsolidationarepresent.Leukocytosis,thrombocytopeniaorthrombocytosis,
fallinghemoglobinconcentration,elevationsinlactatedehydrogenaseandbilirubinlevels,presenceofanew
pulmonaryinfiltrate,involvingatleastonecompletelungsegment(notatelectasis)aretheothercommon
findings.

TheacutemanagementofACSisprimarilyasupportivecare:
1.Patientshouldbegivensupplementaloxygentopreventfurthersickling
2.Painshouldbecontrolledwithjudicioususeofnarcotics
3.Dehydrationshouldbecorrectedwithvolumereplacement
4.Incentivespirometrycanbeusedtopreventatelectasis.SomeSCDpatientsmaymanifestairway
hyperreactivity,andtheseshouldbegivenbronchodilators.
5.AntibioticstocoverS.pneumoniae,H.influenzaetypeb,Mycoplasmapneumoniae,andChlamydia
pneumoniae.
6.Exchangetransfusionisindicatedforpatientswithhypoxiawithnewinfiltrates.
Chronictherapy:
1.PenicillinprophylaxiswithpenicillinVpotassium125mgBIDisrecommendedinchildrenaged4
monthstoatleast3years.
2.Polyvalentpneumococcalvaccineforbothadultsandchildren.
3.Hydroxyureaforrecurrentvasoocclusivecrisis.
PrimaryDiagnosis:
SickellcellAnemia

Location:ER
CC:A28yearoldCaucasianfemalewithsuddenonsetofshortnessofbreathandswellingoftheface
Vitalsigns:
Hertemperatureis36.7C(98F),bloodpressureis100/62mmHg,pulseis126/min,andrespirationsare38/min.
HPI:A28yearoldCaucasianfemaleisbroughttotheemergencyroombyherfriendbecauseofthesudden
onsetofshortnessofbreathandswellingofherlips,faceandextremities.Thesymptomshavestartedsuddenly
afterabeesting.Shedeniesanyothermedicalproblems.Otherhistoryisunobtainable.
Howdoyouapproachthispatient?
Youhavea28yearold,healthy,femalewithnoothermedicalproblemswithsuddenonsetshortnessofbreath
andwhatappearstobeanasymmetricedemaofthebodyafterabeesting.Thisisallquitconsistentwith
anaphylacticreactionwithangioedema.Othercommonacuteconditionssuchasasthmaexacerbation,
pneumothorax,acutepulmonaryembolism,andpanicattacksareunlikelygiventheasymmetricedemaofthe
body.
Thefirststepinthemanagementofthispatientistosecureairway,breathingandcirculation.Inallpatientswith
airwaycompromiseorvasomotorinstability,epinephrineshouldbepromptlyadministered.
Order:
Pulseoximetry,stat
Oxygeninhalation,stat
Cardiacmonitor,stat
ContinuousBPmonitoring,stat
IVaccess,stat
Epinephrine,intramuscular,stat(repeatevery15to20minutesifthereisnoimprovement)
Normalsaline,IV,bolus
Normalsaline,IV,continuous
*Advanceclockby5minutesandrepeatepinephrineifneeded.
Herearetheresults:
Sheissaturating82%onroomairand93%on4litersoxygen
Sheisfeelingbetterwithonedoseofepinephrine
*Ifthepatientishavingsevererespiratorycompromise(cyanosis,stridor,lowoxygensaturationsdespiteof
100%oxygen),sheshouldbeintubatedandmechanicalventilationisrequired.Youcanorderlike:
Intubation,endotracheal,stat
Ventilation,mechanical,stat
Now,performthephysicalexam:
General
Skin
HEENT/Neck
Lungs
Cardiovascular
Abdomen
Extremities
Physicalexamfindings:
Thepatientisveryanxious,frightenedandinmildtomoderaterespiratorydistress.Thereisnoskinrash.Face,
lips,neckandtongueareswollen.Pupilsareequalandreactingtolight.Mucusmembranesaredry.Diffuse

expiratorywheezingispresent.Extremitiesareacyanoticandswollen.Nonpittingedemaispresentinan
asymmetricfashionoverallextremities.
Revieworders:
ABG,stat
CXR,portable,PA,stat
12LeadEKG,stat
CBCwithdiff,stat
BMP,stat
Albuterol,nebulization,continuous(forbronchospasm)
Hydrocortisone,IV,continuous
Diphenhydramine,IV,continuous
Ranitidine,IV,continuous
Herearetheresults:
ABGPo270%on4litoxygen,PCo230,pH7.5(consistentwithrespiratoryalkalosis)
BMPisinsignificant
CBCWBCcountiselevatedat12,000/cmmwithmildleftshiftHbandplateletsarenormal.
EKGshowssinustachycardia
CXRiswithinnormallimits
Revieworders:
TransfertoICU
Dopamine,IV,continuousThiscanbestartedifthepatientisstillhypotensivedespiteofreceivingintravenous
fluids.
ManagementinICU:
Bedrest
Vitalsroutine(perICUprotocol)
Pulseoximetry,routine,every4hours
Urineoutput,routine
Diet,NPO
*Examinethepatientforevery30minutesinthebeginning
Results:
Patientisimprovingwiththetreatment
*Usuallythecasewillendifyoumanagethismuchincaseifitcontinue
Discussion:
Oncethepatientisimprovedchangethelocationtoward/floor
NowdiscontinueIVfluids,IVsteroids,IVdiphenhydramineandotherIVmedications
Startprednisone,oral,continuous
Startdiphenhydramine,oral,continuous
Dischargeplanning:
Dischargethepatientafter24hoursofobservation.Scheduleanoutpatientradioallergosorbenttest(RAST)
testingafter6weeks.Also,requestanimmunologistconsultasanoutpatient.Iftheyaskreason,type:Patient
withrecentepisodeofanaphylaxis.Pleaseevaluatewithskintesting.Ordermedicalalertbraceletandcounsel
aboutmedicationcompliance.Ideally,weshouldprovidepreloadedsyringeswithepinephrine(EpiPen)tothe
patientandadviceavoidallergens.
*USMLEsoftwaredoesn'thaveoptionsforEpiPenandavoidallergens.

Finaldiagnosis:
AcuteanaphylaxiswithangioedemafromBeesting
Discussion:
Managementofanaphylaxisstartswithrapidassessmentofairwayandbreathing.Airwayshouldbesecuredand
patientsmayrequireintubationifthereissignificantupperairwayedema,stridor,cyanosis,orifthepatientis
notimprovingwithoxygeninhalation(frombronchospasm).Thedrugofchoiceforacuteanaphylaxisis
epinephrine,typicallygiveninstramuscularlyinmildtomoderatecasesintravenousepinephrineshouldbe
giveninpatientswithseverehemodynamicinstability,severeupperairwayedema,orseverebronchospasm.If
thepatientisonbetablockersforsomereason,theeffectofepinephrinemaynotbepromptandenoughin
thesepatientsglucagonisgiven.Oncetheairwayandbreathingissecure,hemodynamicstatusshouldbe
assessed.Patientsmayrequireseverallitersofcrystalloidstomaintaintheperfusion.Ifthepatientisnot
respondingtocrystalloids,vasopressorssuchasdopamineorphenylephrineshouldbestarted.
AcombinationofbothH1andH2blockersshouldbegiventoallpatients.Corticosteroidsarealsoroutinely
giventopreventthelatephasereaction.
Oncetheacuteepisodeisover,thepatientshouldhaveformalevaluationwithallergist.Thepatientshouldalso
haveskintestsandRASTtoidentifytheallergen.Patient'sshouldbeeducatedandprovideEpiPen.

Location:Office
Vitalsigns:
Temperatureis36.7C(98F),bloodpressureis162/94mmHg,pulseis78/min,andrespirationsare14/min.
CC:A35yearoldAfricanAmericanmalecameforroutineexam.
HPI:
A35yearoldAfricanAmericanmancomestothephysician'sofficeforaroutinehealthmaintenance
examination.Hecomplainsofweaknessandanorexiaforthepastfewweeks.Hisreviewofsystemsispositive
onlyforflankdiscomfort.Hehasnoothermedicalproblems.Thepatientisapostalworker.Hedoesnotdrink
alcoholandhassmokedonepackofcigarettesdailyfor15years.Hisfatherdiedattheageof45fromkidney
disease.Hetakesnomedicationandhasnoothercomplaints.
Howdoyouapproachthispatient?
Wehavea35yearoldmalewithweakness,anorexia,andflankdiscomfort.Hisfamilyhistoryispositivefor
kidneydisease.
Performphysicalexam:
General
HEENT/Neck
Lungs
Heart
Abdomen
Extremities
Herearetheresultsoftheexam:
Examinationissignificantforbilateralpalpablecystickidneys.Clearlungfieldsandnormalfirstandsecond
heartsoundsarenoted.Noankleorperiorbitaledemaisnoted.Restisunremarkable.
Discussion:
Basedonhispresentationandthepresenceofbilaterallyenlargedcystickidneys,itismostlikelyanautosomal
dominant,polycystickidneydisease(ADPKD).Othercausessuchascongenitalsimplecysts,medullarycystic
kidneydisease,andacquiredcystickidneydiseaseshouldbeconsideredintheDD.
Order:
CBCwithdifferential,stat
BMP,stat
UA,stat
Ultrasoundkidneys,stat
Resultsofyourtests:
TheUAshowedmanyRBC(50+),negativenitrite,negativeesterase,nobacteria,andfewwhitecells.Nocasts
areseen.
CBCshowedHbof9.0g/dL,MCV88fl,WBCof6,000/cmmandplateletsof230,000/cmm.
BMP
SerumNa
SerumK
Bicarbonate
BUN
SerumCreatinine
Calcium
BloodGlucose

135mEq/L
6.8mEq/L
17mEq/L
76mg/dL
6.4mg/dL
8.0mg/dL
118mg/dL

Ultrasoundofthekidneysshowsbilateralpolycystickidneys.
Discussion:
Thispatienthasevidenceofrenalfailure(mostlikelychronic)withresultingmetabolicacidosis,hyperkalemia,
hypocalcemia,andanemiaofchronicdisease.Thispatientneedstobeadmittedinthehospitaltomanagehis
acidosis,hyperkalemia,andforlongtermdialysisbyanephrologist.Onceyouseehyperkalemiaofthisdegree,
anEKGshouldbeobtainedand,ifthereareanyEKGchanges(peakedTwaves),calciumgluconateshouldbe
giventostabilizethemembrane.Insulinanddextroseisusefulintheacutesettingforrapidcorrection.Sodium
bicarbonateisusefultocorrectbothacidosisandhyperkalemia.Kayexalateistheonlymedication,which
removesthepotassiumfromthebody.Oncethesetreatmentsaregiven,thepotassiumlevelshouldberechecked
in24hoursand,ifitisstillhigh,medicaltreatmentshouldberepeated.Ifitisstillhighafterthesecondround
oftreatment,dialysisisindicated.
Oncethepatientisoutofdanger,otherevaluationshouldbecontinued.Serumphosphoruslevelshouldbe
checkedandPhosLo(calciumacetatebindsphosphorusandpreventsabsorption)isrecommendedifitis
elevated.Irondeficiencyisalsocommonlyassociatedwithchronicrenalfailureso,thisshouldberuledout
withironstudies.AnemiaofchronicdiseasewithanHbof<10isanindicationtostarterythropoietin.Blood
pressureshouldbecontrolledwitheitherACEinhibitors(iftheCrislessthan22.5)orcalciumchannel
blockers(iftheCrisveryhigh).Arenaldiet,whichhaslowpotassium,lowprotein,andlowphosphorus,
shouldbeordered.
Ifthereisnofamilyhistoryofbrainaneurysms,routinescreeningforbrainaneurysmsisnotindicated.
Order:
EKG,12lead,stat
Results:
EKGshowspeakedTwaves.
Order:
CalciumgluconateIV,bolus
D50andinsulin,stat,IV,bolus
Sodiumbicarbonate,oral,continuous
Kayexalate,oral,continuous
Now,changethelocationofthepatienttoward:
Vitals:Q6hours
Diet:renal
Cardiacmonitor:routine
Urineoutput:routine
ActivityOutofbedtochair
Nephrologyconsult,stat(Reason:35yearoldmalewithrenalfailuresecondarytoADPKDwithresulting
acidosisandhyperkalemia)
Checkserumpotassium24hourslater
BMP:every12hours
Phosphorus,serum,stat
Serumiron,ferritin,andTIBC:routine
Amlodipine,oral:continuous
Results:
Phosphorusishigh.
Festudiesindicateanemiaofchronicdisease.
Order:
Calciumacetate,oral,continuous

Erythropoietin,subcutaneous,routine
Finallycounseling:
Counselpatient,smokingcessation,routine
Counselpatient,noillegaldruguse,routine
Counselpatient,exerciseprogram,routine
Counselpatient,medicationcompliance,routine
Finaldiagnosis:
Adultautosomaldominant,polycystickidneydisease(ADPKD)resultinginrenalfailure.

Location:Emergencyroom
Vitals:Temperatureis36.7C(98F),bloodpressureis120/76mmHg,pulseis80/min,andrespirationsare
16/min.Heightis5'6"(167.5cm)andweightis60kg(132.3lb)
HPI:
A68yearoldwhitefemalewholivesaloneathomeispresentedtotheERbecauseofthesuddenonsetof
severelowbackpainwhileremovingaturkeyfromthefreezer.Thepainis10/10inseverity,constant,and
thereisnoradiation.Shehasneverhadthispainbefore.Shedeniesweakness,sensorychangesinthelegs,or
bowelorbladderincontinence.Shealsodeniesanyfever,chills,lossofappetite,andlossofweight.Sincethe
ageof53,shehasbeenpostmenopausalandhasnottakenanyhormonereplacementtherapy.Heronlyother
medicalproblemishighbloodpressure,forwhichshetakeshydrochlorothiazide.Shehashadnosurgeries.
Shesmokesonepackofcigarettesperday,forthepast40years.Occasionally,shedrinksalcoholandhasno
knowndrugallergies.Familyhistoryisnotsignificant.HerlastPapsmearwassixyearsago,anditwas
normal.Shehadherlastmammogramoneyearago,anditwasnormal.Acolonoscopyperformedfiveyears
agowasalsonormal.TherestofROSisnegative.
Howwouldyouapproach?
Backpainhasbroaddifferentialdiagnosis.Sincehervitalsignsarestable,performaphysicalexam.
Order:
SelectGeneral,HEENT,Skin,Lymphnodes,Lungs,Heart,Abdomen,Rectal,Extremities,andNeurological.
Results:
Patientismoaninginseverepain.SeveretendernessispresentovertheL3andL4area.Straightlegraisingtest
(SLR)at90degreesisnegative.Analsphinctertoneisnormal.Neurologicalexaminationisunremarkable.
Therestoftheexamisnormal.
Order:
IVaccess,stat
IVToradol,stat,onedose
CBCwithdiff,stat
BMP,stat
Serumcalcium,stat
ESR,routine
Spinexray,lumbosacral,stat
*Callmewhentheresultsareavailable.
Results:
CBC,BMP,calcium,andESRarenormal.
SpinexrayshowsdiffuseosteopeniaandavertebralcompressionfractureatL3andL4.
*Orderbriefhistory.
Results:
Patientisfeelingmuchbetter
Order:
SPEP,routine(toruleoutmyeloma)
TSH,serum,routine
Naproxen,oral,continuous
Calciumcarbonate,oral,continuous
VitaminDtherapy,oral,continuous

Fosamax(alendronate),oral,continuous
Calciumenricheddiet
Counseling:
Patienteducation
Smokingcessation
Limitalcoholintake
Regularexercise
Medicationcompliance
Seatbeltuse
*Changethelocationofthepatienttohome.Scheduleanoutpatientappointmentin37days.
Order:
Onceshecomes,orderextremitiesandneuroexam
DEXAscan,routine
Finaldiagnosis:
Osteoporoticvertebralcompressionfracture
DDofacutebackpain(ExtremelyimportantforStep3MCQS):
Vascularcauses:
Acuteaorticdissection:Tearingabdominalpainradiatingtotheback,andassociatednauseaandvomitingare
present.Cluesontheexamarehypertension,unequalupperextremitybloodpressure,andnewonsetofan
aorticinsufficiencymurmur.
Abdominalaorticaneurysm:Painradiatingtotheback,unstablevitalsigns,abdominalbruit,andpulsatile
abdominalmassaretheclues.
Mechanical:
Vertebralfracturewithorwithoutcordcompression(osteoporosisormalignancy):Riskfactorsforosteoporosis
(smoker,thinbodyhabitus,steroiduse,etc.)arepresent.Systemicfeaturessuggestiveofmalignancy(weight
loss,lossofappetite)maybepresent.Historyofliftingheavyobjectsmaybepresent.Spinaltendernesswillbe
present.SLRisusuallynegative.Malignancyshouldberuledoutinelderlypatients,especiallymultiple
myelomaAgeappropriatescreeningproceduresshouldbedone.
Diskprolapsewithorwithoutcordcompression:Backpain,buttockpain,andparesthesiasradiatingdownthe
leginadermatomaldistributionisaclassicpresentation.Historyofliftingheavyobjectsisoftenpresent.SLR
testatorabove60willbepositive.
Lumbarstrain:Paravertebralmusclespasmwithoutfocalspinaltendernessispresent.SLRwillbenegative.
MusclerelaxantsandNSAIDsaresufficient.
Caudaequinasyndrome:Suddenonsetofbackpain,lowerextremityweakness,bowelorbladderincontinence,
andsaddleanesthesiaaretheclues.
Epiduralhematoma:Suddenonsetofbackpain,withunderlyingcoagulationdisorder(hereditaryor
anticoagulantuse)ispresent.
Spinalepiduralabscess:Predisposingfactors,suchasDM,renalfailure,andIVdruguse,etc.,arepresent.
Fever,radicularpain,elevatedwhitecount,andwithorwithoutcordcompressionisseen.
Management:
Basiclabstoruleoutinfection,suchasCBC,areindicated.ESRisusefultofindoutanyoccultmalignancy
suchasmyelomaoranyevidenceofinfection.Spinalxraysareindicatedinmostofthepatients,exceptifthe

patienthaslumbarstrain.MRIisindicatedonlywhenthereareneurologicalmanifestations.Referraltoan
orthopedicsurgeonorneurosurgeonmaybeneedediftheMRIisabnormal.MyelomashouldberuledbySPEP
and/orurineBenceJonesprotein.NSAIDs(betterthannarcotics),andearlymobilizationisthemaintreatment
forvertebralfractures.Secondarycausesofosteoporosis(hyperthyroidism)shouldbelookedforifthereisa
suspicion.ObtainingTSHisroutineinthesepatients.Calcium,vitaminD,andbisphosphonatesshouldbe
giventopreventtheosteoporosisandsecondfracture.Counselingregardingtheriskfactors(asadvisedabove)
isalsoveryimportant.Surgeryisindicated,ifthereisnoimprovementwithconservativemanagement.

Location:Emergencyroom
Vitalsigns:Pulse:130/minBP:80/40mmHg,Temperature:39.4C(103F),RR:22/min.
HPI:
Ahealthy22yearoldCaucasianwomanisbroughttotheemergencydepartment.Shesaysthatshehasbeen
havingfever,chills,myalgias,andheadacheforthepast36hours.Forthepast12hours,shehasbeennauseous,
hadtwoepisodesofvomitingandcomplainingofabdominalpainanddiarrhea.Shehasalsodevelopeda
diffuseskinrash.Shehasnoothermedicalproblems.Shedoesnotusetobacco,alcohol,ordrugs.Family
historyisnotsignificant.Shetakesnomedication.Shehasneverbeenpregnant.Hermenstrualcyclesare
alwaysregular.Thisisherfifthdayofmenstruation.Sheissexuallyactivewithherboyfriendanduses
condoms.Shehasnoknowndrugallergies.
Howwouldyouapproachthispatient?
Youhavea22yearoldhealthyfemalepresentingwithafebrileillnesscomplicatedbyhypotension.The
definitivediagnosisisextensiveatthisstage.Sheishemodynamicallyunstableatthistime.Resuscitation
(ABC)isthefirstpriority.
Order:
Pulseoximetry,stat
IVaccess,stat
Cardiacmonitor,stat
Continuousbloodpressuremonitoring,stat
Normalsaline,0.9%NaCl,continuous
Results:
Pulseoximetryis97%onroomair
Exam:
Ordercompletephysicalexam,exceptbreast,lymphnodes,andneuro/psych(i.e.,General,Skin,HEENT/Neck,
Lungs,Heart,Abdomen,Genitalia,Rectal,andExtremities).
Physicalexamresults:
Patientappearsverysickandanxious.Lungs,heart,abdomen,andrectalisnormal.
Diffuse,blanching,macular,erythematousrashispresentalloverthebody.Hyperemiaofthevulvaandvagina
aswellastendernessoftheexternalgenitaliaandadnexaispresent.Avaginaltamponisnoted.
Orderreview:
Tamponremoval,stat
TamponCandS,stat(optional)
CBCwithdiff,stat
CMP,stat(CMPconsistsofbothbasicmetabolicpanelandliverfunctiontests)
PT/INR,stat
PortableCXR,AP,stat
EKG,stat(12lead)
UA,stat
UrineC&S,stat
Bloodcultures,stat
IVclindamycin,continuous
Herearetheresults:
CBCshowsaWBCof24,000/cmmwith30%bands,Hbof11g/dLandplateletcountof120,000/cmm.
CMPshowsBUNof32,Crof1.8,elevatedAST,ALT,andtotalbilirubin.
EKGshowssinustachycardiaCXRandPT/INRarewithinnormallimits.

UAandUrineCandSarepending.
*Evaluatepatientbyorderingabriefhistoryandcheckvitalsigns.
Results:
Patient'sBPisstill80/60afterfluidbolusandcontinuousIVNS
Patientisstillfeelingnauseated
Orderreview:
IVDopamine,continuous
IVPhenergan(Promethazine),stat,onedose(fornausea/vomiting)
*TransferthepatienttoICU
Orderreview(ICUorders):
NPO
Bedrest,complete
Foleycatheter
Urineoutput
Acetaminophen(Tylenol)therapy,oral,continuous(forfever)
CBCwithdiff,nextday
CMP,nextday
*Evaluatethepatientbyorderingabriefhistoryandcheckvitalsigns.
Results:
Patient'sBPis100/70afterfluidsanddopamine
Patientisfeelingbetter
*Callmewiththenextsetofvitals.
Ifyoucandothismuch,thecasewillendautomaticallyhowever,ifitcontinues,followthis:
Oncethepatientisfeelingbetter,discontinue(D/C)alltheIVsubstances.Doubleclickwiththemouseonthe
orders,anditwillaskyou,"Doyouwanttocancelthisorder?"
Discontinuedopamine,IVfluids,IVantibiotics,IVPhenergan,cardiacmonitor,andFoleycatheter.
Shiftthepatienttoward
Startoralclindamycin,continuous
Contraceptioncounseling
Safesexcounseling
Smokingcessation(ifpresent)
Limitalcoholintake
Noillegaldruguse
Seatbeltuse
Reassurethepatient
Dischargethepatienttohomewith1weekfollowupappointment
Finaldiagnosis:
Toxicshocksyndromesecondarytovaginaltamponuse
Discussion:
Upto98%ofmenstrualTSScasesareduetoS.aureus.Mostpatientspresentwithaprodromeoffever,chills,
andmyalgiasthatprogresstothesystemicsyndromehowever,somepatientsmaybecomeabruptlyillwithin
hours,resultinginhypotensionandshock.TSSshouldbesuspectedinanyyoungfemalewhopresentswitha

feverof38.9C(102F),tachycardia,hypotension(SBP<90mmHg),andrash.Therashistypicallya
nonpruritic,diffuse,blanching,macularerythroderma.Desquamationstartsafter12weeks.SinceTSScan
involveanyorgan,patientsmaypresentwithGIsymptoms(nausea,vomiting,abdominalpain),hepatomegaly,
intenseheadache,andalteredmentalstatus,etc.
PotentialcomplicationsincludeARDS,shock,DIC,andrenalfailurelesscommonlyrhabdomyolysis,seizures,
andpancreatitismaybeseen.
Labs:
1.CBCshowseitherleukocytosisorleukopenia,butamarkedbandemia(upto50%)isoftenpresent.
Anemiaandthrombocytopeniaarealsocommon.
2.ElevatedBUNandcreatininesuggestsrenalfailure.
3.Hypoalbuminemiaandlifethreateninghypocalcemiacanbeseen.
4.Hyperbilirubinemiaandelevatedtransaminaselevels(ALT,AST)suggestliverinjury.
DD:(TheseareimportantforStep3MCQSaswell)
Kawasakidisease:Seenpredominantlyinchildren,rarelycauseshock,anddoesnothavesystemicinvolvement.
Staphylococcalscaldedskinsyndrome:Desquamationstartsearly,whereas,inTSS,desquamationoccursinthe
convalescentphase.Also,itrarelycausesshockanddoesnothavesystemicinvolvement.
StevensJohnsonsyndrome:Usuallyhistoryofdrugadministrationandcharacteristicmucusmembrane
involvementispresent,butlacksdesquamation.
RockyMountainspottedfever:Historyofatickbiteispresent,rashstartsperipherally,andisassociatedwitha
severeheadache,withoutanalteredmentalstatus.
Management:
Therapyismainlysupportive.Removalofthefocusofinfectionanduseofantibioticsshouldbedoneassoon
aspossible.ThepatientshouldbeadmittedinICU,andhemodynamicmonitoringandvasopressorsmaybe
necessary.Theymayneedupto1020litersofIVfluids(0.9%NS).Clindamycinhasbecomethedrugof
choicebecauseofitspotentsuppressoreffectonbacterialtoxinsynthesis.Theuseofcorticosteroidsis
controversial.

Location:Office
Vitasigns:Histemperatureis36.7C(98F),bloodpressureis120/76mmHg,pulseis80/min,andrespirations
are16/min.
HPI:A43yearoldAfricanAmericanmalepresentedtotheofficebecauseofeasyfatigability.Hehasno
fever,chills,weightloss,orlossofappetite.Hedeniescough,breathingproblem,constipation,diarrhea,
melena,bleedingperrectum,orhematuria.Hispastmedicalhistoryisnothingsignificant.Inthepast,hewas
incarceratedfordrivingundertheinfluence.Hehasbeendrinking78beersadayforthepast20years.He
smokes1packofcigarettesdailyforthepast22years.Hedeniesintravenousdruguseormultiplesexual
partners.Heisaconstructionworker,andhisdietismainlyofteaandtoast.Hetakesnomedicationandhasno
knowndrugallergies.Hislastofficevisitwastwoyearsago.TherestoftheROSisnegative.
Howwouldyouapproachthispatient?
Wehavea43yearoldalcoholicwitheasyfatigability.TheDDisextensive.Physicalexamandthebasiclabs
willnarrowthediagnosis.Hisvitalsarestable.
*Mark"completephysicalexam,"includingrectalandgenitalia.
Herearetheresults:
Theonlyabnormalfindingsontheexamare:Mucusmembranesandskinhavepallor,andnolymphnodesare
palpable.
*Thispatientmostlikelyhasanemia.ThemostlikelycausesinthispatientareirondeficiencyfromGIblood
loss(GERD,gastritis,ulcer,cancer,etc.),megaloblasticanemia(folicacidorvitaminB12deficiency),and
anemiaofchronicdisease.CBC,fecalbloodtesting,andbasicmetabolicpanelarethebasiclabsatthistime.
Order:
CBCwithdiff,stat
BMP,stat
FOBT,stat
Results:
CBCshowsHbof9.0g/dL,MCVof118fl,WBCcountof7,000/cmmwithnormaldifferential,andplatelet
countof210,000/cmm.Peripheralsmearshowshypersegmentedneutrophils.BMPisWNL.FOBTis
negative.
*Wehaveanalcoholicwhosedietismainlyofteaandtoast(heatedfoodtype)presentingwithpallor,HBof9,
andMCVof118.Themostlikelydiagnosisatthistimewouldbefolicaciddeficiency.B12deficiencyisaless
likelypossibilitybutmustberuledoutbeforeyoutreatwithfolicacidalone.Reticulocytecountisveryuseful
tofollowthesepatientsoncetreatmentisstarted.Liverfunctionstestsshouldbeobtained,sincehehasa
significantalcoholichistory.Whilewaitingforthebloodtests,thispatientshouldbetreatedwithfolicacid,
vitaminB12,ironreplacement,thiamine,andmultivitamins.Folicacidaloneshouldnotbegivenwithoutruling
outvitaminB12.Oncethefolicacidsupplementisstarted,bonemarrowironstoresarequicklyutilized,and
theymaybecomeirondeficientso,theyshouldbestartedonironaswell.Thiamineandothermultivitamins
shouldalsobegiveninalcoholics.
Order:
SerumFolate,routine
SerumvitaminB12,routine
LFT,routine
Reticulocytecount,routine
Folicacidtherapy,oral,continuous

VitaminB12therapy,oral,continuous
Ironsulfatetherapy,oral,continuous
Thiamine,oral,continuous
Multivitamin,regular,oral,continuous
Ironenricheddiet
Givecounselingatthesametime:
Patienteducation
Noalcohol
Nosmoking
Safesex
Drivewithseatbelts
Noillegaldrugs
Afterorderingtheabovetestsandappropriatecounseling,sendthepatienthomeandmakeafollowup
appointmentinoneweek.
*Patientisbackinoneweekfeelingalittlebetter
Results:
Folicacidlevelis1.5ng/mL(N=6.015.0ng/mL).
LFTsandvitaminB12arenormal.
Reticulocytecountislow.
*Ifyoudothismuch,thecasewillendautomatically.Orderareticulocytecountasafollowup.Discontinue
vitaminB12therapyatthisvisit.
Primarydiagnosis:
Folicaciddeficiency
Discussion:
Thenumberonecauseoffolatedeficiencyisnutritional(poordietand/oralcoholism).Megaloblastosis
developswithin4to5monthsinindividualsonafolatedeficientdiet.However,alcoholicsmayhaveasharp
fallinserumfolateandmaydevelopmegaloblastosiswithin5to10weeksifthedietispoorandlowinfolate.
Theothercausesoffolatedeficiencyaredrugs(methotrexate,trimethoprim,andanticonvulsants),
malabsorption(Sprue,IBD,etc.),andincreasedfolaterequirements(pregnancy,lactation,chronichemolysis,
exfoliativeskindisease).Theabovepatienthasalltheriskfactorstobecomefolatedeficient,sothereisno
needtogetironstudies.Thispatient'sMCVmaynotreturntonormalbecauseofthealcoholicliverdisease.
However,ifhisanemiaisnotcorrected,thentheothercauses(suchasassociatedirondeficiency,etc.)shouldbe
lookedfor.

Location:
Office
Vitalsigns:
T36.1C(97F)BP140/100mmHgP50/minR16/minH162.5cm(5'4")W105kg(231.5lb)
CC:
Fatigue
HPI:
A53yearoldwomancomestothephysicianwith3monthsofprogressivefatigue,constipation,cold
intolerance,andimpairedsleep.Shesaysthatshehasgainedalmost9kg(20lb)duringthistime.Thepatient
hasnofever,cough,nausea,vomiting,abdominalpain,orblackstools.Pastmedicalhistoryincludes
hypertension,carpaltunnelsyndrome,andallergicrhinitis.Hermedicationsincludehydrochlorothiazideand
loratadine.Shedoesnotusetobacco,alcohol,orillicitdrugs.Familyhistoryisnotsignificant.Thepatientisin
amonogamousrelationshipwithherhusband.Shehasnotseenaphysicianin2years.Herlastmenstrual
periodwas2yearsago,andPapsmearandmammogramperformedatthattimewerenormal.Colonoscopy
performedatage50wasalsoreportedtobenormal.Hervaccinationsareuptodate.
Approachtothiscase:
Thispatienthasbradycardia,diastolichypertension,constipation,weightgain,coldintolerance,andimpaired
sleep.Thisisaclassicpresentationofhypothyroidism.Performaphysicalexamination.
Order:
Completephysicalexamination
Results:
Onexamination,thepatientappearstobeobese.Skiniscoolanddry.Lungsarecleartoauscultation.Heart
soundsarenormal.Examinationoftheabdomenisunremarkable.Neurologicexaminationshowsdelayed
relaxationofanklereflexes.Therestoftheexaminationisnormal.
Orders:
CBC,routine
BMP,routine
FOBT,stat
SerumTSH,routine
12leadECG,stat
Results:
FOBTisnegative.ECGshowssinusbradycardia.
Note:Scheduleappointmentin3dayswithresults.
Results:
CBCshowsHbis11.8g/dLWBCis7,000/mm3withnormaldifferentialandplateletcountis200,000/mm3.
MCVis86fL.BMPshowssodiumis133mEq/LandTSHis24U/mL(normal,0.55.0U/mL).
Orders:
FreeT4,routine
Lipidpanel,routine
Levothyroxine,oral,continuous
Lowfatdietwithadequatecalciumintake
Counseling:

Patientcounseling
Medicationcompliance
Note:Followupin1week.
Results:
FreeT4is0.03g/dL(normal,0.92.4g/dL).Lipidpanelshowsincreasedtotalcholesterol.
Note:Followupin6weeks.
Order:
SerumTSH,routine
Primarydiagnosis:
Primaryhypothyroidism
Discussion:
Theclinicaldiagnosisofhypothyroidismisstraightforward,asdepictedinthiscase.Thebestscreeningtestis
TSHmeasurement.Onceitiselevated(>10),confirmationofhypothyroidismismadebymeasuringfreeT4.
BothTSHandfreeT4identifywhethertheproblemisprimary(freeT4isdecreasedandTSHiselevated)or
secondary(bothTSHandfreeT4aredecreased).
Hypothyroidismisassociatedwithnormocyticandnormochromicanemiafrombonemarrowhypoproliferation
andusuallyrespondstothyroxinereplacement.Infemalepatients,menorrhagiaresultingfromthyroid
dysfunctioncancauseirondeficiencyanemia(microcytosis).About10%ofpatientsmayhavepernicious
anemia(macrocytosis).RoutineironstudiesandvitaminB12levelsaregenerallynotindicatedifpatientshave
mildnormocytic,normochromicanemia,asseeninthispatient.Thereappearstobeanincreasedriskofceliac
diseasewithautoimmunethyroiddisorders,whichleadstomicrocyticanemiaduetoironmalabsorption.
Screeningforceliacdiseaseiswarrantedifpatientswithautoimmunehypothyroidismdevelopunexplained
microcyticanemia.Screeningforhypothyroidismisindicatedinpatientswithunexplainedhyponatremia,
hypercholesterolemia,orelevatedCK.Otherconditionscommonlyassociatedwithhypothyroidisminclude:
Sleepapnea(duetomacroglossia)
Carpaltunnelsyndrome
Depression
Cognitiveimpairment
Hashimoto'sthyroiditisisthemostcommoncauseofprimaryhypothyroidismintheUnitedStates.Antibody
measurementisnotrequiredintheworkupofoverthypothyroidism.Measurementofantithyroidantibodies
(antithyroidperoxidaseandantithyroglobulin)isperformedinsubclinicalhypothyroidism(TSHis510,freeT4
isnormal)topredicttheprobabilityofdevelopingpermanenthypothyroidism.Patientswithsubclinical
hypothyroidismandhightitersofantithyroidantibodiesaremorelikelytodeveloppermanenthypothyroidism.
Thyroxinereplacementshouldbestarted,andTSHshouldberecheckedin6weekstomonitortheresponse.
Thispatientispostmenopausaltherefore,sheshouldbeadvisedtomaintainadequatecalciumandvitaminD
intake.
Finaldiagnosis:
Primaryhypothyroidism

Location:Inhospital
Vitals:
Histemperatureis39.4C(103F),bloodpressureis90/62mmHg,pulseis110/min,andrespirationsare18/min.
HPI:
A70yearoldwhitemaleisadmittedtothehospitalforresectionofasuperficialbladdercarcinomathrough
cystoscopy.Postoperatively,hehadmildbleedingandhehasaFoleycatheterforcontinuousbladderirrigation.
Thenextday,thenursecalledyoubecausethepatientishavingahighgradefever,chills,andbecame
hypotensive.Heisalsofeelingnauseous.Hedeniescough,chestpain,shortnessofbreath,andheadache.He
hasnoothercomplaints.Hisothermedicalproblemsincludedegenerativejointdisease,BPH,andhearingloss.
Hehasnoknowndrugallergies.Socialhistoryisnotsignificant.Heiscurrentlynotonmedications,except
Tylenolforpain.HispreoperativelabsincludeHb,WBCcount,plateletcount,BUN,Cr,andPt/PTTarewithin
normallimits.
Howwouldyouapproachthispatient?
Thispatienthasahighgradefever,chills,nausea,andhypotensiononthefirstpostoperativeday.Hehasalso
hadmanipulationoftheurogenitaltracthence,thispatientisprobablyhavingsepsisfromGramnegative
bacteremiaoftheurogenitaltract.
Order:
IVaccess,stat
Pulseoximetry,stat(Result:98%onroomair)
Examine:
General
Lungs
Heart
Abdomen
Genital
Extremities
Results:
Patientishavingshakingchills.Lungsareclear.Abdomenisnottender.Bowelsoundsarepresent.Thereis
norigidityorrebound.Peripheralextremitiesarewarm.Therestoftheexamisnormal.
Order:
Normalsaline,IV,bolus
Normalsaline,IV,continuous
D/CFoleycatheter
Continuouscardiacmonitor
UA,stat
UrineC&S,stat
Bloodcultures,stat
CBCwithdiff,stat
BMP,stat
Ceftriaxone,IV,stat,continuous(orderonlyafterorderingbloodandurinecultures)
TransfertoICU
Results:
UAshowedmanybacteria,manyWBC,positivenitrite,positiveesterase,andfewRBC.CBCshowsaWBC
countof13,000with10%bands.BMPisnormal.Therestofthelabsarenormal.
Order:

ICUvitals
Pulseoximetry,every4hours
CXR,Portable,PA,stat(result:normal)
Acetaminophen(Tylenol),oral,continuous
Pneumaticcompressionstockings
NPO
Bedrest
Urineoutput
*AdvancetheclockforonehourandcheckBP
*IfhisBPisstilllowdespitethegivingofIVfluids,heshouldbestartedonIVdopamineinfusion.
*OncetheBPisstableandheisafebrile:
Shifttoward
D/CIVnormalsaline
D/Ccardiacmonitor
D/Cceftriaxone
Checktheurineandbloodcultures(usuallyafter24hours)
Changetoappropriateoralantibiotic(MostlyoralBactrim(TMPSMX)orciprofloxacin)
Primarydiagnosis:
Urosepsis
Discussion:
Patientsdonothavesignificantdifficultybreathing,lookcomfortable,andhavefewsecretions.Chestxray
showsareasofpartialcollapseWBCcountwouldnotbeelevated.
Nosocomialinfectionsaredefinedasinfectionsacquiredasaresultofhospitalization,andtheymanifestatleast
48hoursafterhospitalization.Aurinarytractinfectionisthemostcommontypeofnosocomialinfection.The
surgicalsiteinfections(accountfor19%offnosocomialinfections)arethesecondmostcommoncauseofa
nosocomialinfection.Nosocomialpneumoniaisthethirdmostcommoncauseofnosocomialinfections.Itis
frequentlyseeninpatientswhoareintubatedandhaveotherassociatedcomorbidities,suchaschroniclung
disease.Themajororganismsofconcerninnosocomialpneumoniaaregramnegativeaerobicbacterialike
pseudomonas.

Location:Office
Vitals:Hertemperatureis36.7C(98F),bloodpressureis120/76mmHg,pulseis80/min,andrespirationsare
14/min.
HPI:
A45yearoldwhitefemalepresentedtotheofficebecauseoffatigueandexertionalshortnessofbreathforthe
pasttwomonths.Thesymptomsaregettingworse.Shedeniesanychestpain,orthopnea,PND,legswelling,
palpitation,syncope,cough,andweightloss.Shehasnoconstipationordiarrhea.Shehasnotlookedather
stoolsformelena,butshedeniesanybloodinthestools.Theonlyotherproblemissomeepigastricdiscomfort
forwhichsheusuallytakesoverthecounterantacids.Shetakesnoothermedication.Hermenstrualcyclesare
alwaysregularandlast34dayseachtime.Shehasnoothermedicalproblems.Shesmokesonepackof
cigarettesforthepast20years.Occasionally,shedrinksalcoholanddoesnotuserecreationaldrugs.Hersister
hasasthmaotherwise,herfamilyhistoryisnotsignificant.Shehasnoknowndrugallergies.ThelastPap
smear,pelvicexam,andbreastexamdoneoneyearagowasnormal.TherestoftheROSisnegative.
Howwouldyouapproachthispatient?
Wehaveaveryhealthy45yearoldfemalepresentingwithfatigueandexertionaldyspnea.Theonlyother
problemthatshehasisepigastricdiscomfort.Atthispoint,theDDisbroad.Physicalexamandbasiclabswill
narrowtheDD.
Orderphysicalexam:
Completephysicalexam
Results:
Thepatientappearscomfortable.Thereisnolymphadenopathy.Pallorispresentonexam.Lungsandheartare
cleartoauscultation.Abdomenissoft,butverymilddiscomforttodeeppalpationisnotedintheepigastric
region.Extremitiesarenormal.Therestoftheexamisnormal.
Order:
FOBT(fecaloccultbloodtesting),stat
CBCwithdiff,stat
BMP,stat
UA,stat
CXR,PA/lateral,stat
Results:
FOBTispositive.CBCshowsHbof8.0g/dL,MCV72fl,WBCof6,000/cmmwithnormaldifferential,
plateletcountof220,000/cmm.Peripheralsmearshowedhypochromicandmicrocyticanemia.BMPshoweda
BUNof25andCrof1.1.UAisnormal.CXRisnormal.Therestofthelabsarenormal.
*Now,wecansaythatthepatient'smostlikelyreasonforfatigueandshortnessofbreathisfromanemia.The
causeismostlikelyduetobleedingfromtheGItract.Sincethepatienthassomeepigastricdiscomfort,wecan
startwithanupperGIendoscopy.
Order:
Admittoward/Floor
NPO
Ambulationatwill
SerumIron,routine
Serumferritin,routine
TIBC,routine
Typeandcrossmatchforblood,stat
TransfusePRBC,stat

Omeprazole,oral,continuous
PT/PTT,stat
*Advanceclockfor12hours(togiveadequatetimeforNPO)
UpperGIflexibleendoscopy,routine
Helicobacterpyloribiopsy,stomach,routine(or)H.pyloriserology,routine
Consentforprocedure
RecheckHbnextday
Results:
Serumironandferritinarelow.TIBCiselevated.Endoscopyshowedcleanbased,nonbleedingduodenal
ulcer.BiopsyforH.Pyloriispositive.
Order:
Clarithromycin,oral,continuous
Amoxicillin,therapy,oral,continuous
Iron,therapy,continuous
VitaminC,therapy,continuous(increasesIronabsorption)
Ironenricheddiet
Noaspirin
Counseling:
Patientcounseling
Nosmoking
Noalcohol
Drivewithseatbelts
Regularexercise
Safesex
*Changethelocationtohome.Followupintwoweeks
Primarydiagnosis:
Duodenalulcer
Discussion:
Ingeneral,wholebloodorserumserologyisamostcosteffectivetesttoidentifyH.pyloriinsymptomatic
patients.However,ifthepatientisundergoingendoscopy,thenendoscopicbiopsycanbeperformed.The
regimenofchoiceforH.Pylorieradicationistripletherapywithaprotonpumpinhibitor,Clarithromycin,and
Amoxicillinforabouttwoweeks.Oncethetreatmentiscompleted,confirmationoferadicationis
recommended.Ureabreathtestingisthetestofchoiceforthispurpose.

You might also like